Em - 1
Em - 1
MATHEMATICS
Serial No. UNIT - EM1 Page No.
16. Matrices 1
1.0 INTRODUCTION
2.0 SPECIAL TYPE OF MATRICES
(i)
Serial No. UNIT - EM1 Page No.
(ii)
Matr ices
MATRICES
1.0 INTRODUCTION
A rectangular array of mn numbers (which may be real or complex) in the form of 'm' horizontal lines (called
rows) and 'n' vertical lines (called columns), is called a matrix of order m by n, written as m × n matrix.
In compact form, the above matrix is represented by A=[a ij]m×n. The number a11, a12,.....etc are known as the
elements of the matrix A, a ij belongs to the ith row and jth column and is called the (i, j) th element of the matrix
A = [aij].
é1 2 3 ù
E.g., A = ê ú is a matrix having 2 rows and 3 columns. Its order is 2 × 3 and it has 6 elements :
ë0 -1 9 û
é a11 ù
êa ú
Column Matrix (Column vector) – A = ê
12 ú
(b) i.e. column matrix has exactly one column.
ê : ú
ê ú
êëa m1 úû
(c) Zero or Null Matrix – (A = Om× n) An m × n matrix whose all entries are zero.
é0 0ù é0 0 0ù
A = ê0 ê0 0úú is 3 × 3 null matrix
ê 0úú is a 3 × 2 null matrix & B = ê 0
Node-1\Target-2020-21\1.JEE(M+A)\Module\Enthusiast\English\Maths\EM-1\16.Matrices\Th.&Ex.
é1 2 3 4ù
(d) Horizontal Matrix – A matrix of order m × n is a horizontal matrix if n > m e.g. ê ú
ë2 5 1 1û
é2 5ù
ê1 1 úú
(e) Vertical Matrix – A matrix of order m × n is a vertical matrix if m > n e.g. ê
ê3 6ú
ê ú
êë2 4úû
(f) Square Matrix – If number of rows = number of columns Þ matrix is a square matrix. If number of
rows = number of columns = n then, matrix is of the order 'n'.
Note – The pair of elements aij & aji are called Conjugate Elements.
1
JEE-Mathematics
æ 1 3 - 2ö æ 1 0 0ö
A= ç0 2 4 ÷ ; B = ç 2 - 3 0÷
ç ÷ ç ÷ Scalar Matrix Unit or Identity Matrix
è0 0 5 ø è 4 3 3ø
NOTE
(i) Minimum number of zeros in triangular matrix of order n = n(n–1)/2.
(ii) Minimum number of zero in a diagonal matrix of order n = n (n–1).
Illustration 1. Find the value of x, y, z and w which satisfy the matrix equation
Node-1\Target-2020-21\1.JEE(M+A)\Module\Enthusiast\English\Maths\EM-1\16.Matrices\Th.&Ex.
é x + 3 2y + x ù é - x - 1 0 ù
ê ú =ê ú
ë z - 1 4w - 8 û ë 3 2w û
Solution As the given matrices are equal so their corresponding elements are equal.
x+3=–x–1 Þ 2x = – 4
\ x=–2 ..........(i)
2y + x = 0 Þ 2y – 2 = 0 [from (i)]
Þ y=1 ...........(ii)
z–1=3 Þ z=4 ...........(iii)
4w – 8 = 2w Þ 2w = 8
\ w=4 ..........(iv) Ans.
2
Matr ices
6. 0 ALGEBRA OF MATRICES
Addition – A + B = [aij + bij] where A & B are of the same order.
(a) Addition of matrices is commutative
i.e. A + B = B + A
(b) Matrix addition is associative
(A + B) + C = A + (B + C)
(c) Additive inverse
If A + B = O = B + A, then B is called additive inverse of A.
(d) Existence of additive identity
Let A = [aij] be an m × n matrix and O be an m × n zero matrix, then A + O = O + A = A. In other
words, O is the additive identity for matrix addition.
(e) Cancellation laws hold good in case of addition of matrices. If A,B,C are matrices of the same order,
then A + B = A + C Þ B = C (left cancellation law) and B + A = C + A Þ B = C (right-
cancellation law)
Note – The zero matrix plays the same role in matrix addition as the number zero does in addition of numbers.
é1 3 ù é -1 -2ù
ê ú ê0 5ú
Illustration 2. If A = ê3 2 ú and B = ê ú and A + B – D = O (zero matrix), then D matrix will be-
êë2 5 úû êë 3 1 úû
é0 2ù é0 2ù é0 1 ù é 0 -2 ù
ê ú
(A) êê3 7úú (B) êê3 7úú (C) ê3 7ú (D) ê -3 -7 ú
ê ú
êë6 5úû êë5 6úû êë5 6úû êë -5 -6 úû
éa b ù
êc dú
Solution Let D = ê ú
êë e f úû
é1 3ù é -1 -2ù éa b ù
ê ú ê ú ê ú
\ A + B – D = ê3 2 ú + ê 0 5 ú - ê c d ú
êë2 5 úû êë 3 1 úû êë e f úû
Node-1\Target-2020-21\1.JEE(M+A)\Module\Enthusiast\English\Maths\EM-1\16.Matrices\Th.&Ex.
é1 - 1 - a 3 - 2 - b ù é0 0ù
ê3 + 0 - c 2 + 5 - d ú = ê0 0ú
Þ ê ú ê ú
êë2 + 3 - e 5 + 1 - f úû êë0 0úû
Þ –a = 0 Þ a = 0, 1 – b = 0 Þ b = 1,
3–c=0 Þ c = 3, 7 – d = 0 Þ d = 7,
5–e=0 Þ e = 5, 6 – f = 0 Þ f=6
é0 1 ù
ê ú
\ D = ê3 7ú Ans. (C)
êë5 6úû
3
JEE-Mathematics
é 2 -1 4ù é5 0 3 ù
1. If 2A + 2B = ê ú and A + 2B = ê1 6 2 ú , then 2B =
ë3 2 5û ë û
é 8 -1 2 ù é8 1 2ù é 8 1 -2 ù é8 1 2 ù
(A) ê ú (B) ê ú (C) ê ú (D) ê ú
ë -1 10 -1û ë -1 10 -1û ë -1 10 -1û ë1 10 1 û
é1 0 0 0ù
ê2 3 0 0 úú
2. If A = ê , then A is
ê4 5 6 0ú
ê ú
ë7 8 9 10 û
(A) a ij ¹ 0, for i > j (B) a ij = 0, for i > j (C) a ij = 0, for i < j (D) None of these
é0 0 ù é1 0 ù é0 1 ù é1 0 ù
(A) ê ú (B) ê ú (C) ê ú (D) ê ú
ë0 0 û ë0 0 û ë1 0 û ë0 1 û
é 5 -3ù é 6 -4 ù
5. A= ê ú and B = ê 3 6 ú , then A – B =
ë2 4 û ë û
é x 0 ù é -2 1 ù é 3 5ù é 2 4ù
6. If ê ú+ê ú=ê ú-ê ú , then
ë 1 y û ë 3 4 û ë6 3 û ë 2 1 û
(A) x = –3, y = –2 (B) x = 3, y = – 2 (C) x = 3, y = 2 (D) x = –3, y = 2
7. If I is a unit matrix of order 10, then the determinant of I is equal to
Node-1\Target-2020-21\1.JEE(M+A)\Module\Enthusiast\English\Maths\EM-1\16.Matrices\Th.&Ex.
é1 2 ù é 3 2 ù
8. If 2X – ê ú=ê ú , then X is equal to
ë 7 4 û ë 0 -2 û
é 2 2ù é 1 2ù é 2 2ù
(A) ê ú (B) ê ú (C) ê ú (D) None of these
ë7 4û ë7 / 2 2 û ë7 / 2 1 û
é1 0 ù é -1 1 ù
9. If A + B = ê ú and A – 2B = ê ú , then A =
ë1 1 û ë 0 -1û
é 1 1ù é 2 / 3 1/ 3 ù é 1/ 3 1/ 3ù
(A) ê ú (B) ê ú (C) ê ú (D) None of these
ë 2 1û ë 1/ 3 2 / 3û ë 2 / 3 1/ 3û
4
Matr ices
é1 2 ù é3 8ù
10. What must be the matrix X if 2X + ê ú = ê ú
ë3 4 û ë7 2û
é1 3 ù é 1 -3ù é2 6 ù é 2 -6ù
(A) ê ú (B) ê ú (C) ê ú (D) ê ú
ë 2 -1û ë 2 -1û ë 4 -2û ë 4 -2û
3
(AB)23 = a21b13 + a22 b23 + a23b33 = å a2r br 3
r =1
-----------------------------------------------------------------------
3
åa
Node-1\Target-2020-21\1.JEE(M+A)\Module\Enthusiast\English\Maths\EM-1\16.Matrices\Th.&Ex.
é 2 3 1ù é x ù
ê ú ê ú
Illustration 3. If [1 x 2] ê0 4 2 ú ê 1ú = O, then the value of x is :-
ëê0 3 2 ûú êë -1ûú
(A) –1 (B) 0 (C) 1 (D) 2
Solution The LHS of the equation
é xù
ê ú
= [2 4x + 9 2x + 5] ê 1ú = [2x + 4x + 9 – 2x – 5] = 4x + 4
êë -1úû
Thus 4x + 4 = 0 Þ x = –1 Ans. (A)
5
JEE-Mathematics
é1 2 ù é 3 2ù
Illust rat ion 4. If A, B are two matrices such that A + B = ê2 4 ú , A – B = ê -2 0ú then find AB.
ë û ë û
é 3 2ù
Solution Given A + B = é1 2 ù ........(i) & A – B = ê -2 0ú .......(ii)
êë2 4 úû ë û
Adding (i) & (ii)
é4 4 ù é2 2 ù
2A = ê0 4 ú Þ A = ê0 2 ú
ë û ë û
Subtracting (ii) from (i)
é -2 0 ù é -1 0 ù
2B = ê 4 4 ú Þ B = ê 2 2 ú
ë û ë û
é 2 2 ù é -1 0 ù é2 4 ù
Now AB = ê0 2 ú ê 2 2 ú = ê4 4 ú Ans.
ë û ë û ë û
é1 1 ù é1 0 ù é1 0 ù é1 1 ù
Let A= ê ú & B= ê ú ; then AB = ê ú ; BA = ê ú
ë0 0û ë0 0û ë0 0û ë0 0û
Þ AB ¹ BA (in general)
é1 1ù é -1 1 ù é1 1ù é -1 1 ù é0 0ù
Let A= ê ú &B= ê ú , then AB = ê ú ê ú = ê ú
ë2 2û ë 1 -1û ë2 2û ë 1 -1û ë0 0û
NOTE
If A and B are two non - zero matrices such that AB = O then A and B are called the divisors of zero. If A and
B are two matrices such that
(i) AB = BA then A and B are said to commute
(ii) AB = –BA then A and B are said to anticommute
(d) Distributivity
A(B + C) = AB + AC ù
Provided A,B & C are conformable for respective products
(A + B)C = AC + BC úû
é1 -3 2 ù é1 4 1 0ù é1 1 -1 -2ù
Illustration 5. Let A = êê2 1 -3 úú ,B = ê ú
ê2 1 1 1 ú & C =
ê
ê3 -2 -1 -1ú
ú be the matrices then, prove
êë 4 -3 -1úû êë1 -2 1 2úû êë2 -5 -1 0 úû
é1 -3 2 ù é1 4 1 0ù é -3 -3 0 1 ù
ê úê ú ê ú
We have AB = ê2 1 -3 ú ê2 1 1 1 ú = ê 1 15 0 -5 ú
êë4 -3 -1úû êë1 -2 1 2 úû êë -3 15 0 -5úû
3´ 4
é1 -3 2 ù é2 1 -1 -2ù é -3 -3 0 1 ù
ê úê ú ê ú
Now, AC = ê2 1 -3 ú ê3 -2 -1 -1ú = ê 1 15 0 -5 ú
êë4 -3 -1úû êë2 -5 -1 0 úû êë -3 15 0 -5 úû
3´ 4
Here, AB = AC though B is not equal to C. Thus cancellation law does not hold in general.
NOTE
(i) A m .A n = A m+n
(ii) (A m )n = Amn, where m,n Î N
(iii) If A and B are square matrices of same order and AB = BA then
(A + B)n = nC0An + nC1An–1B + nC2An–2B2 + ........... + nCnBn
Note that for a unit matrix I of any order , I m = I for all m Î N.
é 2 -2 -4 ù
ê ú
Illustration 6. Show that the matrix A = ê -1 3 4 ú is idempotent.
êë 1 -2 -3 úû
é 2 -2 -4 ù é 2 -2 -4 ù
ê -1 3 ú ê -1 3 4 ú
Solution 2
A = A.A = ê 4 ú × ê ú
êë 1 -2 -3 úû êë 1 -2 -3 úû
é2.2 + (-2).( -1) + ( -4).1 2( -2) + ( -2).3 + ( -4).( -2) 2.( -4) + ( -2).4 + ( -4).(-3) ù
ê (-1).2 + 3.( -1) + 4.1 ( -1).( -2) + 3.3 + 4.( -2) ( -1).( -4) + 3.4 + 4.(-3) úú
= ê
ëê1.2 + ( -2).( -1) + (-3).1 1.( -2) + ( -2).3 + ( -3).( -2) 1.( -4) + ( -2).4 + (-3).(-3) ûú
7
JEE-Mathematics
é 2 -2 -4 ù
= ê -1 3 4 úú = A
ê
êë 1 -2 -3 úû
Hence the matrix A is idempotent.
é 1 1 3ù
Illustration 7. Show that ê 5 2 6 ú is nilpotent matrix of order 3.
ê ú
êë -2 -1 -3úû
é 1 1 3ù
Solution Let A = êê 5 2 6 úú
êë -2 -1 -3 úû
é 1 1 3ù é 1 1 3ù é 1 + 5 - 6 1+2-3 3 + 6 - 9ù
2 ê ú ê 5 2 6ú ê ú
A = A.A = 5 2 6 × ê ú = ê5 + 10 - 12 5 + 4 - 6 15 + 12 - 18 ú
ê ú
ëê -2 -1 -3 ûú êë -2 -1 -3 úû êë -2 - 5 + 6 -2 - 2 + 3 -6 - 6 + 9 úû
é 0 0 0ù
ê 3 3 9ú
= ê ú
êë -1 -1 -3úû
é 0 0 0ù é 1 1 3ù é 0+0+0 0+0+0 0 + 0 + 0ù
A3 = A2.A = ê 3 3 9 ú × êê 5 2 6 úú = ê3 + 15 - 18 3 + 6 - 9 9 + 18 - 37ú
ê ú ê ú
êë -1 -1 -3úû ëê -2 -1 -3 ûú êë -1 - 5 + 6 -1 - 2 + 3 -3 - 6 + 9 úû
é0 0 0ù
= ê0 0 0ú = O
ê ú
ëê0 0 0úû
\ A3 = O i.e., Ak = O
Here k=3
é -5 -8 0 ù
Node-1\Target-2020-21\1.JEE(M+A)\Module\Enthusiast\English\Maths\EM-1\16.Matrices\Th.&Ex.
é -5 -8 0 ù é -5 -8 0 ù é 25 - 24 + 0 40 - 40 + 0 0 + 0 + 0 ù
2 ê ú ê 3 5 0ú ê -15 + 15 + 0 -24 + 25 + 0 0 + 0 + 0 ú
Solution A = A.A= × ê ú = ê
ê 3 5 0ú ú
êë 1 2 -1úû ê
ë 1 2 -1 úû êë -5 + 6 - 1 -8 + 10 - 2 0 + 0 + 1úû
é 1 0 0ù
= ê0 1 0ú = I
ê ú
êë0 0 1úû
8
Matr ices
é cos t sin t ù
2. If R(t) = ê- sin t cos t ú, then R(s). R(t) =
ë û
(A) R(s) + R(t ) (B) R (st ) (C) R(s + t) (D) None of these
é1 2ù
3. If M = ê ú and M 2 - lM - I 2 = 0 , then l =
ë 2 3û
(A) –2 (B) 2 (C) –4 (D) 4
é1 0 0 ù
ê ú
5. If A = ê 0 1 0 ú , then A2 =
êë a b -1úû
é 0 2 0ù é1 2 3 ù
ê 0 0 3ú ê ú
6. If A = ê ú and B = ê 3 4 5 ú , then the element of third row and third column in AB will be
êë -2 2 0 úû êë5 -4 0 úû
Node-1\Target-2020-21\1.JEE(M+A)\Module\Enthusiast\English\Maths\EM-1\16.Matrices\Th.&Ex.
é2 0 0ù
ê ú
7. If A = ê 0 2 0 ú , then A5 =
êë 0 0 2 úû
é 1 -3ù
8. If A = ê 2 k úû
and A2 – 4A + 10 I = A, then k =
ë
9
JEE-Mathematics
é -5 4 0 ù
ê ú
9. If A = [ 1 2 3] and B = ê 0 2 -1ú , then AB =
ëê 1 -3 2 úû
é -5 4 0 ù é 3ù é -5 8 0ù
ê ú ê ú ê 0 4 -3ú
(A) ê 0 4 -2 ú (B) ê1 ú (C) [ -2 -1 4] (D) ê ú
ëê 3 -9 6 úû ëê1 ûú ëê 1 -6 6 úû
é1/ 3 2 ù é3 6 ù
10. If A = ê ú ,B =ê ú and AB = I, then x =
ë 0 2 x - 3û ë 0 -1û
(A) -1 (B) 1 (C) 0 (D) 2
é 2 1ù
Illustration 10. If ƒ (x) = x2 – 3x + 3 and A = ê ú be a square matrix then prove that ƒ (A) = O. Hence
ë - 1 1û
find A4.
é 2 1ù é 2 1ù é 3 3 ù
Solution A 2 = A.A = ê úê ú=ê ú
ë-1 1û ë -1 1û ë -3 0û
é 3 3ù é 2 1ù é1 0 ù é 0 0 ù
Node-1\Target-2020-21\1.JEE(M+A)\Module\Enthusiast\English\Maths\EM-1\16.Matrices\Th.&Ex.
Hence A2 – 3A + 3I = ê ú - 3ê ú + 3ê ú =ê ú=O
ë -3 0 û ë -1 1û ë0 1 û ë0 0û
Aliter –
2-x 1
Q |A – XI| = 0 Þ =0
-1 1-x
Þ (2 – x)(1 – x) + 1 = 0 Þ x2 – 3x + 3 = 0 (characterstic polynomial)
by Cayley-Hamilton Theorem A2 – 3A + 3I = O. Hence proved.
Now A2 = 3A – 3I
squaring on both the sides
A4 = 9(A2 – 2A + I)
æ é 3 3ù é 4 2 ù é1 0 ù ö é3 - 4 + 1 3 - 2 ù é0 1ù é0 9ù
= 9çê ú-ê ú+ê ú÷=9 ê ú = 9ê ú =ê ú
è ë -3 0û ë -2 2 û ë0 1 û ø ë -3 + 2 -2 + 1û ë -1 -1û ë -9 -9 û
10
Matr ices
1
A–1 = -
an
(
a0 A n -1 + a1 A n -2 + ......a n -1 I )
é1 2 0ù
Illustration 11. If A = êê2 -1 0 úú , show that 5A–1 = A2 + A – 5I
êë0 0 -1úû
1- x 2 0
2 -1 - x 0
i.e. =0
0 0 -1 - x
i.e. x3 + x2 – 5x – 5 = 0
Using Cayley – Hamilton theorem
A3 + A2 – 5A – 5I = O Þ 5I = A3 + A2 – 5A
Multiplying by A–1, we get 5A–1 = A2 + A – 5I
Properties of transpose
Node-1\Target-2020-21\1.JEE(M+A)\Module\Enthusiast\English\Maths\EM-1\16.Matrices\Th.&Ex.
(a) (A+B)T = AT+BT ; note that A & B have the same order.
(b) (A B)T = BT AT (Reversal law) A & B are conformable for matrix product AB
Note – In general : (A1. A2,............ An)T = A nT . ............... . A 2T . A1T (reversal law for transpose)
(c) (AT)T = A
11
JEE-Mathematics
Illustration 12. If A and B are matrices of order m × n and n × m respectively, then order of matrix B T(AT)T
is -
(A) m × n (B) m × m (C) n × n (D) Not defined
é a1 a2 a3 ù é a1 b1 c1 ù
(ii) Let A = êb b2 b3 úú Þ A = êêa2
T
b2 c2 úú
ê 1
êë c1 c2 c3 úû êëa3 b3 c3 úû
åa 2
i = å b =å c
2
i
2
i = 1 and åa b = åb c = åc a
i i i i i i =0
i =1 i =1 i =1 i =1
é 0 2b gù
Illustration 13. Determine the values of a, b, g, when êa b -g ú is orthogonal.
êëa -b g úû
é 0 2b gù
Solution Let A = êa b -g ú
ê ú
êëa -b g úû
é 0 a aù
ê2b b -b úú
\ A' = ê
Node-1\Target-2020-21\1.JEE(M+A)\Module\Enthusiast\English\Maths\EM-1\16.Matrices\Th.&Ex.
ëê g -g g úû
But given A is orthogonal.
\ AAT = I
é 0 2b g ù é 0 a aù é 1 0 0ù
Þ êa ú ê
b -g ú ê2b b -b úú = ê0 1 0ú
ê ê ú
êëa -b g úû êë g -g g úû êë0 0 1úû
12
Matr ices
Equating the corresponding elements, we have
4b2 + g2 = 1 ........(i)
2 2
2b – g = 0 ........(ii)
a2 + b2 + g2 = 1 ........(iii)
1 1
From (i) and (ii), 6b2 = 1 \ b2 = and g2 =
6 3
1 1 1
From (iii) a2 = 1 – b2 – g2 = 1 – – =
6 3 2
1 1 1
Hence, a=± ,b=± and g = ± Ans.
2 6 3
1 1 1 T 1
A= (A + A T ) + (A - A T ) and A = (A + A) - (A T - A)
2
144244 3 2
144244 3 2 2
symmetric skew symmetric
13
JEE-Mathematics
é4 6 -1ù é2 4ù é3 ù
ê3 0 2 ú ê0 1 ú ê1 ú
1. A= ê ú,B= ê ú,C= ê ú , then the expression which is not defined is
ëê1 -2 5 úû ëê -1 2 úû ëê 2 ûú
(A) A2 + 2B – 2A (B) CC¢ (C) B¢C (D) AB
2. If A is involutory matrix and I is unit matrix of same order, then (I - A)(I + A) is
(A) Zero matrix (B) A (C) I (D) 2A
é 1/ 2 1/ 2 ù
3. The matrix A = ê ú is
ëê- 1 / 2 - 1 / 2 ûú
(A) Involutory (B) Orthogonal (C) Nilpotent (D) None of the above
4. The product of a matrix and its transpose is an identity matrix. The value of determinant of this matrix can be
(A) – 1 only (B) 0 only (C) +1 or –1 only (D) 1 only
é 1 - 3 - 4ù
ê ú
5. The matrix A = ê - 1 3 4 ú is nilpotent of index
êë 1 - 3 - 4 úû
æ -3 -2 ö æ -3 10 ö æ -3 7 ö
(A) çç 10 ÷
7 ÷ø
(B) çç - 2 ÷
7 ÷ø
(C) çç 10 2 ÷÷ (D) None of these
è è è ø
é0 5 - 7ù
ê ú
- 5 0 11 ú
7. The matrix ê is known as
êë 7 - 11 0 úû
é1 1ù -1 é 1 1ù
(A) A¢ = ê1 - 1ú (B) A = ê - 1 1ú
ë û ë û
Node-1\Target-2020-21\1.JEE(M+A)\Module\Enthusiast\English\Maths\EM-1\16.Matrices\Th.&Ex.
é 1 1ù él -l ù
(C) A. ê - 1 1ú = 2 I (D) lA = ê 1 - 1ú where l is a non zero scalar
ë û ë û
é1 -2ù
10. If A = ê5 ú , then A + AT equals
3û
ë
é2 3ù é2 -4 ù é 2 4ù
(A) ê 3 6 ú (B) ê10 6û
ú (C) ê - 10 6 ú (D) None of these
ë û ë ë û
é 0 1 -2 ù
ê ú
11. If A = ê -1 0 5 ú , then
êë 2 -5 0 úû
14
Matr ices
é1 2 2ù
1ê ú
A = ê 2 1 - 2ú
14. The matrix 3 is
êë - 2 2 - 1 úû
é 8 -6 2 ù
ê ú
Where A = ê -6 7 -4 ú and also prove that A3 – 18A2 + 45A = O..
êë 2 -4 3 úû
T
é C11 C12 C13 ù
Then, adjA = [Cij] T Þ adjA = ê C23 C22 C23 ú
ú
ê
êë C31 C32 C33 úû
æ|A| 0 0 ö æ1 0 0 ö
ç ÷ ç ÷
0 |A| 0 =|A|
ç ÷ ç0 1 0 ÷ Þ A. (Adj. A) = | A | I
ç 0 0 |A|÷ø ç0 0 1 ÷
è è ø
(whatever may be the value only |A| will come out as a common element)
A. (adj. A)
If | A | ¹ 0 then = I = unit matrix of the same order as that of A
|A|
Node-1\Target-2020-21\1.JEE(M+A)\Module\Enthusiast\English\Maths\EM-1\16.Matrices\Th.&Ex.
ép q ù é s -q ù
e.g. If A = ê ú Þ adjA = ê ú
ër sû ë -r p û
15
JEE-Mathematics
é 1 3 5ù
Illustration 15. If A = êê3 5 1úú , then adj A is equal to -
êë5 1 3 úû
T
é 14 -4 -22ù é 14 -4 -22 ù
ê -4 -22 14 ú ê -4 -22 14 ú
Solution adj. A = ê ú = ê ú Ans. (A)
êë -22 14 -4 úû ëê -22 14 -4 ûú
é2 0 0 ù
Illustration 16. If A = êê2 2 0 úú , then adj (adj A) is equal to -
êë2 2 2 úû
é1 0 0ù é1 0 0ù é1 0 0ù
ê1 1 0ú ê 0 úú ê ú
(A) 8 ê ú (B) 16 ê1 1 (C) 64 ê1 1 0ú (D) none of these
êë1 1 1úû êë1 1 1úû êë1 1 1úû
2 0 0
Solution |A| = 2 2 0 = 8
2 2 2
Now adj (adj A) = | A| 3–2 A
é2 0 0ù é1 0 0ù
ê2 2 0ú ê ú
= 8 ê ú = 16 ê1 1 0ú Ans. (B)
ëê2 2 2úû êë1 1 1úû
A -1 = B Û AB = I = BA
We have, A.(adj A) =|A | I n
Node-1\Target-2020-21\1.JEE(M+A)\Module\Enthusiast\English\Maths\EM-1\16.Matrices\Th.&Ex.
A -1 . A(adj A) = A -1 In |A|
In (adj A) = A -1 |A|I n
(adj A)
\ A –1 =
|A|
Note – The necessary and sufficient condition for a square matrix A to be invertible is that |A|¹ 0
Properties of inverse
(i) If A & B are invertible matrices of the same order, then (AB) –1 = B –1 A –1 .
Note–If A1, A2,...............An are all invertible square matrices of order n
then (A1A2..............An)–1 = An–1 An–1–1............A2–1 A1–1
16
Matr ices
(b) (A k )-1 = (A -1 )k = A - k ; k Î N
-1 -1
(iii) If A is invertible, (a) (A ) = A
(iv) If A is non-singular matrix, then |A–1| = |A|–1
(v) If idempotent matrix is invertible then its inverse will be identity matrix.
(vi) A nilpotent matrix will not be invertible because its determinant value is zero.
(vii) Orthogonal matrix A is always invertible and A –1 = AT.
(viii) A = A –1 for an involutary matrix.
Cancellation law – Let A,B,C be square matrices of the same order 'n'.
If A is a non-singular matrix, then
(a) AB = AC Þ B = C (Left cancellation law)
(b) BA = CA Þ B = C (Right cancellation law)
Note that these cancellation laws hold only if the matrix 'A' is non-singular (i.e.|A|¹ 0).
Illustration 17. Prove that if A is non-singular matrix such that A is symmetric then A–1 is also symmetric.
Solution AT = A [Q A is a symmetric matrix]
(AT)–1 = A–1 [since A is non-singular matrix]
Þ (A–1)T = A–1 Hence proved
-1
é 1 - tan q / 2 ù é 1 tan q / 2 ù
Illustration 18. ê ú ê - tan q / 2 ú is equal to -
ë tan q / 2 1 û ë 1 û
é2 - 2 ù é 1 / 3 1 / 3ù
(A) ê ú (B) ê ú
ë2 1 û ë -1 / 3 1 / 6 û
é1 / 3 -1 / 3 ù é 1 / 3 -1 / 3 ù
(C) ê ú (D) ê ú
ë1 / 3 1 / 6 û ë -1 / 3 1 / 6 û
é0 1ù
é0 -1 2 ù ê1 0úú é 1 2ù
Solution : M= ê ú ê = ê ú
ë2 -2 0 û ê1 1 úû ë -2 2 û
ë
é2 -2 ù
|M| = 6 , adj M = ê ú
ë2 1û
1 é2 -2 ù é1 / 3 -1 / 3 ù
\ M–1 = 6 ê ú=ê ú Ans. (C)
ë2 1 û ë1 / 3 1 / 6 û
17
JEE-Mathematics
é a1 x + b1 y + c1 z ù é d1 ù é a1 b1 c1 ù é x ù é d1 ù
ê ú ê ú ê c2 úú êy ú = ê d ú
Þ ê a2 x + b 2 y + c 2 z ú = ê d 2 ú Þ ê a2 b2 ê ú ê 2ú
êë a3 x + b3 y + c 3 z úû êë d 3 úû êë a3 b3 c3 úû êë z úû êë d3 úû
Þ AX=B ....(i)
Multiplying adjA on both the sides of (i)
Þ (adjA) AX = (adjA)B Þ |A|X = (adjA) B
|A | X = (adjA )B
If |A | ¹ 0 If |A |=0
(adjA )B
X =
|A |
or (adjA )B = null m atrix If (adj A )B ¹ 0
X = A B– 1
(unique solution) Infinite solutions
Inconsistent (no solutio n)
or no solutio n
é x ' ù é -1 0 ù é x ù
In terms of matrices, they can be written as ê ú =ê úê ú
ë y 'û ë 0 1û ë y û
éxù é -1 0 ù
Hence the reflection about y-axis is obtained by pre-multiplying the column vector ê ú with ê ú.
ëy û ë 0 1û
é1 0 ù
Similarly the reflection in x-axis is obtained by pre-multiplying with ê ú
ë 0 - 1û
(b) Reflection about the line y = x
Since y' = x x' = y
é x ' ù é0 1 ù é x ù
and so ê ú=ê úê ú
ë y ' û ë1 0 û ë y û
18
Matr ices
Similarly the reflection of (x,y) about the line y = –x is obtained by x' = –y, y' = –x
é x ' ù é 0 -1ù é x ù
or ê ú=ê úê ú
ë y ' û ë -1 0 û ë y û
(c) Rotation about origin – If the coordinates of a point P are (x, y), O being the origin and the line OP is
rotated about O as centre through an angle a in the anti-clockwise direction, then the new coordinates of
P are
x' = r cos(q + a) = r cos q cos a – r sin q sina
= x cos a – y sin a.
y' = r sin(q + a ) = r sin q cos a + r cos q sin a
= y cos a – x sin a
x+ y+ z = 6
x- y+ z = 2
Illustration 20. Solve the system using matrix method.
2x + y - z = 1
é 1 1 1ù éxù é6ù
ê 1 - 1 1ú êy ú ê ú
Solution Let A = ê ú , X = ê ú & B = ê2ú
êë2 1 -1úû êë z ûú êë1 úû
é0 3 3ù
ê 1úú
Cofactor A = ê2 -3
êë2 0 -2úû
é0 2 2ù
Node-1\Target-2020-21\1.JEE(M+A)\Module\Enthusiast\English\Maths\EM-1\16.Matrices\Th.&Ex.
ê3 -3 0úú
adj A = ê
êë3 1 -2 úû
é0 2 2ù é 0 1/ 3 1/ 3 ù
1 1 ê ê
–1
A = adj A = 3 -3 ú
0ú = ê1 / 2 -1 / 2 0 úú
A 6 ê
êë3 1 -2 úû ëê1 / 2 1 / 6 -1 / 3 ûú
é 0 1/ 3 1/ 3 ù é6ù éxù é1 ù
ê1 / 2 -1 / 2 0 úú ê2ú
X=A B= ê–1
ê ú i.e. êê y úú = ê2ú
ê ú
êë1 / 2 1 / 6 -1 / 3 úû êë1 úû êë z úû êë3ûú
Þ x = 1, y = 2, z = 3 Ans.
19
JEE-Mathematics
é 3 - 2 - 1ù
1. Inverse of the matrix êê - 4 ú
1 - 1ú is
êë 2 0 1 úû
é1 2 3 ù é1 - 3 5ù é 1 2 3 ù é1 2 - 4 ù
ê ú ê ú ê ú ê ú
(A) ê 3 3 7 ú
(B) ê7 4 6 ú (C) ê 2 5 7 ú
(D) ê8 - 4 - 5 ú
êë - 2 - 4 - 5 úû êë 4 2 7 úû êë - 2 - 4 - 5 úû êë 3 5 2 úû
(A) ( AB)-1 = A-1B-1 (B) AB = BA (C) ( AB)¢ = A¢B¢ (D) ( AB)-1 = B-1 A-1
é- 4 - 3 - 3ù
N = êê 1 0 1ú
ú
3. Adjoint of the matrix is
êë 4 4 3 úû
-1 adj A
(A) ( AB)¢ = A¢B¢ (B) ( AB)¢ = B¢A¢ (C) A = (D) ( AB)-1 = A-1B-1
A
éi 0 ù
7. If A = ê0 i / 2ú (i = - 1), then A-1 =
ë û
éi 0 ù é -i 0 ù éi 0ù é0 iù
(A) ê0 i / 2ú (B) ê 0 - 2iú (C) ê0 2iú (D) ê 2i 0 ú
ë û ë û ë û ë û
é1 2 1ù
9. The element of second row and third column in the inverse of ê 2 ú
1 0ú is
ê
êë - 1 0 1úû
é1 0 0 ù
10. The inverse of the matrix êê0 1 0 úú is
ëê0 0 1 ûú
é0 0 1 ù é1 0 0 ù é0 1 0 ù é1 0 0 ù
(A) êê0 1 0 úú (B) êê0 1 0 úú (C) ê0 0 1 ú (D) êê0 0 1 úú
ê ú
êë1 0 0 úû êë0 0 1 úû êë1 0 0 úû êë0 1 0 úû
20
Matr ices
é1 0 0ù
11. Let A = êê 5 ú
2 0ú , then the adjoint of A is
êë - 1 6 1úû
é 2 - 5 32 ù é - 1 0 0ù é-1 0 0ù
ê ú ê ú ê ú
(A) ê0 1 - 6 ú (B) ê - 5 - 2 0 ú (C) ê - 5 - 2 0 ú (D) None of these
êë0 0 2 úû êë 1 - 6 1 úû êë 1 - 6 - 1úû
é 1 2 3ù
ê ú
13. The inverse of ê0 1 2ú is
êë0 0 1 úû
é1 - 2 1 ù é1 - 2 1 ù é1 2 1 ù
ê ú
(A) êê0 1 - 2ú
ú
(B) ê0 1 - 2ú (C) êê0 1 2úú (D) None of these
êë0 0 0 úû êë0 0 1 úû êë0 0 1 úû
21
JEE-Mathematics
SINGLE CORRECT
é1 5 ù é -2 5 ù
1. If A – 2B = ê ú and 2A – 3B = ê 0 7 ú , then matrix B is equal to -
ë3 7 û ë û
é -4 -5 ù é 0 6ù é 2 - 1ù é6 -1ù
(A) ê ú (B) ê ú (C) ê ú (D) ê ú
ë -6 -7 û ë -3 7 û ë3 2 û ë0 1 û
é cos a sin a ù
2. If Aa = ê ú , then AaAb is equal to -
ë - sin a cos a û
é1 1 ù é 1 2 ù é1 3 ù é1 n ù é1 378 ù
6. If the product of n matrices ê úê úê ú ...... ê ú is equal to the matrix ê0 1 ú then the
ë 0 1 û ë 0 1 û ë0 1 û ë0 1 û ë û
value of n is equal to -
(A) 26 (B) 27 (C) 377 (D) 378
é 0 1ù 2
7. If A = ê ú and (aI2 +bA) = A , then -
ë - 1 0 û
(B) Every element of A and it's co-factor are additive inverse of each other.
(C) Every element of A and it's co-factor are multiplicative inverse of each other.
(D) None of these
10. Which of the following is an orthogonal matrix -
é 6 / 7 2 / 7 -3 / 7 ù é6 / 7 2 / 7 3 / 7 ù
ê ú ê ú
(A) ê2 / 7 3 / 7 6 / 7 ú (B) ê2 / 7 -3 / 7 6 / 7 ú
ëê3 / 7 -6 / 7 2 / 7 ûú êë3 / 7 6 / 7 -2 / 7 úû
é -6 / 7 -2 / 7 -3 / 7 ù é 6 / 7 -2 / 7 3 / 7 ù
ê ú ê ú
(C) ê 2 / 7 3 / 7 6 / 7 ú (D) ê 2 / 7 2 / 7 -3 / 7 ú
ëê -3 / 7 6 / 7 2 / 7 ûú ëê -6 / 7 2 / 7 3 / 7 ûú
22
Matr ices
11. If A is an orthogonal matrix & | A | = –1, then AT is equal to -
(A) –A (B) A (C) –(adj A) (D) (adj A)
12. det. {adj (2In)} is equal to
(A) 2n – 1 (B) 2 (C) 2n (D) 2n(n – 1)
é3 2ù é3 1 ù
13. Let the matrix A and B be defined as A = ê ú and B = ê ú then the value of Det.(2A9B–1), is -
ë2 1û ë7 3 û
é2 1 ù é - 3 2 ù é1 0 ù
14. If ê úAê ú=ê ú , then matrix A equals -
ë 7 4 û ë 5 -3 û ë 0 1 û
é 7 5ù é2 1 ù é 7 1ù é 5 3ù
(A) ê ú (B) ê ú (C) ê ú (D) ê ú
ë -11 -8 û ë5 3û ë34 5 û ë13 8 û
é0 5 ù
15. If A = ê ú and ¦(x) = 1 + x + x + ...... + x , then ¦(A) =
2 16
ë0 0 û
é1 5 ù é1 5 ù é0 5 ù
(A) 0 (B) ê ú (C) ê ú (D) ê ú
ë0 1 û ë0 0 û ë1 1 û
é1 2 ù 2
16. If M = ê ú and M –lM – I2 = O , then l equals -
ë2 3 û
é1 - 2 ù é -1 4 ù é4 8 ù
17. If A = ê ú,B = ê ú and ABC = ê ú , then C equals -
ë3 0 û ë 2 3 û ë3 7û
é a 0 0 ù
ê ú
19. If A = ê 0 a 0 ú , then | A | | Adj A | is equal to -
êë 0 0 a úû
é2 3 ù
20. If A = ê ú , then 19A–1 is equal to -
ë 5 -2 û
1
(A) AT (B) 2A (C) A (D) A
2
23
JEE-Mathematics
é0 0 1 ù
25. If A = êê0 1 0úú , then-
êë1 0 0úû
é 0 0 -1ù
(A) AdjA is zero matrix (B) Adj A = êê 0 -1 0 úú
êë -1 0 0 úû
24
Matr ices
é x1 ù éxù
ê ú = Aê ú
ë y1 û ëy û
é x2 ù é x ù é x3 ù é x ù é x4 ù é xù
Similarly ê ú = B ê ú, ê ú = C ê ú, ê ú = D ê ú
y
ë 2û y
ë û ë 3ûy y
ë û ë 4ûy ëyû
represents the reflection of point (x, y) in y-axis, origin and the line y = x respectively.
On the basis of above information, answer the following questions
1. The value of A + B + C + D is -
é -1 1 ù é -1 1 ù é1 0 ù é0 1 ù
(A) ê ú (B) ê ú (C) ê ú (D) ê ú
ë 0 0û ë 1 - 1û ë 0 - 1û ë1 0 û
2. Let X be a square matrix given by X = A + AD2 + AD4 +.........+AD2n – 2, then X is -
é -n 0 ù é n 0ù én 0 ù é -n 0ù
(A) ê ú (B) ê ú (C) ê ú (D) ê ú
ë 0 -n û ë0 n û ë0 -n û ë 0 nû
éxù éa ù
3. Let P(a, b) be a point & ê ú = DCBA ê ú then Q(x, y) represents the reflection of point P(a, b) in -
ëy û ëbû
(A) x-axis (B) y-axis (C) origin (D) line y = x
Comprehension – 2
é 2 -2 -4 ù é -4 -3 -3ù
ê ú
If A 0 = ê -1 3 4 ú and B 0 = êê 1 0 1 úú
ëê 1 -2 -3 úû êë 4 4 3 úû
Node-1\Target-2020-21\1.JEE(M+A)\Module\Enthusiast\English\Maths\EM-1\16.Matrices\Th.&Ex.
25
JEE-Mathematics
Comprehension – 3
æ 1 3 l + 2ö æ 3 1 4ö
ç
Let A and B are two matrices of same order 3 × 3, where A = 2 4 ÷ ç ÷
ç 8 ÷ , B = ç 3 2 5÷ .
ç 3 5 10 ÷ ç 2 1 4÷
è ø è ø
7. If A is singular matrix, then tr(A + B) is equal to
(A) 6 (B) 12 (C) 24 (D) 17
8. If matrix 2A + 3B is singular, then the value of 2l is
(A) –111 (B) –113 (C) –115 (D) –117
1
9. If l = 3, then (tr(AB) + tr(BA)) is equal
7
(A) 234/7 (B) 233/7 (C) 231/7 (D) 232/7
Comprehension – 4
Consider some special type of matrices.
A square matrix A is said to be an Idempotent Matrix if A2 = A
A square matrix A is said to be a Nilpotent Matrix if Ak = Null Matrix, for k Î N .
A square matrix is said to be an Involutary Matrix, if A2 = I.
Consider the following matrices
é 2 -3 -5ù é 1 -3 -4 ù é0 1 -1ù
A = ê -1 4 5 ú ;B = ê -1 3 4 ú ;C = êê 4 -3 4 úú
ê ú ê ú
êë 1 -3 -4 úû êë 1 -3 -4 úû êë3 -3 4 úû
é1 1 3ù
ê5 2 6ú
(D) ê ú (s) Orthogonal
êë -2 -1 -3úû
26
Matr ices
14. Co lu mn - I Column-II
(A) If A is a square matrix of order 3 and (p) 6
æ Aö
det A = 162 then det ç ÷ =
è 3ø
(B) If A is a matrix such that A2 = A and (q) 5
2l + 1
(I + A)5 = I + lA then
7
é4 3ù 2
(C) If A = ê ú and A – xA + yI = 0 (r) 0
ë2 5 û
then y – x =
é 9 10 11 12 ù
ê ú
(D) If A = ê13 14 15 16 ú and (s) 9
êë17 18 19 20 ûú
é1 3 5 7 ù
ê -3 -3 -10 -10ú
ê ú
B = ê 5 10 5 0 ú then (AB)23
ê ú
êë 7 10 0 7 úû
15*. Consider a square matrix A of order 2 which has 4 distinct elements as 0, 1, 2 and 4.Let N denote the number
of such matrices, all elements of which are distinct.
Column-I Column-II
(A) Possible non-negative value of det(A) is (p) 2
(B) Sum of values of determinants corresponding to N matrices is (q) 4
(C) If absolute value of (det(A)) is least, then possible value of |adj(adj(adj A)))| (r) –2
(D) If det(A) is algebraically least, then possible value of det(4A –1) is (s) 0
(t) 8
é 3x 2 ù é( x + 2 ) 2 5x 2 2x ù
ê ú ê ú
16. Let A = ê 1 ú , B = [a b c] and C = ê 5x 2 ( x + 2 )2 ú be three given matrices, where a, b, c
Node-1\Target-2020-21\1.JEE(M+A)\Module\Enthusiast\English\Maths\EM-1\16.Matrices\Th.&Ex.
2x
ê 6x ú ê ú
ë û ê 2x
ë
( x + 2 )2 5x 2 ú
û
and x Î R. Given that tr(AB) = tr(C) " x Î R , where tr(A) denotes trace of A. Find the value of (a + b + c).
27
JEE-Mathematics
18. If A is an idempotent matrix and I is an identity matrix of the same order, find the value of n, n ÎN, such that
(A + I)n = I + 127 A.
é 0 1 -1ù
ê ú
19. Let X be the solution set of the equation Ax = I, where A = ê 4 -3 4 ú and I is the corresponding unit
êë 3 -3 4 úû
(a) aij is 1 or –1 ; 2
(b) a11 2
+ a12 2
= a21 2
+ a22 = 2;
21. Let A be a 3 × 3 matrix such that a 11 = a33 = 2 and all the other aij = 1. Let A–1 = xA2 + yA + zI, then find
the value of (x + y + z) where I is a unit matrix of order 3.
é1 2 2ù é2 1 1ù é10 ù
ê 3 úú , C = ê2 1 úú , D = ê13 ú
22. Given that A = ê 2 2 ê 2 ê ú and that Cb = D. Find the sum of all element
êë 1 -1 3 úû êë 1 1 1 úû êë 9 úû
in x, if Ax = b.
Node-1\Target-2020-21\1.JEE(M+A)\Module\Enthusiast\English\Maths\EM-1\16.Matrices\Th.&Ex.
28
Matr ices
é x z ù é1 -1ù é3 6ù
1. Find x,y,z,t if 2 ê ú + 3ê ú = 3ê ú
ë y t û ë0 2 û ë4 6û
2. If A is any m × n such that AB and BA are both defined show that B is an n × m matrix.
3. A,B are two matrices such that AB and A + B are both defined; show that A,B are square matrices of the
same order.
4. Show that the elements on the main diagonal of a skew-symmetric matrix are all zero.
é x - y 2z + w ù é 5 3 ù
5. Find x,y,z and w such that ê ú=ê ú
ë 2x - y 2x + w û ë12 15û
é 2 -1ù é1 4 ù
6. If A = ê ú and B = ê ú , find 3A –2B.
ë3 1 û ë7 2 û
7. If A = diag (1, –1, 2) and B = diag (2, 3, – 1), find A + B, 3A + 4B.
é 2 3 ù é 3 -6 ù
8. Find a matrix A, if A + ê ú=ê ú
ë -1 4û ë -3 8 û
é 2 3 -5ù é 0 5 1ù
11. If A = ê ú and B = ê ú , find A + B and A – B.
ë1 2 -1û ë -2 7 3û
12. Prove that the product of matrices
p
multiple of .
2
é 1 3 2ù é 1 ù
1 x 1] êê 2 5 1 úú êê 2 úú = 0
Find the value of x such that [
Node-1\Target-2020-21\1.JEE(M+A)\Module\Enthusiast\English\Maths\EM-1\16.Matrices\Th.&Ex.
13.
ëê15 3 2ûú êë x ûú
é 1 0ù é1 0 ù
14. If A = ê ú and I = ê ú , then find k so that A 2 = 8A + kI
ë -1 7 û ë0 1 û
16. If A and B are square matrices of order n, then prove that A and B will commute iff A –l I and and B – l I
commute for every scalar l.
17. Use matrix multiplication to divide Rs. 30,000 in two two parts such that the total annual interest at 9% on
the first part and 11% on the second part amounts Rs.3060.
29
JEE-Mathematics
é-1ù
A = êê 2 úú
18. If and B = [– 2 –1 – 4], verify that (AB)T = BT A T
êë 3 úû
19. Let A and B be symmetric matrices of the same order. Then, show that
(i) A + B is a symmetric matrix
(ii) AB – BA is a skew-symmetric matrix
(iii) AB + BA is a symmetric matrix
é-1 2 ù é3 -2 ù
20. Find a matrix X such that 2A + B + X = 0, where A = ê ú ,B = ê ú
ë 3 4û ë1 5 û
é2 3ù
é1 -2 3 ù B = êê-1 2 úú
21. Let A = ê ú and . Find AB and BA and show that AB ¹ BA .
ë3 2 -1û êë 4 -5úû
é2x + 1 3y ù é x + 3 y 2 + 2ù
A=ê , B =
ë 0 y 2 - 5 y úû ê 0
ë - 6 úû
é-2 2 0 ù é 2 0 -2ù
23. Find a matrix A such that 2A – 3B + 5C = O, where B = ê ú and C = ê ú
ë 3 1 4û ë7 1 6 û
é 0 1ù
24. If A = ê ú ,find x and y such that (xI + yA ) 2 = A.
ë -1 0 û
Node-1\Target-2020-21\1.JEE(M+A)\Module\Enthusiast\English\Maths\EM-1\16.Matrices\Th.&Ex.
30
Matr ices
ANSWER KEY
BEGIINNER'S BOX - 1
1. (B) 2. (C) 3. (B) 4. (D) 5. (B) 6. (B)
7. (B) 8. (C) 9. (C) 10. (A)
BEGIINNER'S BOX - 2
1. (A) 2. (C) 3. (D) 4. (C) 5. (A)
6. (B) 7. (C) 8. (C) 9. (C) 10. (B)
BEGIINNER'S BOX - 3
1. (A) 2. (A) 3. (C) 4. (C) 5. (A) 6. (B)
7. (B) 8. (D) 9. (C) 10. (A) 11. (B) 12. (B)
13. (B) 14. (A) 15. l = 0, 3 and tr(A) = 18, |(A)| = 0
BEGIINNER'S BOX - 4
1. (C) 2. (D) 3. (A) 4. (B) 5. (B) 6. (B)
7. (B) 8. (C) 9. (B) 10. (B) 11. (D) 12. (A)
13. (B) 14. (A) 15. (D)
EXERC IS E-1
SINGLE CORR ECT & MORE THAN ONE OPTION CORR ECT
Que. 1 2 3 4 5 6 7 8 9 10
Ans. A A A C C B B D A A
Que. 11 12 13 14 15 16 17 18 19 20
Ans. C D D A B D B B D D
Que. 21 22 23 24 25 26 27 28 29
Ans. ABC AB ABD A B CD ABC A CD AB BC
EXERC IS E-2
l Comprehension Based Questions
Node-1\Target-2020-21\1.JEE(M+A)\Module\Enthusiast\English\Maths\EM-1\16.Matrices\Th.&Ex.
Comprehension – 1 1. B 2. C 3. D
Comprehension – 2 4. C 5. C 6. D
Comprehension – 3 7. C 8. C 9. A
Comprehension – 4 10. B 11. D 12. D
l Match the Column 13. (A) ® (p), (B) ® (q), (C) ® (s), (D) ® (r)
14. (A) ® (p), (B) ® (s), (C) ® (q), (D) ® (r)
15. (A) ® (p,q,t), (B) ® (s), (C) ® (p,r), (D) ® (r)
31
JEE-Mathematics
é 4 -11ù
6. ê -5 -1 ú 7. diag (3, 2, 1); diag (11, 9, 2)
ë û
é 1 -9 ù
8. ê -2 4 ú 9. P is a symmetric matrix.
ë û
é 2 -2 -6 ù
10. It can be prove that AT A is symmetric. 11. ê 3 -5 -4 ú 14. k = – 7.
ë û
15. Thus the given matrix equation is true if the matrices A and B commute with each other.
22. AB ¹ BA
Node-1\Target-2020-21\1.JEE(M+A)\Module\Enthusiast\English\Maths\EM-1\16.Matrices\Th.&Ex.
32
Set, Relation & Function
(i) Roster Method : In this method a set is described by listing elements, separated by commas and
enclose then by curly brackets
Ex. The set of vowels of English Alphabet may be described as {a, e, i, o, u}
(ii) Set Builder From : In this case we write down a property or rule p Which gives us all the element
of the set
A = {x : P(x)}
Illustration 2. A = {x : x Î N and x = 2n for n Î N}
i.e. A = {2, 4, 6, ....}
Illustration 3. B = {x2 : x Î z}
i.e. B = {0, 1, 4, 9, ....}
Subsets : Let A and B be two sets if every element of A is an element B, then A is called a subset of
B if A is a subset of B. we write A Í B
Illustration 11. A = {1, 2, 3, 4} and B = {1, 2, 3, 4, 5, 6, 7} Þ A Í B
The symbol ''Þ'' stands for "implies"
Proper subset : If A is a subset of B and A ¹ B then A is a proper subset of B. and we write A Ì B
Note-1 : Every set is a subset of itself i.e. A Í A for all A
Note-2 : Empty set f is a subset of every set
Note-3 : Clearly N Ì W Ì Z Ì Q Ì R Ì C
Note-4 : The total number of subsets of a finite set containing n elements is 2n
Universal set : A set consisting of all possible elements which occur in the discussion is called a Universal
set and is denoted by U
Note : All sets are contained in the universal set
Illustration 12. If A = {1, 2, 3}, B = {2, 4, 5, 6}, C = {1, 3, 5, 7} then
U = {1, 2, 3, 4, 5, 6, 7} can be taken as the Universal set.
Power set : Let A be any set. The set of all subsets of A is called power set of A and is denoted by
P(A)
Illustration 13. Let A = {1, 2} then P(A) = {f, {1}, {2}, {1, 2}}
Illustration 14. Let P(f) = {f}
Q P(P(f)) = {f, {f}}
Q P(P(P(f)) = {f, {f}, {{f}}, {f, {f}}
Note-1 : If A = f then P(A) has one element
Note-2 : Power set of a given set is always non empty
A ÈB
A È B = { x ; x Î A or x Î B }
(ii) Intersection of two sets : The intersection of two sets is represented by A Ç B or AB. It contains those
all elements which are contained in both sets A and B both, So
A B
A ÇB
A Ç B = { x ; x ÎA and x Î B }
34
Set, Relation & Function
(iii) Difference of two sets : If A and B are two sets then A–B represents the set of those elements which
are in A and not in B. In the same manner B–A represents the set of those elements which are in B and
not in A. So
A B
A-B B–A
or or
– –
AÇ B BÇA
A – B = A Ç B = {x ; x Î A and x Ï B}
2.0 RELATION
INTRODUCTION :
35
JEE-Mathematics
10 - x
Solution. We have (x, y) Î R Û x + 2y = 10 Û y = , x, y Î A
2
where A = {1, 2, 3, 4, 5, 6, 7, 8, 9, 10}
10 - 1 9
Now, x=1Þy= = Ï A.
2 2
This shows that 1 is not related to any element in A. Similarly we can observe. that 3, 5, 7, 9 and 10 are not
related to any element of A under the defined relation
Further we find that :
10 - 2
For x = 2, y = =4ÎA \ (2, 4) Î R
2
10 - 4
For x = 4, y = =3ÎA \ (4, 3) Î R
2
10 - 6
For x = 6, y = =2ÎA \ (6, 2) Î R
2
10 - 8
For x = 8, y = =1ÎA \ (8, 1) Î R
2
Thus R = {(2, 4), (4, 3), (6, 2), (8, 1)}
Þ R–1 = {(4, 2), (3, 4), (2, 6), (1, 8)}
Clearly, Dom(R) = {2, 4, 6, 8} = Range(R –1)
and Range (R) = {4, 3, 2, 1} = Dom(R–1)
(1) Void Relation : Let A be a set. Then f Í A × A and so it is a relation on A. This relation is called the void
or empty relation on A.
(2) Universal Relation : Let A be a set. Then A × A Í A × A and so it is a relation on A. This relation is called
the universal relation on A.
(3) Identity Relation : Let A be a set. Then the relation I A = {(a, a) : a Î A} on A is called the identity relation
on A.
In other words, a relation IA on A is called the identity relation if every element of A is related to itself only.
Illustration 18. The relation IA = {(1, 1), (2, 2), (3, 3)} is the identity relation on set A = {1, 2, 3}.
But relations
R1 = {(1, 1), (2, 2)} and R2 = {(1, 1), (2, 2), (3, 3), (1, 3)} are not identity relations
on A, because (3, 3) Ï R1 and in R2 element 1 is related to elements 1 and 3.
36
Set, Relation & Function
(4) Reflexive Relation : A relation R on a set A is said to be reflexive if every element of A is related to itself.
Thus, R on a set A is not reflexive if there exists an element A Î A such that (a , a) Ï R.
Illustration 19. Let A = {1, 2, 3} be a set. Then R = {(1, 1), (2, 2), (3, 3), (1, 3), (2, 1)} is a reflexive
relation on A.
But R1 = {(1, 1), (3, 3), (2, 1), (3, 2)} is not a reflexive relation on A, because 2 Î A
but (2, 2) Ï R1.
Note : Every Identity relation is reflexive but every reflexive ralation is not identity.
Illustration 24. Let R be a relation on the set of all lines in a plane defined by (l1, l2) Î R Û line l1
is parallel to line l2. R is an equivalence relation.
Note : It is not neccessary that every relation which is symmetric and transitive is also reflexive.
3.0 FUNCTION
A relation R from a set A to a set B is called a function if each element of A has unique image in B.
ƒ
It is denoted by the symbol. ƒ : A ® B or A ¾¾® B which reads ‘ƒ ’ is a function from A to B ‘or’ ƒ maps
A to B,
If an element a Î A is associated with an element b Î B, then b is called ‘the ƒ image of a’ or ‘image of a under
ƒ ‘or’ the value of the function ƒ at a’. Also a is called the pre-image of b or argument of b under the function
ƒ. We write it as b = ƒ (a) or ƒ : a ® b or ƒ : (a, b)
Thus a function ‘ƒ ’ from a set A to a set B is a subset of A × B in which each 'a' belonging to A appears in one
and only one ordered pair belonging to ƒ.
37
JEE-Mathematics
Representation of Function
(a) Ordered pair – Every function from A ® B satisfies the following conditions :
(i) ƒ Í A x B (ii) " a Î A there exist b Î B and (iii) (a, b) Î ƒ & (a, c) Î ƒ Þ b = c
ìx + 1 -1 £ x < 4
(iii) ƒ(x) = í (non-uniformly defined)
î-x 4£ x <7
ìx2 x³0
(iv) ƒ(x) = í (non-uniformly defined)
î- x - 1 x<0
y1 y1
x1 x1
y2
1. If R is a relation from a finite set A having m elements to a finite set B having n elements, then the number of
relations from A to B is-
(A) 2mn (B) 2mn –1 (C) 2mn (D) mn
2. In the set A = {1, 2, 3, 4, 5}, a relation R is defined by R = {(x, y) | x, y Î A and x < y}. Then R is-
(A) Reflexive (B) Symmetric (C) Transitive (D) None of these
3. For real numbers x and y, we write x R y Û x – y + 2 is an irrational number. Then the relation R is-
(A) Reflexive (B) Symmetric (C) Transitive (D) none of these
4. Let X = {1, 2, 3, 4} and Y = {1, 3, 5, 7, 9}. Which of the following is relations from X to Y-
Node-1\Target-2020-21\1.JEE(M+A)\Module\Enthusiast\English\Maths\EM-1\17.Set, Relation & Function\Th.
(A) R1 = {(x, y) | y = 2 + x, x Î X, y Î Y} (B) R2 = {(1, 1), (2, 1), (3, 3), (4, 3), (5, 5)}
(C) R3 = {(1, 1), (1, 3), (3, 5), (3, 7), (5, 7)} (D) R4 = {(1, 3), (2, 5), (2, 4), (7, 9)}
5. Let A = {2, 3, 4, 5} and let R = {(2, 2), (3, 3), (4, 4), (5, 5), (2, 3), (3, 2), (3, 5), (5, 3)} be a relation in A.
Then R is-
(A) Reflexive and transitive (B) Reflexive and symmetric
(C) Reflexive and antisymmetric (D) none of these
6. If A = {2, 3} and B = {1, 2}, then A × B is equal to-
(A) {(2, 1), (2, 2), (3, 1), (3, 2)} (B) {(1, 2), (1, 3), (2, 2), (2, 3)}
(C) {(2, 1), (3, 2)} (D) {(1, 2), (2, 3)}
7. Let R be a relation over the set N × N and it is defined by (a, b) R (c, d) Þ a + d = b + c. Then R is-
(A) Reflexive only (B) Symmetric only
(C) Transitive only (D) An equivalence relation
8. If A = {1, 2, 3}, B = {1, 4, 6, 9} and R is a relation from A to B defined by 'x is greater than y'. Then range
of R is-
(A) {1, 4, 6, 9} (B) {4, 6, 9} (C) {1} (D) none of these
38
Set, Relation & Function
A f
B
P 1
q 2
r 3
s 4
5
y-axis
y-axis
y
(i)
O x-axis (ii) O x-axis (iii) O x
y
y y
O
x
(iv) (v) (vi)
O x O x
y
y
O
x
(vii) (viii) O x
Node-1\Target-2020-21\1.JEE(M+A)\Module\Enthusiast\English\Maths\EM-1\17.Set, Relation & Function\Th.
y
y
O
x
(ix) (x) x
O
39
JEE-Mathematics
14. Let A = {–2, –1, 0, 1, 2} and B = {0, 1, 2, 3, 4, 5, 6}. Consider a rule f(x) = x 2. Find domain & range
15. Let f : R ® R be given by f(x) = x2 + 3. Find (a) {x : f(x) = 28} (b) the pre-images of 39 and 2 under f.
ì 1, if x Î Q
16. If f : R ® R be defined as followings : f (x) = í .
î -1, if x Ï Q
Find (a) f(1/2), f ( p), f ( 2) (b) Range of f (c) pre-images 1 of and –1.
17. If a, b Î {1, 2, 3, 4}, then which of the following are functions in the given set ?
(a) f1 = {(x, y) : y = x + 1} (b) f2 = {(x, y) : x + y > 4}
(c) f3 = {(x, y) : y < x } (d) f4 = {(x, y) : x + y = 5}
Also, in case of a function give its range.
18. Express the following functions as sets of ordered pairs and determine their ranges :
(a) f : A ® R, f(x) = x2 + 1, where A = {–1, 0, 2, 4}
(b) g : A ® N, g(x) = 2x, where A = { x : x Î N, £ 10 }
19. Is g = {(1, 1), (2, 3), (3, 5), (4, 7)} a function ? If this is described by the formula, g(x) = ax + b , then what
values should be assigned to a and b ?
20. The value of b and c for which the identity f (x + 1) – f(x) = 8x + 3 is satisfied, where f(x) = bx 2 + cx +
d , are –
(a) b = 2 , c = 1 (b) b = 4 , c = – 1 (c) b = –1 , c = 4 (d) b = –1 , c = 1
21. Given the function f(x) = (x + 1)/(x – 1). Find f(1/x), f(2x), 2f(x), f(x 2), [f(x)]2.
1- x
22. Given the function f ( x ) = log .
1+ x
æ x1 + x 2 ö
Show that at x1, x2 Î (–1, 1) the following identity holds true: f ( x1 ) + f ( x 2 ) = f ç ÷
è 1 + xlx2 ø
23. Given the function f(x) = (ax + a–x)/2 (a > 0).
Show that f(x + y) + f(x – y) = 2f (x) f(y).
ì -x
ï 3 - 1, -1 £ x < 0,
ï
24. Given the function f ( x ) = í tan ( x / 2 ) , 0 £ x < p,
ï
( 2
)
ïîx / x - 2 , p £ x £ 6,
æ p ö æ 2p ö
Find f ( -1), f ç ÷ , f ç ÷ , f (4), f (6)
è 2ø è 3 ø
25. The function f(x) is defined over the whole number scale by the following law:
Node-1\Target-2020-21\1.JEE(M+A)\Module\Enthusiast\English\Maths\EM-1\17.Set, Relation & Function\Th.
ì 2x 3 + 1, if x £ 2,
f(x)
ï
f x= í1/ ( x - 2 ) , if 2 < x £ 3, Find: f( 2 ) , f( 8 ), f ( )
log 2 1024 .
ï 2x - 5, if x > 3,
î
40
Set, Relation & Function
NOTE :
(i) It should be noted that range is a subset of co-domain.
(ii) If only the rule of function is given then the domain of the function is the set of those real numbers, where
function is defined. For a continuous function, the interval from minimum to maximum value of a function
gives the range
æ æ 1 ö ö
(iii) f(x) = log2 ç - log1 / 2 çè 1 + 4 ÷ø - 1÷
è x ø
Solution (i) sinx ³ 0 and 16 – x2 ³ 0 Þ 2np £ x £ (2n + 1)p and –4 £ x £ 4
\ Domain is [–4, –p] È [0, p]
(ii) y = log(x–4)(x2 – 11x + 24)
Here 'y' would assume real value if,
x – 4 > 0 and ¹ 1, x2 – 11x + 24 > 0 Þ x > 4 and ¹ 5, (x – 3) (x – 8) > 0
Þ x > 4 and ¹ 5, x < 3 or x > 8 Þ x > 8 Þ Domain = (8, ¥)
æ æ 1 ö ö
(iii) We have f(x) = log2 ç - log1/ 2 ç 1 + ÷ø - 1÷
è è 4
x ø
æ 1 ö
f(x) is defined if - log1/ 2 çè 1 + 4 ÷ø - 1 > 0
x
æ 1 ö æ 1 ö -1
or if log1 / 2 ç 1 + 4 ÷ < -1 or if ç 1 + 4 ÷ > (1 / 2)
è xø è xø
1 1
or if 1+ 4
> 2 or if > 1 or if x1/4 < 1 or if 0 < x < 1
x 4
x
\ D(f) = (0, 1)
Illustration 26. Find the range of following functions :
1 æ sin x - cos x + 3 2 ö
(i) f(x) = (ii) f(x) = log2 ç ÷ø
8 - 3sin x è 2
Node-1\Target-2020-21\1.JEE(M+A)\Module\Enthusiast\English\Maths\EM-1\17.Set, Relation & Function\Th.
1
Solution (i) f(x) =
8 - 3sin x
–1 £ sinx £ 1
é 1 1ù
\ Range of f = ê , ú
ë11 5 û
æ ö
(ii) Let y = log2 sin x - cos x + 3 2
çè ÷ø
2
æ pö
Þ 2y = sin çè x - ÷ø + 3 Þ –1 £ 2y – 3 £ 1
4
41
JEE-Mathematics
Þ 2 £ 2y £ 4 Þ y Þ [1, 2]
l To check whether relation is a function, vertical line test can be applied on its graph.
l Domain of f1, f2 be D1, D2 then domain of f1 + f2, f1 – f2, f1 × f2 will be D1 Ç D2.
l f and g one are two functions defined for same domain, range of f is R and g is a bounded in domain. Then range
of f(x) + g(x) is R.
l If domain consist of discrete number of elements range can be found by direct substituting the values of x.
æ 5x - x 2 ö
Node-1\Target-2020-21\1.JEE(M+A)\Module\Enthusiast\English\Maths\EM-1\17.Set, Relation & Function\Th.
1
(C) y = log x 2 (D) y = log 1 - x + x + 2
10 ( )
5. Find the domains of the following function:
3
(A) y = + log10 (x 3 - x) (B) y = log10 sin(x - 3) + 16 - x 2
4 - x2
(C) y = sin x + 16 - x 2
6. Find the domains of the following function:
x-2 1- x 2 1
(A) y = + (B) y = x - 3x + 2 +
x+2 1+ x 3 + 2x - x 2
42
Set, Relation & Function
7. Find the domains of the following function:
æ 5 - 2x ö
8. The number of integers lying in the domain of the function ƒ (x) = log 0.5 ç ÷ is -
è x ø
(A) 3 (B) 2 (C) 1 (D) 0
1- | x |
9. The domain of the function f (x) = is -
2 -| x |
(A) x Î ( & ¥, & 1) È (1, ¥) (B) x Î ( & ¥, & 2) È (2, ¥)
(C) x Î éë&1, 1ùû È ( & ¥, & 2) È (¥, 2) (D) None of these
æ 3x -1 ö
11. The domain of ƒ(x) = log1/3 ç ÷ -1 is -
è x +2ø
æ 1 5ù æ5 ö æ5 ö æ 5ö
(A) ç , ú (B) ç , ¥ ÷ (C) ( -¥, -2 ) È ç , ¥ ÷ (D) ç -2, ÷
è 3 8û è8 ø è8 ø è 8ø
43
JEE-Mathematics
B a s i c a lg e b ra i c f u n c t i o n
y
(i) y = x2 2 Do ma in : R
y=x
Range : R + È {0} or [0,¥)
o x
y
1 y = 1/x
(ii) y = Do ma in : R – {0} or R 0
x
o x
Range : R – {0}
y
1
(iii) y = y = 1/x
2
Do ma in : R0
x2
x
o Range : R + or (0, ¥)
y y = x3
(iv) y = x3 1 Do ma in : R
o x
1 Range : R
x
function on A and is denoted by I A. It is easy to observe that identity
function is a bijection.
(e) Constant function
y
f : A ® B is said to be constant function if every element of A has y=C
the same f image in B. Thus f : A ® B ; f(x) = c, " x Î A, c Î
x
B is constant function. Note that the range of a constant function is o
a singleton set.
Domain – R Range – {C}
Y
(f) Tr ig onom e t ri c f u nc t ions (–3p/2,1) (p/2,1)
44
Set, Relation & Function
Domain – R
f(x) = cos x
o p /2 X
Domain : R
(–p ,–1) (p ,–1)
Range – [–1, 1], period 2p
Y
(iii) Tangent function
ì (2n + 1)p ü
Domain – R - í x|x = ,n Î I ý
î 2 þ
Y
Range – R , period p
y=1
(–3p/2,1) (p/2,1)
(iv) Cosecant function
o X
f(x) = cosec x (–p/2,–1)
y=–1
y=1
f(x) = sec x (–2p,1) (0,1)
o X
(-p ,-1)
Domain – R – {x|x = (2n + 1) p/2 : n Î I} (p , -1)
y=-1
f(x) = cot x
O
x
Domain – R – {x|x = np, n ÎI} (– 3p ,0) (– p ,0) (
p
,0) ( 3 p ,0)
2 2 2 2
Range – R, period p
x=–2p x=–p x=p x=2p
45
JEE-Mathematics
Doma in of a x is R Range R +
y y
+¥ +¥
(0,1)
f(x)=a,x a>1 (0,1) f(x)=ax, a Î(0,1)
45°
(1,0) x (1,0) x
x x g(x)=logax
y= y=
g(x)=logx
a
y y = log2x
y = log3x
1 y = log5x
y = log10x
0 1 x
y y
y=e
x
y=e–x
x x
O O
Node-1\Target-2020-21\1.JEE(M+A)\Module\Enthusiast\English\Maths\EM-1\17.Set, Relation & Function\Th.
1
N o t e - 2 – f(x) = log x a = D om a i n – R + – {1} Range – R – {0}
log a x
(a > 0) (a ¹ 1) y
y = –x y=x
(h) A b s ol u t e v a l u e f u n c t i o n
The absolute value (or modulus) of a real number
x (written |x|) is a non negative real number that
satisfies the conditions. x
O
ì x if x³0
|x| = í Domain : R
î- x if x<0
Range : [0, ¥)
46
Set, Relation & Function
The properties of absolute value function are
(i) The inequality |x| £ a means that - a £ x £ a ; if a > 0
(ii) The inequality |x| ³ a means that x ³ a or x £ – a if a > 0
(iii) |x ± y| £ |x| + |y| ( Tr ia n g le I ne q ua lit y ) Equality holds when x .y ³ 0
(iv) |x ± y| ³ ||x| – |y|| ( Tr ia n g le I ne q ua lit y ) Equality holds when x .y ³ 0
(v) |xy| = |x|.|y|
x |x|
(vi) = , (y ¹ 0)
y |y|
1
Note – f(x )= , Doma in : R – {0}, Ra nge : R +
|x|
Solution The equality |a – b| = |a|–|b| holds true if and only if a and b have the same sign and
|a|³|b|.
In our case the equality will hold true for the value of x at which x4 – 4 ³ x2 + 2.
Hence x2 – 2 ³ 1; |x| ³ 3.
(A) [–1, 15] (B) [1, 15] (C) [–7, 7] (D) [5, 10]
5. Sum of all the integers in the range of the function f ( x ) = x 2 - 4 x + 5 , where xÎ[0,3] , is
2 e
(A) 2e (B) (C) (D) 1
e 2
47
JEE-Mathematics
9. Sum of natural numbers which are not contained in the range of function f ( x) = esec x , is
1
11. Range of F ( x ) = is
8sin x + 5
æ 1ù æ 1ù é1 ö æ 1ù é 1 1ù
(A) ç -¥, (B) ç -¥, - ú È ê , ¥ ÷ (C) ç -¥, (D) ê , ú
è 3 ûú è 3 13
û ë ø è 13 úû ë13 3 û
1
12. Range of function f ( x) = x is
e -3
æ 1ö æ 1ö æ1 ö æ 1 1ö
(A) ç -¥, - ÷ È (0, ¥) (B) ç -¥, - ÷ È ç , ¥ ÷ (C) (–¥, ¥) (D) ç - , ÷
è 3ø è 3ø è 3 ø è 3 3ø
æ 11 ù é 11 ö é 11 11 ù
(A) ç -¥, log e (B) ê log e ,¥÷ (C) ê - log e , log e ú (D)None of these
è 3 úû ë 3 ø ë 3 3û
4
14. Range of function f ( x ) = is
6sin x + 5cos x + 7
Node-1\Target-2020-21\1.JEE(M+A)\Module\Enthusiast\English\Maths\EM-1\17.Set, Relation & Function\Th.
é 61 - 7 ö æ 61 - 7 ù
(A) ê , ¥ ÷÷ (B) çç -¥, ú
ë 3 ø è 3 û
æ æ 61 + 7 ö ù é 4 ö æ 4 ù
ç
(C) ç -¥, - çç ÷÷ú È ê ,¥ ÷ (D) ç -¥, ú
è è 3 øúû ë 7 + 61 ø è 7 - 61 û
æ pö
15. Number of integers in the range of function f ( x) = 5sin x + 4cos ç x + ÷ + 3 is
è 6ø
48
Set, Relation & Function
p2
16. The range of the function y = 3sin - x 2 is
16
é 3 ù é 3 3 ù
(A) ê 0, (B) ê - ,
ë 2 úû ë 2 2 úû
é 3 ù
(C) ê - ,0
2 úû
(D) None of these
ë
x 2 - 3x + 2
17. The value of the function f(x) = 2 lies in the interval
x + x -6
ì1 ü
(A) (– ¥ , ¥ )– í ,1ý (B) (– ¥ , ¥ ) (C) (– ¥ , ¥ )–{1} (D) None of these
î5 þ
18. Sum of least integer and greatest integer in the range of the function f ( x) = sin 2 x - 3sin x + 5 is
x2 + x + 1 a
20. If product of least and greatest value of the function f ( x ) = 2 is where a and b are coprime then
x -x+2 b
a+b=
(A) 5 (B) 10 (C) 15 (D) 20
x
21. Range of the function f ( x ) = is
1 + x2
Node-1\Target-2020-21\1.JEE(M+A)\Module\Enthusiast\English\Maths\EM-1\17.Set, Relation & Function\Th.
é 1 1ù é 1ù é 1 ù
(A) [–1, 1] (B) ê - , (C) ê -1, (D) ê - ,1ú
ë 2 2 úû ë 2 úû ë 2 û
22*. Let set ‘S’ be the range of the function f ( x ) = 9 tan 2 x + 16cot 2 x then
æ sin x - cos x + 3 2 ö
23. If [p, q] is range of function f ( x ) = log 2 çç ÷÷ then 2p + 5q =
è 2 ø
49
JEE-Mathematics
ìx ì1 for x > 0
ï ,x ¹ 0 ï
y =í x = í0 for x = 0 o x
ïî0, x = 0 ï y = Sgn x
î-1 for x < 0
Do ma in – R
Range – {–1, 0, 1} y = -1
(j) G r e a t es t i n t e g e r or s t e p u p f un c t i o n y
The function y = f(x) = [x] is called graph of y = [x] 3
the greatest integer function where [x]
denotes the greatest integer less than
or equal to x. Note that for : 2
x [x] 1
[–2,–1) –2
x
–3 –2 –1 0 1 2 3
[–1,0) –1
–1
[0,1) 0
–2
[1,2) 1
Do ma in – R –3
Range – I
Pr op er ti e s of gr e a te s t in t e ge r f un c ti on
(i) [x] £ x < [x] + 1 and x – 1 < [x] £ x, 0 £ x – [x] < 1
(ii) [x + m] = [x] + m if m is an integer.
ì0, x Î I
(iii) [x] + [–x] = í
î -1, x Î
/I
1 1
Note – f(x) = Domain – R – [0, 1) Range – {x|x = , n ÎI0 }
[x] n
(k) Fr ac t ional pa r t f unc tion
y
It is defined as – g(x) = {x} = x – [x] e.g.
the fractional part of the number 2.1 graph of y = {x}
is 2.1–2 = 0.1 and the fractional part
Node-1\Target-2020-21\1.JEE(M+A)\Module\Enthusiast\English\Maths\EM-1\17.Set, Relation & Function\Th.
x {x} x
-2 0 2 3
-1 1
[–2,–1) x+2
[–1,0) x+1
[0,1) x
[1,2) x–1
Illustration 29. If y = 2[x] + 3 & y = 3[x – 2] + 5 then find [x + y] where [ . ] denotes greatest integer function.
Solution y = 3[x – 2] + 5 = 3[x] – 1
so 3 [x] – 1 = 2 [x] + 3
[x] = 4 Þ 4 £ x < 5
then y = 11
so x + y will lie in the interval [15, 16)
so [x + y] = 15
é1 ù é1 1 ù é 1 2946 ù
Illustration 30. Find the value of ê ú + ê + ú + ...... ê + where [ . ] denotes greatest integer
ë 2 û ë 2 1000 û ë 2 1000 ûú
function ?
Þ x £ –17 or x ³ 17
so x Î (–¥, –17] È [17, ¥)
x - [x]
Illustration 32. Find the range of f(x) = , where [.] denotes greatest integer function.
1 + x - [x]
x - [x] {x}
Solution y= =
1 + x - [x] 1 + {x}
1 1 1 1-y y
\ = +1 Þ = Þ {x} =
y {x} {x} y 1- y
y
0 £ {x} < 1 Þ 0 £ <1
1- y
Range = [0, 1/2)
51
JEE-Mathematics
Illustration 33. Solve the equation |2x – 1| = 3[x] + 2{x} where [.] denotes greatest integer and { .} denotes
fractional part function.
Solution We are given that, |2x – 1| = 3[x] + 2{x}
1
Let, 2x – 1 £ 0 i.e. x £ . The given equation yields.
2
1 – 2x = 3[x] + 2{x}
1 - 5[x]
Þ 1 – 2[x] – 2{x} = 3[x] + 2{x} Þ 1 – 5[x] = 4{x} Þ {x} =
4
1 - 5[x] 3 1
Þ 0£ < 1 Þ 0 £ 1 – 5[x] < 4 Þ – < [x] £
4 5 5
3 1
Now, [x] = 0 as zero is the only integer lying between – and
5 5
1 1 1 1
Þ {x} = Þx= which is less than , Hence is one solution.
4 4 2 4
1
Now, let 2x – 1 > 0 i.e x >
2
Þ 2x – 1 = 3[x] + 2{x} Þ 2[x] + 2{x} – 1 = 3[x] + 2{x}
1
Þ [x] = –1 Þ –1 £ x < 0 which is not a solution as x >
2
1
Þ x= is the only solution.
4
2 æ px ö æ px ö
(iv) f(x) = (v) f(x) = cos ç ÷ (vi) f(x) = – sin ç ÷
x -3 è 2 ø è 3 ø
2. Sketch the graphs of the following functions :
ì x, 0 £ x £1 ì3 - x, x £1
(i) f (x) = í (ii) f (x) = í
î2 - x 1< x £ 2 î2 - x x >1
Node-1\Target-2020-21\1.JEE(M+A)\Module\Enthusiast\English\Maths\EM-1\17.Set, Relation & Function\Th.
ì1
ï , x<0
f (x) = íx ì1 - x 2 , 0 < x £1
(iii) (iv) f (x) = í
ïî x, x ³0 î2 - x , 1 <x £2
2
l nx
3. Draw the garph of the function f(x) = .
ln x
4. Find the number of solutions of the equation y = | sinx | and x 2 + y2 = 1.
5. Draw the graph of the function f(x) = | sin x | + sin x on the interval [0, 3 p].
ì1, x > 0
ï
6. Draw the graph of the function y = x 2 Sgn (x) where Sgn (x) = í 0, x = 0 .
ï-1, x < 0
î
52
Set, Relation & Function
7. Sketch the graphs of the following functions :
| (x - 1)(x - 2) |
(i) f(x) = 1 + | sin x | (ii) f(x) =
(x - 1) (x - 2)
æ fö f(x)
(c) çè g ÷ø (x) = g(x) domain A Ç B – {x|g(x) = 0}
æ x - 1ö
Illustration 34. The functions f(x) = log (x – 1) – log (x – 2) and g(x) = log ç are identical when x lies in the
è x - 2 ÷ø
interval
(A) [1, 2] (B) [2, ¥) (C) (2, ¥) (D) (–¥, ¥)
Solution Since f(x) = log (x – 1) –log (x – 2).
Domain of f(x) is x > 2 or x Î (2, ¥) .....(i)
æ x - 1ö x -1
g(x) = log ç
è x - 2 ÷ø
is defined if >0 Þ x Î (–¥, 1) È (2, ¥) .....(ii)
x-2
From (i) and (ii), x Î (2, ¥). Ans. (C)
53
JEE-Mathematics
8.0 HOMOGENEOUS FUNCTIONS
A function is said to be homogeneous with respect to any set of variables when each of its terms is of the
same degree with respect to those variables.
For examples 5x2 + 3y2 – xy is homogenous in x & y. Symbolically if, f(tx, ty) = tn f(x, y) then f(x, y)
is homogeneous function of degree n.
x2 + e x + 5 x 2 log x
(A) y = (B) y = x2 (C) xy – sin(x + y) = 0 (D) y =
1 - cos -1 x sin x
Solution It is clear that in (C) y is not clearly expressed in x. Ans. (C)
(iv) Every function which has '–x' in it's domain whenever 'x' is in it's domain, can be expressed as the
sum of an even & an odd function .
f(x) + f(-x) f(x) - f(-x)
+
e.g. f(x) = 2 2
EVEN ODD
(v) The only function which is defined on the entire number line & even and odd at the same time is f(x) = 0
ƒ(x) g(x) ƒ (x) + g(x) ƒ(x) – g(x) ƒ (x) . g(x) ƒ (x)/g(x) (goƒ) (x) (ƒog)(x)
odd odd odd odd even even odd odd
even even even even even even even even
odd even neither odd nor even neither odd nor even odd odd even even
even odd neither odd nor even neither odd nor even odd odd even even
54
Set, Relation & Function
Illustration 37. Which of the following functions is (are) even, odd or neither :
æ1 - xö
(i) f(x) = x2sinx (ii) f(x) = 1 + x + x2 - 1 - x + x2 (iii) f(x) = log çè ÷
1 + xø
e x + e- x
(iv) f(x) = sinx – cosx (v) f(x) =
2
Solution (i) f(–x) = (–x)2 sin(–x) = –x2 sinx = –f(x).Hence f(x) is odd.
(ii) f(–x) = 1 + ( - x) + ( - x)2 - 1 - ( - x) + ( - x)2
e - x + e - ( - x) e - x + e x
(v) f(–x) = = = f(x). Hence f(x) is even
2 2
Illustration 38. Identify the given functions as odd, even or neither :
x x
(i) f(x) = + +1 (ii) f(x + y) = f(x) + f(y) for all x, y Î R
e -1 2
x
x x
Solution (i) f(x) = + +1
ex - 1 2
Clearly domain of f(x) is R ~ {0}. We have,
-x x -e x .x x (e x - 1 + 1)x x
f(–x) = - +1 =
-x x
- +1 = - +1
e -1 2 1-e 2 (e x - 1) 2
x x x x
= x+ - +1 = x + + 1 = f(x)
ex - 1 2 e -1 2
Hence f(x) is an even function.
(ii) f(x + y) = f(x) + f(y) for all x, y Î R
Replacing x, y by zero, we get f(0) = 2f(0) Þ f(0) = 0
Replacing y by –x, we get f(x) + f(–x) = f(0) = 0 Þ f(x) = –f(–x)
Hence f(x) is an odd function.
Node-1\Target-2020-21\1.JEE(M+A)\Module\Enthusiast\English\Maths\EM-1\17.Set, Relation & Function\Th.
x
(a) f (x) = and j (x) º 1; (b) f(x) = log x2 and j (x) = 2 log x;
x
( x)
2
(c) f(x) = x and j (x) = ;
55
JEE-Mathematics
2. Are the following functions identical?
x 1 x2
(a) f(x) = 2 and j ( x ) = ; (b) f ( x ) = and j(x) = x; (c) f(x) = x and j ( x ) = x 2 ;
x x x
3. In what interval are the following functions identical?
x2 y y2 x x ( x - y) x2y y y
(A) l n + ln (B) (C) (D) x sin - ycos
y2 x x2 y y ( x + y) x 3 + y3 x x
6. Determine whether even or odd:
( )
æ ax +1ö
(i) f(x) = log x + 1 + x
2
(ii) f (x) = x çç x ÷÷
è a -1 ø
æ 1- x ö
(v) f(x) = log ç ÷
è1+ x ø
(vi) f(x + y) + f(x – y) = 2f(x). f(y); where f(0) ¹ 0 and x, y Î R.
7. If f(x) satisfies the relation, f (x + y) = f(x) + f(y) for all x, y Î R and
m
ì x | x |, 0 £ x < 1
8. A function defined for all real numbers is defined for x ³ 0 as follows : f (x) = í
î 2x, x ³1
ì x|x| , x £ -1
ï
9. Find out whether the given function is even, odd or neither even nor odd where, f(x) = í[1 + x] + [1 - x], -1 < x < 1
ï -x | x | , x ³1
î
. where | | and [ ] represents modulus and greatest integral function.
10. Find whether the given function is even or odd function, where
x(sin x + tan x)
f(x) = é x + p ù - 1 where, [ ] denotes greatest integer function.
êë p úû 2
56
Set, Relation & Function
11. Function f ( x) = x + x 3 is
(A) Even (B) Odd (C) Neither even nor odd (D) None of these
æ 1- x ö
14. The function f ( x) = ln ç ÷ is
è 1+ x ø
(A) An even function (B) An odd function
(C) Neither even nor odd function (D) None of these
æ 1 - x2 ö
15. The function f ( x ) = ln ç 2 ÷ is
è 1+ x ø
(A) An even function (B) An odd function
(C) Neither even nor odd fucntion (D) None of these
16. Which of the following is not an odd function ?
(A) g(x) – g(–x) (B) sin (g4(x) – g4(–x))
æ x 4 + x2 + 1 ö
(C) ln ç 2 ÷ (D)x3g6(x).g6(–x)+ tan3(sin5x)
è x + x +1 ø
Match the entries of column- I with one or more entries of the elements of column - II.
17*. Column-I
Column-II
(A) f ( x) = 1 - x + x 4 - 1 + x + x 4 (P) Is even
(D) f ( x ) = 0
(S) Has graph symmetric about y-axis
18*. Column-I
Column-II
3
(A) f ( x ) = x + 8 x + 9 (P) Has graph symmetric about y-axis
e x - e- x
(B) f ( x) = (Q) Has graph symmetric about origin
e x + e- x
x x
(C) f ( x ) = x
+ +1 (R) Is neither even nor odd function
e -1 2
g 2 ( x ) - g 2 (- x)
(D) f ( x ) = "xÎ R (S) Is odd function
g 2 ( x) + g 2 (- x)
57
JEE-Mathematics
y=f(x)+b,b > 0
y=f(x)
A1
x0 y=f(x)+b,b < 0
O A
A2
It is obvious that domain of f(x) and f(x) + b are the same. Let us take any point x 0 in the domain of
f(x). y x = x0 = f(x 0 ) .
For b > 0 Þ f(x0) + b > f(x0) it means that the corresponding point on f(x) + b would be lying at a
distance 'b' units above the point on f(x).
For b < 0 Þ f(x0) + b < f(x0) it means that the corresponding point on f(x) + b would be lying at a
distance 'b' units below the point on f(x).
Accordingly the graph of f(x) + b can be obtained by translating the graph of f(x) either in the positive
y-axis direction (if b > 0) or in the negative y-axis direction (if b < 0), through a distance |b| units.
(ii) Drawing the graph of y = –f(x) from the known graph of y = f(x)
To draw y = –f(x), take the image of the curve y = f(x) in the x-axis as plane mirror.
y y=f(x) y
Þ
x x
0
0
y=– f(x)
Node-1\Target-2020-21\1.JEE(M+A)\Module\Enthusiast\English\Maths\EM-1\17.Set, Relation & Function\Th.
(iii) Drawing the graph of y = f(–x) from the known graph of y = f(x)
To draw y = f(–x), take the image of the curve y = f(x) in the y-axis as plane mirror.
y y = f(x) y
Þ y = f(–x )
x x
0 0
(iv) Drawing the graph of y = |f(x)| from the known graph of y = f(x)
|f(x)| = f(x) if f(x) ³ 0 and |f(x)| = –f(x) if f(x) < 0. It means that the graph of f(x) and |f(x)| would
coincide if f(x) ³ 0 and for the portions where f(x) < 0 graph of |f(x)| would be image of y = f(x) in x-
axis.
58
Set, Relation & Function
y y
y=|f(x)|
y=f(x)
x Þ x
O O
(v) Drawing the graph of y = f(|x|) from the known graph of y = f(x)
ìf(x), x ³ 0
It is clear that, f(|x|) = í . Thus f(|x|) would be a even function, graph of f(|x|) and f(x)
îf(- x), x < 0
would be identical in the first and the fourth quadrants (ax x ³ 0) and as such the graph of f(|x|) would
be symmetric about the y-axis (as (|x|) is even).
y y
y=f(x) y=f(|x|)
Þ
O x O x
(vi) Drawing the graph of |y| = f(x) from the known graph of y = f(x)
Clearly |y| ³ 0. If f(x) < 0, graph of |y| = f(x) would not exist. And if f(x) ³ 0, |y| = f(x) would give
y = ±f(x). Hence graph of |y| = f(x) would exist only in the regions where f(x) is non-negative and will
be reflected about the x-axis only in those regions.
y y
|y|=f(x)
Þ
O x O x
y=f(x)
(vii) Drawing the graph of y = f(x + a), a Î R from the known graph of y = f(x)
y=f(x)
y=f(x+a),a>0
y=f(x–a)
a<0
x0–|a| x0 x0+|a|
Node-1\Target-2020-21\1.JEE(M+A)\Module\Enthusiast\English\Maths\EM-1\17.Set, Relation & Function\Th.
(i) If a > 0, shift the graph of f(x) through 'a' units towards left of f(x).
(ii) If a < 0, shift the graph of f(x) through 'a' units towards right of f(x).
(viii) Drawing the graph of y = af(x) from the known graph of y = f(x)
y=af(x), a >1
y=f(x)
y=af(x),
0<a<1
It is clear that the corresponding points (points with same x co-ordinates) would have their ordinates in the ratio
of 1 : a.
59
JEE-Mathematics
(ix) Drawing the graph of y = f(ax) from the known graph of y = f(x).
y y=f(x)
y=f(ax), a >1
y=f(ax),0<a<1
O
x
x0
Let us take any point x0 Î domain of f(x). Let ax = x0 or x = .
a
1
Clearly if 0 < a < 1, then x > x0 and ƒ (x) will stretch by units along the y-axis and if a > 1, x < x 0, then
a
f(x) will compress by 'a' units along the y-axis.
0,1
ì1 - x ; x < -1
ï (–1,0) (1,0)
f(x) = í2 ; -1 £ x £ 1 (0,0)
x
ï1 + x ; x > 1
î
1 y=|x–1|
x– y=–|x–1|
y= (0,1)
(1,0)
(1,0) x
Solution O Þ Þ O 1
O 1 (0,–1)
(–1,0)
2 y=|2–|x–1|
Node-1\Target-2020-21\1.JEE(M+A)\Module\Enthusiast\English\Maths\EM-1\17.Set, Relation & Function\Th.
2 y=2–|x–1|
Þ
O 1 x (–1,0) O 1 (3,0)
4
Illustration 41. Draw the graph of y = 2 -
|x - 1|
y
y y
1 1
y= 1 y=
x y= |x–1|
x–1
x=1
x O x
Solution O Þ x Þ O
(0,–1)
60
Set, Relation & Function
y y
y y=2– 4
1 x=1 x=1 y=2 |x–1|
y=– –4
|x–1| y=
|x–1|
O x O x
Þ Þ (3,0)
x
(–1,0)
(0,–1) (0,–4)
(0,–2)
y=ex
1 y=e|x|
Solution O Þ
y y |x|
y=|e –2|
|x|
y=e –2 (0,1)
O x
Þ x
O
(0,–1)
Illustration 43. Draw the graph of f(x) = cosx cos(x + 2) – cos2(x + 1).
Solution f(x) = cosx cos(x + 2) – cos2(x + 1)
1 1 0
= éëcos(2x + 2) + cos 2 ùû – éëcos(2x + 2) + 1ùû
2 2
½ cos2 – ½
1 1
= cos 2 - < 0 .
2 2
1
Node-1\Target-2020-21\1.JEE(M+A)\Module\Enthusiast\English\Maths\EM-1\17.Set, Relation & Function\Th.
2 3
0 1
–1
f (x )
(v) y =
f (x)
61
JEE-Mathematics
3. Draw the graph of f(x) = 3| x | – x2 – 2
4. Draw the graph of f(x) = | x2 – 2 | x | – 3 |
-|x| 1
5. Draw the graph of f (x) = e -
2
6. Draw the graph of | f (x) | = (x – 1) (x – 2)
1
7. Sketch the graph of f(x) =
2- | x |
| x - 1|
8. Sketch the graph of f(x) =
| x | -1
1
9. Draw the graph of the function f(x) = -2 .
x
éx for 0 £ x £1
ê1 for x > 1 and f 2 ( x ) = f1 ( - x ) for all x ;
10. Let f1(x) = ê
êë 0 otherwise
(a) O n e - O n e f u n c t io n ( I n j e c t iv e m a p p i n g )
A function f : A ® B is said to be a one-one function or injective mapping if different
elements of A have different f images in B. Thus for x1, x2 Î A & f(x1), f(x2) Î B, f(x1) =
f(x2) Û x1 = x2 or x 1 ¹ x 2 Û f(x 1 ) ¹ f(x 2 ).
Note – (i) Any continuous function which is entirely increasing or decreasing in whole domain
is one-one.
(ii) If a function is one-one, any line parallel to x-axis cuts the graph of the function at atmost one point
or
62
Set, Relation & Function
(b ) M a ny- one f unc t ion
A function f – A ® B is said to be a many one function if two or more elements of A have the
same f image in B.
Thus f – A ® B is many one if $ x 1 , x 2 Î A, f(x 1 ) = f(x 2 ) but x 1 ¹ x 2
Diagramatically a many one mapping can be shown
A B
x1
or x2
Note – If a continuous function has local maximum or local minimum, then f(x) is many-one because
atleast one line parallel to x-axis will intersect the graph of function atleast twice.
(c ) Onto funct ion (Surje c t ive mapping)
If the function f : A ® B is such that each element in B (co-domain) is the ‘f’ image
of atleast one element in A, then we say that f is a function of A 'onto' B. Thus f : A ®
B is surjective if " b Î B, $ some a Î A such that f(a) = b
or
or
63
JEE-Mathematics
Note – (i) If ‘f ’ is both injective & surjective, then it is called a Biject ive mapping. The bijective
functions are also named as invertible, non singular or biuniform functions.
(ii) If a set A contains n distinct elements then the number of different functions defined
from A ® A is n n & out of it n! are one one and rest are many one.
(iii) f–R ® R is a polynomial
(a) Of even degree, then it will neither be injective nor surjective.
(b) Of odd degree, then it will always be surjective, no general comment can be given
on its injectivity.
Illustration 44. Let A = {x : –1 £ x £ 1} = B be a mapping f : A ® B. For each of the following functions from
A to B, find whether it is surjective or bijective.
(a) f(x) = |x| (b) f(x) = x|x| (c) f(x) = x3
px
(d) f(x) = [x] (e) f(x) = sin
2
Solution (a) f(x) = |x|
Y
Graphically ;
Which shows many one, as the straight line
is parallel to x-axis and cuts at two points.
Here range for f(x) Î [0, 1]
Which is clearly subset of co-domain i.e., –1 O +1 X
[0, 1] Í [–1,1]
Thus, into.
Hence, function is many-one-into
\ Neither injective nor surjective
ì-x2 , -1 < x < 0üï
(b) f(x) = x|x|= ïí ý,
2
îïx , 0 £ x < 1 þï
Y
Graphically, 1
The graph shows f(x) is one-one, as the straight
line parallel to x-axis cuts only at one point. –1 O 1 X
Here, range
f(x) Î [–1, 1] –1
Thus, range = co-domain
Hence, onto.
Therefore, f(x) is one-one onto or (Bijective).
(c) f(x) = x3,
Node-1\Target-2020-21\1.JEE(M+A)\Module\Enthusiast\English\Maths\EM-1\17.Set, Relation & Function\Th.
Graphically; Y
1
Graph shows f(x) is one-one onto
–1 O 1 X
(i.e. Bijective)
[as explained in above example] –1
(d) f(x) = [x],
Graphically;
Which shows f(x) is many-one, as the straight line Y
2
1
parallel to x-axis meets at more than one point. –2 –1
O 1 3 X
Here, range
–1
f(x) Î {–1, 0, 1}
which shows into as range Í co-domain –2
Hence, many-one-into
64
Set, Relation & Function
px
(e) f(x) = sin Y
2
Graphically; 1
Which shows f(x) is one-one and onto as range
–1
O 1 X
= co-domain.
Therefore, f(x) is bijective. –1
x 2 + 3x + a
Solution f(x) =
x2 + x + 1
+ 1
1*. Let f : R ® R : f ( x ) = x + then
x
Node-1\Target-2020-21\1.JEE(M+A)\Module\Enthusiast\English\Maths\EM-1\17.Set, Relation & Function\Th.
+ x
4*. Let f : R ® R : f ( x ) = then
x2
(A) f is one-one function (B) f is many-one function
(C) f is on-to function (D) f is In-to function
65
JEE-Mathematics
Match the entries of column- I with one or more entries of the elements of column - II.
5*. Column-I Column-II
(A) f : R ® R : f ( x) = x 3 + x 2 + 5 x + 7 (P) One-one function
e x - e- x
(A) f : R ®[ -1,1]: f ( x) = (P) One-one mapping
e x + e- x
(B) f : R ® R : f ( x) = x + | x | (Q) Many-one mapping
x f(x)
Diagramatically f g g (f(x))
Thus the image of every x Î A under the function gof is the g-image of f-image of x.
Note that gof is defined only if " x Î A, f(x) is an element of the domain of ‘g’ so that we can take its
g-image. Hence in gof(x) the range of ‘f’ must be a subset of the domain of ‘g’.
Node-1\Target-2020-21\1.JEE(M+A)\Module\Enthusiast\English\Maths\EM-1\17.Set, Relation & Function\Th.
(b) The composite of functions is associative i.e. if f, g, h are three functions such that fo(goh) & (fog)oh
are defined, then fo(goh) = (fog)oh.
(c) The composite of two bijections is a bijection i.e. if f & g are two bijections such that gof is defined,
then gof is also a bijection.
l f & g are two one-one function defined for all real values then both fog & gof are one-one.
l f : R ® R, G : R ® R and range of f is A then range of fog is A.
66
Set, Relation & Function
æ 5ö æ 5ö
Illustration 47. If f be the greatest integer function and g be the modulus function, then (gof) ç - ÷ – (fog) ç - ÷ =
è 3ø è 3ø
(A) 1 (B) –1 (C) 2 (D) 4
æ -5 ö æ -5 ö ì æ -5 ö ü ì æ -5 ö ü æ 5ö
Solution Given (gof) ç ÷ – (fog) ç ÷ = g íf çè ÷ø ý - f íg çè ÷ø ý = g(–2) – f ç ÷ = 2 – 1 = 1
è 3ø è 3ø î 3 þ î 3 þ è 3ø
Ans.(A)
Illustration 48. Find the domain and range of h(x) = g(f(x)), where
ì [x], -2 £ x £ -1 ì [x], -p £ x £ 0
f(x) = í and g(x) = í , [.] denotes the greatest integer function.
î x + 1, -1 < x £ 2 îsin x, 0 £ x £ p
ì [f(x)], -p £ f(x) < 0 f(x)=p
Solution h(x) = g(f(x)) = í
î sin(f(x)), 0 £ f(x) £ p +3
From graph of f(x), we get
–2 –1 +1
ì [[x]], -2 £ x £ -1 –1 2
h(x) = í
îsin( x + 1), -1 < x £ 2 –2
Þ Domain of h(x) is [–2, 2] f(x)=–p
ì x + 1, x £1 ì x 2 , -1 £ x < 2
Illustration 49. Let f(x) = í and g(x) = í , find (fog)
î2x + 1, 1 < x £ 2 î x + 2, 2 £ x £ 3
ì g(x) + 1, g(x) £ 1
Solution f(g(x)) = í
y
î2g(x) + 1, 1 < g(x) £ 2
Here, g(x) becomes the variable that means 5
2
we should draw the graph. 4 x+
ì1 + x, 0£x£2
1. Let f (x) = í find (fof) (x).
î3 - x, 2<x£3
æ pö æ pö æ5ö
2. If f(x) = sin²x + sin² ç x + ÷ + cos x cosç x + ÷ and g ç ÷ = 1 , then find (gof) (x).
è 3ø è 3ø è4ø
4. {
If ƒ (x) = x + 1 2 if
5 - x if
x £ 1 and g(x) = x
x >1
{ if
2 - x if
x £1
x > 1 then ƒ (g(x)) =
x
5. If f(x) = , then (fofof ) (x) =
1 + x2
3x x 3x
(A) 2 (B) 2 (C) (D) none
1+ x 1 + 3x 1 + x2
ïì x + 1, x < 0
2
ìï 3 - x, x < 0
6*. Let f ( x ) = í and g ( x) = í 2 then
ïî 5 - x, x ³ 0 ïî x - 5, x ³ 0
ì x 4 - 10 x 2 + 26 x<0 ì x 4 - 10 x 2 + 26 0 £ x < 5
ïï 2 ï
(C) g ( f ( x)) = í x - 10 0£ x£5 (D) f ( g ( x)) = í x+2 x<0
ï x-2 x>5 ï 2
ïî î 10 - x x³ 5
ì- x - 8 x £ -1
ïì x - 9 x < 0 ï
2
7*. Let f ( x) = í and g ( x) = í x - 8 -1 < x < 2 then
ïî 8 - x x ³ 0 ï 2
î5 - x x³2
ì x + 16 x Î(-¥, - 8]
ï 2
ì(5 - x 2 ) 2 + 9, x Î( -¥, - 1] ï ( x + 8) - 9 x Î( -8, - 1]
ï ï 2
(A) f ( g ( x)) = í x + 5, x Î( -1,2) (B) f ( g ( x)) = í ( x - 8) - 9 x Î( -1,2)
ï ï 2
ï x +3 x Î[2, 5]
2
î ( x + 8) + 9 x Î[2, ¥ )
ï(5 - x 2 )2 - 9 x Î( 5, ¥)
î
Node-1\Target-2020-21\1.JEE(M+A)\Module\Enthusiast\English\Maths\EM-1\17.Set, Relation & Function\Th.
68
Set, Relation & Function
(e) Since f(a) = b if and only if f –1(b) = a, the point (a, b) is on the graph of ‘f’ if and only if the point (b,
a) is on the graph of f–1. But we get the point (b, a) from (a, b) by reflecting about the line y = x.
y y y
(b, a)
y = f(x) y=x
–1
f
(a, b)
0 x
0 x 0 x (–1,0)
(0,–1)
y=x y=x f –1
y = f (x)
The graph of f -1 is obtained by reflecting the graph of f about the line y =x.
é p pù
Þ the inverse mapping would be f –1 – [-1, 1] ® ê - , ú .
ë 2 2û
y
(1,p/2)
p/2 y=x
y=sinx
1 (p/2,1)
–p/2 –1 x
O 1 p/2
(0,–1)
(–p /2,1)
(-1-p/2)
Node-1\Target-2020-21\1.JEE(M+A)\Module\Enthusiast\English\Maths\EM-1\17.Set, Relation & Function\Th.
Illustration 50. Let f – R ® R be defined by f(x) = (ex–e–x)/2. Is f(x) invertible ? If so, find its inverse.
Solution Let us check for invertibility of f(x) :
(a) One-One
Let x1, x2 Î R and x1< x2
Þ e1 <e2x x (Because base e > 1) .......... (i)
Also x1 < x2 Þ –x2 < –x1
Þ e- x2 < e - x1 (Because base e > 1) .......... (ii)
(i) + (ii) Þ e x1 + e - x2 < e x2 + e- x1
1 x1
Þ ( e - e- x1 ) < 1 ( e x2 - e - x2 ) Þ f(x1) < f(x2) i.e. f is one-one.
2 2
(b) Onto
As x tends to larger and larger values so does f(x) and
when x ® ¥, f(x) ® ¥.
Similarly as x ® -¥, f(x) ® -¥ i.e. - ¥ < f(x) < ¥ so long as x Î (-¥, ¥)
69
JEE-Mathematics
Hence the range of f is same as the set R. Therefore f(x) is onto.
Since f(x) is both one-one and onto, f(x) is invertible.
(c) To find f–1
Let f–1 be the inverse function of f, then by rule of identity fof–1(x) = x
-1 -1
ef ( x)
- e- f ( x)
-1 -1
= x Þ e2f (x)
- 2xe f (x)
-1 = 0
2
-1 2x ± 4x2 + 4 -1
Þ ef ( x)
= Þ ef ( x) = x ± 1 + x2
2
-1
Since e f (x)
> 0, hence negative sign is ruled out and
-1
Hence e f (x)
= x + 1 + x2
ì x; < 1
ï 2
Illustration 51. Find the inverse of the function f(x) = í x ;1 £ x £ 4
ï
î 8 x; x > 4
ì x; < 1
ï 2
Solution Given f(x) = í x ;1 £ x £ 4
ï
î 8 x; x > 4
Let f(x) = y Þ x = f–1(y)
ì ì
ï y, y <1 ï y, y <1
ïï ïï
\ x = í y, 1£ y £4 Þ f -1 (y) = í y, 1 £ y £ 16
ï 2 ï 2
ïy , y2 ïy , y > 16
>4 ïî 64
ïî 64 64
ì x; < 1
ï
ïï
Hence f–1(x) = í x;1 £ x £ 16 Ans.
ï 2
ï x
; x > 16
ïî 64 Node-1\Target-2020-21\1.JEE(M+A)\Module\Enthusiast\English\Maths\EM-1\17.Set, Relation & Function\Th.
1- x
1. If f(x) = , the domain of f -1 ( x) is
1+ x
(A) R (B) R– {–1} (C) (– ¥ , –1) (D) (–1, ¥ )
e x - e- x
2. The inverse of the function f ( x) = + 1 is
e x + e- x
æ x ö æ x ö
(A) log10 ç ÷ (B) ln ç ÷
è 2- xø è 2- xø
1/ 2
æ x ö 1 æ x ö
(C) ln ç ÷ (D) log ç ÷
è 2- x ø 2 è 2- x ø
70
Set, Relation & Function
10 x - 10- x
3. The inverse of the function f ( x ) = is
10 x + 10- x
1 æ1+ x ö
(A) log10 ( 2 - x ) (B) log10 ç ÷
2 è 1- x ø
1 1 æ 2x ö
(C) log10 (2 x - 1) (D) log ç ÷
2 4 è 2- x ø
x ( x-1)
æ1ö 1 1
(A) ç ÷ (B) {1 + 1 + 4 log 2 x } (C) {1 - 1 + 4 log 2 x } (D) not defined
è2ø 2 2
æ 3ö
6. Show that the functions f(x) = x2 – x + 1, x ³ 1/2 and g(x) = 1/2 + ç x - ÷ are mutually inverse and
è 4ø
solve the equation
æ 3ö
x2 – x + 1 = 1/2 + çx - ÷ .
è 4ø
7. For what values of the parameter m are the following functions invertible ?
(i) f(x) = x3 – mx2 + 3x – 11
(ii) f(x) = (m + 2) x3 – 3mx2 + 9mx – 1.
A function f(x) is called periodic if there exists a least positive number T(T >0) called the period of the function
such that f(x + T) = f(x), for all values of x within the domain of f(x).
e.g. The function sinx & cosx both are periodic over 2p & tan x is periodic over p.
Note – For periodic function
(i) f(T) = f(0) = f(–T), where 'T' is the period.
(ii) Inverse of a periodic function does not exist.
(iii) Every constant function is periodic, but its period is not defined.
(iv) If f(x) has a period T & g(x) also has a period T then it does not mean that f(x) + g(x)
must have a period T. e.g. f(x) = |sin x| + |cos x|.
1
(v) If f(x) has period p, then and f(x) also has a period p.
f(x)
(vi) If f(x) has period T then f(ax + b) has a period T/|a| (a ¹ 0).
71
JEE-Mathematics
Illustration 52. Find the periods (if periodic) of the following functions, where [.] denotes the greatest integer
function
(i) f(x) = eln(sinx) + tan3x – cosec(3x – 5) (ii) f(x) = x – [x – b], b Î R
sin x + cos x p
(iii) f(x) = (iv) f(x) = tan [x]
sin x + cos x 2
p p p p
tan [x + T]= tan [x] Þ [x + T] = np + [x]
2 2 2 2
\ T=2
\ Period = 2
(v) Let f(x) is periodic then f(x + T) = f(x)
Þ cos(sin(x + T))+ cos(cos(x + T)) = cos(sinx) + cos(cosx)
If x = 0 then cos(sinT) + cos(cosT)
è 2ø è 2ø
p
On comparing T =
2
(1 + sin x)(1 + sec x) (1 + sin x)(1 + cos x) sin x
(vi) f(x) = =
(1 + cos x)(1 + cos ec x) cos x(1 + sin x)(1 + cos x)
Þ f(x) = tanx
Hence f(x) has period p.
(vii) f(x) = e x-[x]+ cos px + cos 2 px +........+ cos np
Period of x – [x] = 1
Period of |cospx| = 1
1
Period of |cos2px| =
2
......................................
1
Period of |cosnpx| =
n
So period of f(x) will be L.C.M. of all period = 1
72
Set, Relation & Function
Illustration 53. Find the periods (if periodic) of the following functions, where [.] denotes the greatest integer function
px px
(i) f(x) = ex–[x] + sinx (ii) f(x) = sin + cos
2 3
px px
(iii) f(x) = sin + cos
3 2 3
Solution (i) Period of ex–[x] = 1
period of sinx = 2p
Q L.C.M. of rational and an irrational number does not exist.
\ not periodic.
px 2p
(ii) Period of sin = =2 2
2 p/ 2
px 2p
Period of cos = =2 3
3 p/ 3
Q L.C.M. of two different kinds of irrational number does not exist.
\ not periodic.
px2p
(iii) Period of sin = =2 3
3 p/ 3
px 2p
Period of cos = =4 3
2 3 p/2 3
Q L.C.M. of two similar irrational number exist.
\ Periodic with period = 4 3 Ans.
px px px
1. The period of f (x) = sin + 2 cos - tan , is -
2 3 4
(A) 6 (B) 3 (C) 4 (D) 12
2. Which of the following function has a period of 2p ?
æ pö æ pö px px
(A) f(x) = sin çè 2px + ÷ø + 2 sin çè 3px + ÷ø + 3 sin 5px (B) f(x) = sin + sin
3 4 3 4
(C) f (x) = sin x + cos 2x (D) none
3. If f : R ® R is a function satisfying the property f(x+1) + f(x+3) = K " x Î R then the period of f (x) can be
(A) 4 (B) K (C) 1 (D) p
Node-1\Target-2020-21\1.JEE(M+A)\Module\Enthusiast\English\Maths\EM-1\17.Set, Relation & Function\Th.
4. Let f(x) = sin [a] x (where [ ] denotes the greatest integer function). If f is periodic with fundamental period
p, then a belongs to -
(A) [2, 3) (B) {4, 5} (C) [4, 5] (D) [4, 5)
(A) Periodic with fundamental period 1 (B) Periodic with fundamental period 2
(C) Periodic with fundamental period 3 (D) Non periodic
p
(A) p (B) 2p (C) (D) 3p
2
73
JEE-Mathematics
p p
(A) (B) (C) 2p (D) p
2 4
8. f ( x) = tan 2 x is
p
(A) Periodic with fundamental period (B) Periodic with fundamental period p
2
p p p
(A) (B) p (C) (D)
4 8 16
p
(A) Non periodic function (B) Periodic with fundamental period
2
æxö
13. Find period of f(x) = tan3x + sin ç ÷.
è3ø
14. Find the period of,
x x x x x
f(x) = sinx + tan + sin 2 + tan 3 + ..... + sin n -1 + tan n .
2 2 2 2 2
Node-1\Target-2020-21\1.JEE(M+A)\Module\Enthusiast\English\Maths\EM-1\17.Set, Relation & Function\Th.
15*. Which of the following function (s) is/are periodic with period p .
(A) f (x) = |sin x| (B) f (x) = [x + p ] (C) f (x) = cos (sin x) (D) f (x) = cos2x
(where [.] denotes the greatest integer function)
16. Find the period of the following functions:
(i) f(x) = sin4x + cos4x (ii) f(x) = cos(sinx) + cos(cosx)
(iii) f(x) = | sinx | + | cosx | (iv) f(x) = sin2x + cos3x
1
17. Find the period of function f(x) = .
|| sin 2x | - | cos 2x ||
æ 3 ö
18. Is the function f(x) = 1 + ç 2 ÷ is periodic ?
è 2 - sin x ø
If yes, mention its period.
74
Set, Relation & Function
19. Find the period of following functions:
(i) f(x) = [sin3x] + | cos6x |(where [.] is gretest integer function (ii) f(x) = {
1 | sin x | | cos x |
2 cos x
+
sin x }
px px
(iii) f(x) = 3sin + 4cos (iv) f(x) = cos3x + sin 3px
3 4
px px
(v) f(x) = sin - cos (vi) f(x) = eln(sin x) + tan 3 x - cosec(3x - 5)
n! (n + 1) !
20. Find the period of the real valued function satisfying,
f(x) + f(x + 4) = f(x + 2) + f(x + 6).
21. Check whether the function defined by :
f (x + l) = 1 + 2f (x) - f 2 (x), " x Î R is periodic if yes, then find the period where f(x) Î [0,2].
Match the entries of column- I with one or more entries of the elements of column - II.
22. Column-I Column-II
(A) f ( x ) = sin(sin x ) is (P) Periodic with fundamental period 2 p
p
(C) f ( x ) = 3sin x is (R) Periodic with fundamental period
2
cos(3x )
(D) f ( x ) = is (S) Non periodic
sin(7 x)
23. Column-I Column-II
p
(C) f ( x ) = sin 2 x + cos 4 x + 8 is (R) Periodic with fundamental period
2
(A) f ( x ) = ln({x}) is (where {.} fractional part function) (P) Is periodic with fundamental period 2p
(B) f ( x ) = x - [ x ] + sin 2 px is (where [.] = G.I.F) (Q) Is periodic with fundamental period p
17.0 GENERAL
If x, y are independent variables, then
(a) f(xy) = f(x) + f(y) Þ f(x) = kln x
(b) f(xy) = f(x) . f(y) Þ f(x) = xn, n Î R or f(x) = 0
(c) f(x + y) = f(x) . f(y) Þ f(x) = akx or f(x) = 0
(d) f(x + y) = f(x) + f(y) Þ f(x) = kx, where k is a constant.
75
JEE-Mathematics
9x æ 1 ö æ 2 ö æ 3 ö
1. Let f (x) = x then find the value of the sum f ç ÷+f ç ÷ +f ç ÷ + .............
9 +3 è 2006 ø è 2006 ø è 2006 ø
æ 2005 ö
+f ç ÷
è 2006 ø
f(x) - 1
2. Given that f(11) = 11 and f(x + 3) = for all x ÎR find f(2000).
f(x) + 1
3. Assume that ¦(1) = 0 and that for all integers m and n, ¦(m + n) = ¦(m) + ¦(n) + 3(4mn – 1) then
¦(19) =
5. Let f be a linear function for which f (6) – f (2) = 12. The value of f (12) – f (2) is equal to -
6. A certain function ¦(x) satisfies ¦(x) + 2¦(6–x) = x for all real numbers x. The value of ¦(1), is -
(A) 3 (B) 2
7. If f(x) is a polynomial function satisfying the condition f(x). f(1/x) = f(x) + f(1/x) and f(2) = 9 then -
(A) 2 f(4) = 3f(6) (B) 14 f(1) = f(3) (C) 9 f(3) = f(5) (D) f(10)= f(11)
(A) f : [0, p] ® [– 1, 1], f (x) = sin x (B) f : [0, p] ® [– 1, 1], f (x) = cos x
Node-1\Target-2020-21\1.JEE(M+A)\Module\Enthusiast\English\Maths\EM-1\17.Set, Relation & Function\Th.
n
9. If f : R ® R satisfies f(x + y) = f(x) + f(y), for all x, y Î R and f(1) = 7, then å f(r) is -
r= 1
7n 7(n + 1) 7n(n + 1)
(A) (B) (C) 7n(n + 1) (D)
2 2 2
76
Set, Relation & Function
11. Number of polynomial functions satisfying ƒ (x) + ƒ (3x) = ƒ (2x) " x Î R is/are -
12. Let a function ƒ (x) satisfying functional rule ƒ (x + y) = ƒ (x) ƒ (y) + 3ƒ (x – y) + 1 " x,y Î R, then -
A) ƒ (1) + ƒ (2) = –2 (B) ƒ (1) + ƒ (2) = 0 (C) ƒ (1) + ƒ (2) = 2 (D) ƒ (1) + ƒ (2) = 4
æ 1ö
13. Let ¦(x) is even and g(x) is an odd function which satisfies x 2¦(x) – 2¦ çè ÷ø = g(x),
x
14. If ƒ (x) is an even function defined for all x & satisfy ƒ (1) = 2, ƒ (x + 3) = ƒ (x) + x for x ³ 0, then ƒ (–4) is
equal to -
æ x 4 + x2 + 1 ö
ln
(A) ç 2 2 ÷
è (x + x + 1) ø
æ1ö æ1ö
(C) ƒ (x), where ƒ(x) + ƒ ç ÷ = ƒ(x)ƒ ç ÷ " x Î R – {0} & ƒ(2) = 33
èxø èxø
(D) ƒog(x), where ƒ(x) & g(x) both are odd function & fog(x) is defined.
Node-1\Target-2020-21\1.JEE(M+A)\Module\Enthusiast\English\Maths\EM-1\17.Set, Relation & Function\Th.
77
JEE-Mathematics
l
Case-I when x = AP £ OA, i.e., x £ O
2 F l
P
1 x
ar(DAEF) = x.2x = x2 (Q PE = PF = AP = x)
2 A
E
B
l
Case-II when x = AP > OA, i.e., x > but x £ 2l
2 D
F
C
2–lx
ar(ABEFDA) = ar(ABCD) – ar(DCFE)
P
= l -
2 1
2
( ) (
2l - x .2 2l - x ) [Q CP = 2l - x ]
O E
= ( )
l 2 - 2l2 + x 2 - 2 2lx = 2 2lx - x 2 - l 2 A B
\ the required function s(x) is as follows :
ì 2 l
ï x , 0£x£ ì 1 1
ï 2 ï 2 at x =
s(x) = í ; area of s(x) = í 2
ï2 2 lx – x2 – l2 , l < x £ 2l ï
ïî 2 î8( 2 – 1) at x = 2
Ans.
Illustration 2. If the function f(x) satisfies the functional rule, f(x + y) = f(x) + f(y) " x,y Î R & f(1) = 5, then
m
find å f(n)
n =1
and also prove that f(x) is odd function.
Solution Here, f(x + y) = f(x) + f(y); put x = t – 1, y = 1
f(t) = f(t – 1) + f(1) ....(1)
\ f(t) = f(t – 1) + 5
Þ f(t) = {f(t – 2) + 5} + 5
Þ f(t) = f(t – 2) + 2(5)
Þ f(t) = f(t – 3) + 3(5)
.............................
.............................
Þ f(t) = f{t – (t –1)} + (t – 1)5
Þ f(t) = f(1)+ (t – 1)5
Þ f(t) = 5 + (t – 1)5
Þ f(t) = 5t
Node-1\Target-2020-21\1.JEE(M+A)\Module\Enthusiast\English\Maths\EM-1\17.Set, Relation & Function\Th.
m m
5m(m + 1)
\ å
n =1
f(n) = å (5n) = 5[1 + 2 + 3 + ..... + m] =
n =1 2
m
5m(m + 1)
Hence, å f(n) = .....(i)
n =1 2
Now putting x=0, y=0 in the given function, we have
f(0 + 0) = f(0) + f(0)
\ f(0) = 0
Also putting (–x) for (y) in the given function.
f(x – x) = f(x) + f(– x)
Þ f(0) = f(x) + f(–x)
Þ 0 = f(x) + f(–x)
Þ f(–x) = –f(x) ....(ii)
m
5m(m + 1)
Thus, å f(n) =
n =1 2
and f(x) is odd function.
78
Set, Relation & Function
x x
Illustration 3. Range of f(x) = 4 + 2 + 1 is
x x
Solution f(x) = 4 + 2 + 1
x
Let 2 = t > 0, " x Î R
2
\ f(x) = g(t) = t + t + 1 t>0
2
æ 1ö 3
g(t) = ç t + ÷ +
è 2ø 4
2 2
æ 1ö 1 æ 1ö 1 æ 1ö 3
çt + ÷ > Þ çt + ÷ > Þ çt + ÷ + >1
è 2ø 2 è 2ø 4 è 2ø 4
Range is (1, ¥)
x–1
Illustration 4. Let f : N ® N, where f(x) = x + ( – 1) , then the inverse of f is
x–1
Solution f(x) = x + (–1)
é3 1 ù é3 2 ù é3 3 ù é 3 24 ù
Solution êë 4 + 100 úû + êë 4 + 100 úû + êë 4 + 100 úû + ....... êë 4 + 100 úû
é 3 25 ù é 3 26 ù é 3 27 ù é 3 99 ù
+ê + ú +ê + ú +ê + ú + ....... ê +
ë 4 100 û ë 4 100 û ë 4 100 û ë 4 100 úû
= 0 + 75 = 75
1
Illustration 7. Fundamental period of the function f(x) = + cos (cos6x) is
sin 4x - cos 4x
p
Solution As |sin 4x| – |cos 4x| has period
4
79
JEE-Mathematics
æ pö æp ö æp ö
we get g ç x + 8 ÷ = sin ç 2 + 4x ÷ - cos ç 2 + 4x ÷
è ø è ø è ø
p
\ Fundamental period of g(x) is .
8
Now h(x) = cos (cos 6x)
æ pö
then h ç x + ÷ = cos(cos(p + 6x))
è 6ø
p
= cos (–cos 6x) = cos (cos 6x) \ Period is
6
p p p
Taking L.C.M. of , we get
6 8 2
Illustration 9. If x, y Î [0, 10], then the number of solution (x, y) of the inequality 3sec
2
x -1
9y 2 - 6y + 2 £ 1
are
Solution If x, y Î [0, 10], then the.......
We have, 3 sec
2
x -1
9y 2 - 6y + 2 £ 1
2
2 æ 1ö 1
Þ 3sec x
çy - ÷ + £ 1
è 3 ø 9
2
æ 1ö 1 1
but 3 sec 2
³ 3 and çy - ÷ + ³
è 3 ø 9 3
1
So, using boundness we must have sec2 x = 1 and y - =0
Node-1\Target-2020-21\1.JEE(M+A)\Module\Enthusiast\English\Maths\EM-1\17.Set, Relation & Function\Th.
1
Þ x = 0, p 2p, 3p and y =
3
Þ There are 4 solution.
Illustration 10. Let f (x) and g (x) be bijective functions where f: {7, 8, 9, 10} ® {1, 2, 3, 4} and
g:{3, 4, 5, 6} ® {13, 14, 15, 16 } respectively then find the number of elements in domain and
range of gof(x) ?
Solution domain of gof is {x Î {7, 8, 9, 10} : f(x) Î {3,4}}
\ there are 2 elements in the domain of gof
Since gof is one-one, therefore, there are 2 elements in the range of gof.
80
Set, Relation & Function
SINGLE CORRECT
1. If f : R® R, which of the following rule is NOT a real function-
2 2 2
(A) y = 4 – x (B) y = 3x (C) y = x – |x| (D) y = 3x +5
log 2 ( x + 3)
2. The domain of definition of f (x) = is
x 2 + 3x + 2
(A) R - {-1, -2} (B) ( -2, ¥ ) (C) R – {–1, –2, –3} (D) ( -3, ¥ ) - {-1, -2}
3. The domain of f(x) = loge |logex| is-
(A) (0, ¥) (B) (1, ¥) (C) (0, 1) È (1, ¥) (D) (–¥, 1)
1
4. The domain of the function f(x) = + x + 2 , is -
log10 (1 - x)
(A) [– 2, 0) È (0, 1) (B) (–2, 0) È (0, 1] (C) (– 2, 0) È (0, 1] (D) (– 2, 0) È [0, 1]
6. The domain of definition of ƒ(x) = {log10 (log10 x) – log10 (4 – log10 x) –log10 3}0.5
( 3
(A) 10 ,10
4
) (B) éë10 ,10 ùû
3 4
(C) [103 ,104 ) (D) (103 ,104 ]
1
7. The domain of the function f(x) = 2
is-
[x] - [x] - 6
(A) (–¥, –2) È [4, ¥) (B) (–¥, –2] È [4, ¥) (C) (–¥, –2) È (4, ¥) (D) none of these
x –x
8. The range of the function f (x) = e - e , is -
(A) [0, ¥) (B) (– ¥, 0) (C) (– ¥, ¥) (D) none
1
9. The range of the function f(x) = , is -
4 + 3cos x
(A) [1/ 7, 1] (B) ]1/ 7, 1] (C) (1/ 7, 1] (D) none
Node-1\Target-2020-21\1.JEE(M+A)\Module\Enthusiast\English\Maths\EM-1\17.Set, Relation & Function\Ex.
x2 + x + 2
10. The range of the function f (x) = x Î ( -¥, ¥ ) is
x2 + x +1
æ 11 ö æ 7ù é 7ù
(A) [1, ¥) (B) ç1, ÷ (C) ç1, ú (D) ê1, ú
è 7ø è û3 ë û5
16- x
11. Range of f (x) = C2 x -1 + 20-3 x C4 x -5 is
(A) [ 728, 1474] (B) {728, 1474} (C) {0 , 728} (D) {728, 1617}
x
ex - e
12. ( )
The range of the function f x = x x is
e +e
(A) ( -¥ , ¥ ) (B) [ 0, 1) (C) ( -1, 0] (D) ( -1, 1)
81
JEE-Mathematics
æ 1+ x ö æ 2x ö
13. If f(x) = ln ç ÷ , then f ç 1 + x 2 ÷ equals-
è 1- x ø è ø
2 3
(A) [f(x)] (B) {f(x)} (C) 2f(x) (D) 3f(x)
4x
14. If f(x) = , then f(x) + f(1 – x) is equal to-
4x + 2
(A) 0 (B) –1 (C) 1 (D) 4
1ì æ xö ü
15. If f(x) = cos (log x), then f ( x). f ( y ) - í f ç ÷ + f ( xy ) ý is equal to
2î è yø þ
3 6
17. Function f(x) = loge(x + 1 + x ) is-
(A) even (B) odd (C) neither even nor odd (D) none of these
18. Which of the following is odd function ?
2 x x 2
(A) sin x (B) (a + 1)/(a – 1) (C) x – |x| (D) None of these
sin4 x + cos4 x
19. The function f(x)= is-
x + tan x
(A) odd (B) even (C) neither even nor odd (D) odd and periodic
3
20. If f : R ® R, f(x) = x + x, then f is-
(A) one-one onto (B) one-one into (C) many-one onto (D) many-one into
21. Let f : {x, y, z} ® {a, b, c} be a one-one function and only one of the conditions
(i) f (x) ¹ b,. (ii) f (y) = b (iii) f (z) ¹ a is true then the function f is given by the set
e x
- e- x
23. Let f : R ® R be a function defined by f ( x ) = then
e x + e- x
(A) ‘f’ is one– one and onto (B) ‘f’ is one –one but not onto
(C) ‘f’ is not one–one but onto (D) ‘f’ is neither one – one nor onto
82
Set, Relation & Function
2 2
24. If f : R ® R, f(x) = (x + 1) and g : R ® R, g(x) = x + 1 then (fog)(–3) is equal to-
(A) 121 (B) 144 (C) 112 (D) 11
sin101 x
f (x) =
25. Let é x ù 1 , where [x] denotes the integral part of x is
êpú + 2
ë û
px px
26. Fundamental period of f ( x ) = sin + cos is (n Î N)
( n - 1) ! n!
(A) n ! (B) 2 (n !) (C) 2 ( n - 1) ! (D) None of these
p px p
28. The fundamental period of sin [x] + cos + cot [x], where [x] denotes the integral part of x is
4 2 3
(A) 8 (B) 4 (C) 3 (D) 24
p
(A) (B) 2p (C) p (D) none of these
2
30. The number of solution of 2 cos x = | sin x |, 0 £ x £ 4p is
(A) 0 (B) 2 (C) 4 (D) infinite
31. The numer of solutions of the equation sin px = | ln | x | | is
(A) infinite (B) 6 (B) 8 (D) 0
ì -1 - 2 £ x £ 0
32. Let f(x) be defined on [–2, 2] and is given by f ( x ) = í and g(x) = f(|x|) + |f(x)|. Then g(x)
î x -1 0 < x £ 2
is equal to
ï ï ì- x - 2 £ x < 0
0 0 £ x <1 0 0 £ x <1
(A) í (B) í (C) í (D) None of these
ï x -1 1 £ x £ 2
î
ï 2( x - 1) 1 £ x £ 2
î î x -1 0 £ x £ 2
33. The function ƒ(x) = max {(1 – x), (1 + x), 2}, x Î ( -¥, ¥ ) is equivalent to
ì1 - x x £ -1 ì1 + x x £ -1
ï ï
(A) f(x) = í 2 -1 < x < 1 (B) f(x) = í 2 -1 < x < 1
ï1 + x x ³1 ï1 - x x ³1
î î
ì1 - x x £ -1
ï
(C) f (x) = í1 -1 < x < 1 (D) None of these
ï1 + x x ³1
î
83
JEE-Mathematics
æ1ö 2
34. If 2f(x) – 3f ç ÷ = x , x is not equal to zero, then f(2) is equal to-
èxø
7 5
(A) - (B) (C) –1 (D) none of these
4 2
2 2 2 2
35. If 2 f(x ) + 3 f(1/x ) = x – 1 " x Î R0 then f(x ) is -
1 - x4 1 - x2 5x2 3 - 2x 4 - x2
(A) (B) (C) (D)
5x2 5x 1 - x4 5x2
36. The graph of the function y = f(x) is as shown in the figure. Then which of the following could represent the
graph of the function y = f(x) ?
(–2, 1) (2, 1)
(–1, 0) (1, 0)
(0, –1)
y
y y
y 1
1 1 1
4x æ 1 ö æ 2 ö æ 1996 ö is equal to
37. If f(x) = x
4 +2
, then f ç ÷+ fç ÷ + .... + fç ÷
è 1997 ø è 1997 ø è 1997 ø
(A) 1997 (B) 998 (C) 0 (D) None of these
( fog ) ( x ) is equal
-1
39. Let f : R ® R, g : R ® R be two functions given by f (x) = 2x – 3, g(x) = x 3 + 5. Then
to
1/ 3 1/ 3 1/ 3 1/ 3
æ x-7ö æ x+7 ö æ 7ö æ x-2ö
(A) ç ÷ (B) ç ÷ (C) ç x - ÷ (D) ç ÷
è 2 ø è 2 ø è 2ø è 7 ø
2 f (n) +1
40. If f ( n + 1) = , n = 1, 2,.... and f (1) = 2 , then f (101) equals
2
84
Set, Relation & Function
42. The graph of f(x) is given then the number of positive solution of f(x) -1 = 1 are-
y
4
x
–4 –2 O 2 4
–4
(A) 5 (B) 2 (C) 3 (D) 1
43. If g(x) is a polynomial satisfying g(x) g(y) = g(x) + g(y) + g(xy) - 2 for all real x and y and g(2) = 5 then g(3)
is equal to -
(A) 10 (B) 24 (C) 21 (D) none of these
1
44. The graph of function f(x) is as shown, adjacently. Then the graph of is -
f(|x|)
y = f(x)
a b
y
y
ab x
o o a b x
(A) (B)
y y
(C) –b –a o a b 2 (D) –a a x
O
Node-1\Target-2020-21\1.JEE(M+A)\Module\Enthusiast\English\Maths\EM-1\17.Set, Relation & Function\Ex.
( )
(A) sgn e
-x
(B) sin x + |sin x|
é 1ù é 1ù
(C) min {sin x,| x |} (D) ê x + ú + ê x - ú + 2 [ - x ]
ë 2 û ë 2 û
85
JEE-Mathematics
p
46. The equation sin x – + 1 = 0 has one root in interval.
2
æ pö æp ö æ 3p ö
(A) ç 0, ÷ (B) ç ,p÷ (C) ç p, ÷ (D) None of these
è 2ø è2 ø è 2 ø
(A) two distinct real roots if a > 12 (B) four distinct real roots if 8 < a < 12
(C) can have at most 4 real roots (D) have sum of real roots to be zero, if a > 8.
n
(C) fn–1(x)
Node-1\Target-2020-21\1.JEE(M+A)\Module\Enthusiast\English\Maths\EM-1\17.Set, Relation & Function\Ex.
86
Set, Relation & Function
1 y = f(x)
0 2
(A)
1 y = f(x)
–1 0 1 –2 –1
(B) (C)
1. Figure B represents the graph of the function
(A) – f(x) (B) –f(x – 1) + 1 (C) – f(x + 1) – 1 (D) – f(x + 1) + 1
2. [1, 3] and [0, 1] are the domain and range (respectively) of the function
(A) –f(x) (B) f(x – 1) (C) –f(x + 1) + 1 (D) – f(x + 1)
3. Figure C represents the graph of function
(A) 2f(x) (B) f(x – 2) (C) f(x + 2) (D) f(x – 2) + 1
Comprehension-2
y
1
3p –p p p 3p
The graph of y = sin x is –2p -
2
-
2 0 2 2
2p x
–1
–1 –1
y
Node-1\Target-2020-21\1.JEE(M+A)\Module\Enthusiast\English\Maths\EM-1\17.Set, Relation & Function\Ex.
3p –p p p 3p
(C) –2p -
2
-
2 0 2
p
2
2p x (D) None of these
–1
y
1
3p p
-
2 2
p 3p
(C) –2p –p -
2 0 p
2
2p x (D) None of these
–1
87
JEE-Mathematics
| sin x |
6. The graph of y = is
sin x
y
y 1
1
(A) (B) 3p p 0 p 5p x
0 x –2p -
2
–p -
2
p 3p 2p 3p
2 2 2
–1
–1
y 1
(C) 1 (D) 3p p 0 p 5p x
–2p - –p - 2 p 3p 2p 3p
0 x 2 2 2 2
–1
Comprehension-3
Comprehension–4
ì x + 1, if x £1 ì x, if x £1
If ¦(x) = í & g(x) = í
2
î5 - x , if x >1 î2 - x, if x >1
88
Set, Relation & Function
Match the column
Following question contains statements given in two columns, which have to be matched. The statements in
Column-I are labelled as A, B, C and D while the statements in Column-II are labelled as p, q, r and s. Any given
statement in Column-I can have correct matching with ONE OR MORE statement(s) in Column-II.
15. Column - I Column - I I
2
(A) If f(x) = x – 4x + 3, then graph of f(|x|) is (p) o x
1
1
1
(B) If g(x) = , then it's graph is (q)
ln x –2 –1 O 1 2 3 x
2
(C) If f(x) = x – 4x + 3, then graph of |f(x)| is (r)
–3 –1 1 3 x
1
(D) If k(x) = , then its graph is (s)
{ x} 1 3 x
¦(x) =
3x + 4
(C) ¦ : R ® R (r) many one
sinx –sinx
¦(x) = e + e
(D) ¦ : R ® R (s) into
2
¦(x) = log(x + 2x + 3)
17. Let f(x) = (x + 1)(x + 2)(x + 3)(x + 4) + 5 where x Î [–6, 6]. If the range of the function is [a, b] where
a, b Î N then find the value of (a + b).
é3ù é4ù
18. The set of real values of ‘x’ satisfying the equality ê ú + ê ú = 5 (where [ ] denotes the greatest integer function)
ëxû ëxû
æ bù b
belongs to the interval ç a, ú where a, b, c Î N and is in its lowest form. Find the value of a + b + c + abc.
è cû c
89
JEE-Mathematics
19. Suppose p(x) is a polynomial with integer coefficients. The remainder when p(x) is divided by x – 1 is 1 and
the remainder when p(x) is divided by x – 4 is 10. If r(x) is the remainder when p(x) is divided by
(x –1)(x – 4), find the value of r(2006).
20. Find the domains of definitions of the following functions :
(Read the symbols [*] and {*} as greatest integers and fractional part functions respectively)
3 2
(a) f(x) = log7 log5 log3log2(2x + 5x – 14x)
1 2 x
(b) f(x) = + lnx(x – 1) (c) f(x) = log 1
2
4x2 - 1 2 x -1
(d) f(x) = logx sinx
f
(e) If f(x) = x2 - 5x + 4 & g(x) = x + 3, then find the domain of g (x)
21. Solve the following problems from (a) to (e) on functional equation.
(a) The function f(x) defined on the real numbers has the property that f(f(x)) . (1 + f(x)) = –f(x) for all x in the
domain of f. If the number 3 is in the domain and range of f, compute the value of f(3).
(b) Suppose f is a real functional satisfying f(x + f(x)) = 4f(x) and f(1) = 4. Find the value of f(21).
(c) Let’f’ be a function defined from R+ ® R+. If [f(xy)]2 = x(f(y))2 for all positive numbers x and y and f(2) = 6,
find the value of f(50).
(d) Let f(x) be a function with two properties
(i) For any two real number x and y, f(x + y) = x + f(y) and
(ii) f(0) = 2.
Find the value of f(100)
(e) Let f be a function such that f(3) = 1 and f(3x) = x + f(3x – 3) for all x. Then find the value of f(300).
é1 - x if x£0 é - x if x <1
22. f ( x) = ê 2 and g ( x) = ê find (fog)(x) and (gof) (x)
ë x if x >0 ë1 - x if x ³1
(
(a) f(x) = ln x + x + 1
2
) (b) f(x) =
2
x
x-1 (c) y =
10 x - 10- x
10 x + 10- x
f ( x) - 5
24. Let f : R ® R – {3} be a function with the property that there exist T > 0 such that f(x + T) = for
f ( x) - 3
every x Î R. Prove that f(x) is periodic.
25. Let f(x) = x135 + x125 – x115 + x5 + 1. If f(x) is divided by x3 – x then the remainder is some function of x say
Node-1\Target-2020-21\1.JEE(M+A)\Module\Enthusiast\English\Maths\EM-1\17.Set, Relation & Function\Ex.
26. Let {x} & [x] denote the fractional and integral part of a real number x respectively. Solve 4{x} = x + [x]
9x 1 ö æ 2 ö æ 3 ö æ 2007 ö
27. Let f(x) = then find the value of the sum f æç + fç + fç + .......f ç
9x + 3 è 2008 ÷ø è 2008 ÷ø è 2008 ÷ø è 2008 ÷ø
28. Let f be a one-one function with domain {x, y, z} and range {1, 2, 3}. It is given that exactly one of the
following statements is true and the remaining two are false.
f(x) = 1; f(y) ¹ 1; f(z) ¹ 2. Determine f –1 (1)
æ 1- x ö
29. A function f : R ® R is such that f ç ÷ = x for all x ¹ -1 . Prove the following.
è1+ x ø
(a) f(f(x)) = x (b) f(1/x) = –f(x), x ¹ 0 (c) f(–x – 2) = –f(x) – 2.
90
Set, Relation & Function
æx 2ö æ 5 1ö
2. If ç + 1, y - ÷ = ç , ÷ , find the values of x and y.
è3 3 ø è 3 3ø
3. If the set A has 3 elements and the set B = {3, 4, 5}, then find the number of elements in (A ´ B).
4. If A ´ B = {(a, x), (a, y), (b, x), (b, y)}, find A and B.
1
5. Find the domain of the function f (x) defined by : f (x) = 4- x +
x2 - 1
1
6. Find the domain of the function f (x) defined by : f (x) =
x+ | x |
(i) f -1{26}
f -1{-5}
91
JEE-Mathematics
18. Find the domain and range of each of the following functions given by :
1
(i) f (x) = (ii) f (x) = 1 - | x - 3 |
x - [x]
ì x , 0 £ 0x £ 1
ï 3- x
ï
19. Draw the graphs of the function : f (x) = í , 1< x < 3
ï 2
ïî - x + 3 , 3 £ x £ 5
ì
ï x2 , x £ 0
ï
20. Draw the graphs of the function : f (x) = í x, 0 < x < 1
ï 1
ï , x ³1
î x
x 2 + 2x + 1
21. Find the domain of the function: f (x) = .
x 2 - 8x + 12
22. Find the domain and the range of the real function f defined by f (x) = (x - 1).
ìïæ x2 ö üï
23. Let f = íç x, 2÷
, : x Î R ý be a function from R into R. Determine the range of f.
ïîè 1 + x ø þï
f
24. Let f , g : R ® R be defined respectively by f (x) = x + 1, g(x) = 2x - 3. Find f + g, f - g and .
g
25. Let f = {(1,1), (2, 3), (0, - 1), (-1, - 3)} be a function from Z to Z defined by f (x) = ax + b, for some
integers a, b. Determine a, b.
26. Let A = {1, 2, 3, 4}, B = {1, 5, 9,11,15,16} and
f = {(1, 5), (2, 9), (3,1), (4, 5), (2,11)}. Are the following true?
Node-1\Target-2020-21\1.JEE(M+A)\Module\Enthusiast\English\Maths\EM-1\17.Set, Relation & Function\Ex.
92
Set, Relation & Function
ANSWER KEY
BEGINNER'S BOX-1
1 (A) 2 (C) 3 (A) 4 (A) 5 (B) 6 (A)
7 (D) 8 (C) 9 (A) 10 (B)
11. Domain of f = {p, q, r, s}. Co-domain of f = {1, 2, 3, 4, 5}. Range of f = {1, 2, 3, 4}
12. (iii), (iv), (vi), (vii), (ix)
13. (iv, v)
14. Domain = {–2, –1, 0, 1, 2}, Range = {0, 1, 4}
15. (a) {–5, 5} (b) Pre-image of 39 are ± 6. Does not have any pre-image of 2 under f.
(b) Range of g = {(x, g(x)): x Î A }; Range of g = {2, 4, 6, 8, 10, 12, 14, 16, 18, 20}
19. g is a function. a = 2, b = –1.
20. (b)
2
æ 1 ö 1+ x 2x + 1 æ1+ x ö 2 x2 + 1 æ x +1ö
21. fç ÷= , f (2x) = , 2f (x) = 2 ç ÷ , f (x ) = (f (x)) 2 = ç ÷
x
è ø 1 - x 2x - 1 è x - 1 ø 2
x -1 è x -1 ø
æpö æ 2p ö
24. f(–1) = 2, f ç ÷ = 1, f ç ÷ = 3 , f(4) = 2/7, f(6) = 3/17
è2ø è 3 ø
25. f ( 2) = 4 2 +1 , f ( 8) = 2 1
2 -2
, f ( )
log 2 1024 = 2 10 - 5
BEGINNER'S BOX-2
1. (A) x > -3 (B) x £5/2 (C) R – {1, - 1}
(D) R (E) R – {0, 1, - 1} (F) R – {1, 2}
93
JEE-Mathematics
BEGINNER'S BOX-4
4. (2)
BEGINNER'S BOX-5
1. (a) No (b) No (c) No (d) Yes (e) No
2. (a) Yes (b) No (c) No
3. (a) (0, ¥) (b) [1, ¥)
4. (B,C)
5. (A,B,C)
6. (i) odd (ii) even (iii) neither even nor odd
(iv) even (v) odd (vi) even
5
7. m(m + 1)
2
8. (i) for even f(x) = – x|x| –1 < x £ 0 (ii) for odd f(x) = x|x| –1 £ x £ 0
– 2x x£–1 2x x£–1
9. Even 10. Odd 11. (B) 12. (B) 13. (C) 14. (B)
15. (A) 16. (C)
17. [(A)-Q ; (B)-Q ; (C)-R ; (D)-P,Q,S] 18. [(A)-R ; (B)-Q, S ; (C)-P ; (D)-Q, S]
BEGINNER'S BOX-6
2
1. (a) y = x - 1 (b) y = 4 - x (c) y = x2 – 2 x
y y y
(–2,0) o
(0, 1) x
(2, 0)
x
(–2,0) o
x
(–1, 0) (0, 0) (1, 0) (2, 0)
y y
y
Node-1\Target-2020-21\1.JEE(M+A)\Module\Enthusiast\English\Maths\EM-1\17.Set, Relation & Function\Ex.
(0, 1)
o x
o x o x
(–1, 0) (1, 0)
y y y
1 1
1 2 6 x
o x o o x
–1 4 2 3
–1
94
Set, Relation & Function
y
y
1
1
2 3 x
0
0 2 3 x 0 2 3 x
–1
y y
x
–2 –1 1 2
3. f(x) = 3| x | – x – 2 2
4. f(x) = | x2 – 2 | x | – 3 | o 3
x
–3
½ y
1
5. f (x) = e- | x | - x 6.| f (x) | = (x – 1) (x – 2) x
2 – ln 2 o o 1 2
ln 2
1
–1 o x
1 ½ | x - 1| 1
7. f(x) = 2- | x | –2 2 x 8. f(x) = | x | -1 –1
o
y
Node-1\Target-2020-21\1.JEE(M+A)\Module\Enthusiast\English\Maths\EM-1\17.Set, Relation & Function\Ex.
2
1
9. f(x) = -2 . 10. (B) 12. (i) 5, (ii) 2
x o x
–1/2 ½
BEGINNER'S BOX-7
1. (B D) 2. (BC) 3. (ABC) 4. (A D)
5. [(A)-P, R ; (B)-Q, R ; (C)-P, R ; (D)-Q, S]
6. [(A)-P, S ; (B)-Q,S ; (C)-P, S ; (D)-P, R]
7. (1) 8. (3) 9. (2) 10. (2)
95
JEE-Mathematics
BEGINNER'S BOX-8
ì4 - x; 2 < x £ 3
ï
1. fof ( x ) = í2 + x; 0 £ x £ 1 2. (1)
ï2 - x;1 £ x £ 2
î
p p p
15. (ACD) 16. (i) (ii) (iii) (iv) 2p
2 2 2
p 2p
17. 18. p 19. (i) (ii) 2p (iii) 24 (iv) does not exist (v) 2 n + 1
4 3
(vi) 2p 20. 8 21. 2l 22. [(A)-P ; (B)-Q ; (C)-P ; (D)-Q]
23. [(A)-R ; (B)-R ; (C)-R ; (D)-R] 24. [(A)-R ; (B)-R ; (C)-P ; (D)-Q]
BEGINNER'S BOX-11
1. 1002.5 2. –6/5 3. (D)
4. (D) 5. (D) 6. (A) 7. (B)
8. (B) 9. (D) 10. (C) 11. (B)
12. (A) 13. (B) 14. (B) 15. (A,B,D)
EXERC IS E-1
SINGLE CORR ECT & MORE THAN ONE OPTION CORR ECT
Que. 1 2 3 4 5 6 7 8 9 10
Node-1\Target-2020-21\1.JEE(M+A)\Module\Enthusiast\English\Maths\EM-1\17.Set, Relation & Function\Ex.
Ans. C D C A D C A C A C
Que. 11 12 13 14 15 16 17 18 19 20
Ans. D C C C C A B B A A
Que. 21 22 23 24 25 26 27 28 29 30
Ans. C D D A B B B D C C
Que. 31 32 33 34 35 36 37 38 39 40
Ans. B B A A D C B A A A
Que. 41 42 43 44 45 46 47 48 49 50
Ans. A B A C ABCD AB BC ABCD BC ABD
Que. 51 52
Ans. ABD ABC
96
Set, Relation & Function
EXERCISE-2
Comprehension – 1 1. D 2. B 3. C
Comprehension – 2 4. C 5. A 6. B
l Match the Column 15.(A) ® (r); (B) ® (p); (C) ® (s); (D) ® (q)
16. (A) ® (r, q); (B) ® (p, s); (C) ® (r, s); (D) ® (r, s)
æ 1ö
20. (a) ç - 4, - ÷ È (2, ¥)
è 2ø
é1 - 5 ö é1 + 5 ö
(b) (–1 < x < –1/2) U (x > 1) (c) ê 2 , 0 ÷÷ È ê 2 , ¥ ÷÷
êë ø êë ø
(d) 2Kp < x < (2K + 1)p but x ¹ 1 where K is non-negative integer
3
21. (a) - (b) 64 (c) 30 (d) 102 (e) 5050
4
Node-1\Target-2020-21\1.JEE(M+A)\Module\Enthusiast\English\Maths\EM-1\17.Set, Relation & Function\Ex.
ì x if x£0 ì x2 if x<0
22. ( gof )( x ) = íï -x 2 ï
if 0 < x < 1; ( fog )( x ) = í1 + x if 0 £ x < 1
ï1 - x 2 if x ³1 ï x if x ³1
î î
5
25. (21) 26. 0, 27. 1003.5 28. y
3
97
JEE-Mathematics
NCERT CORNER
1. x = 3 and y = -1
2. x = 2 and y = 1.
3. 9
4. A = {a, b} and B = {x, y}.
5. Domain (f ) = ( -¥, - 1) È (1, 4].
6. Domain (f ) = (0, ¥).
7. f is not onto.
8. Domain of f = [-3, 3], Range = [0, 3]
9. (i) f (0) = 2 ´ 0 - 5 = -5 (ii) f (7) = 2 ´ 7 - 5 = 9 (iii) f ( -3) = 2 ´ (-3) - 5 = -11.
10. R f = (-¥, 2).
11. R f = [2, ¥).
13. G ´ H = {(7, 5), (7, 4), (7, 2), (8, 5), (8, 4), (8, 2)} , H ´ G = {(5, 7), (5, 8), (4, 7), (4, 8), (2, 7), (2, 8)}.
14. A ´ A ´ A = {( -1, - 1, - 1), (-1, - 1,1), ( -1,1, - 1), ( -1,1,1), (1, - 1, - 1), (1, - 1,1), (1,1, - 1), (1,1,1)}
15. (i) {-5, 5}. (ii) Pre-images of 39 are –6 and 6, 2 does not have any pre-image under f.
16. (i) Does not exist. (ii) Does not exist.
17. f
18. (i) Domain ( f ) = R - Z , Range (f ) = (1, ¥). (ii) Domain ( f ) = R , Range (f ) = ( -¥,1].
4 5
3 4
2
2
1 1
19. 20.
1 2 3 4 5
1 2 3 4 5 -1
-1
-2
-2
Node-1\Target-2020-21\1.JEE(M+A)\Module\Enthusiast\English\Maths\EM-1\17.Set, Relation & Function\Ex.
21. D f = R - {2, 6}
22. Df = [1, ¥) , R f = [0, ¥)
23. R f = [0,1). [Q y ¹ 1 ]
æf ö x +1
24. ( f + g ) = 3 x - 2 , ( f - g ) = 4 - x , ç ÷ ( x) =
ègø 2x - 3
25. a = 2, b = –1
26. (i) True (ii) False
****
98
Inverse Trig onometric Fu nctions
1. 0 INTRODUCTION
The inverse trigonometric functions, denoted by sin–1x or (arc sinx), cos–1x etc., denote the angles whose sine,
cosine etc, is equal to x. The angles are usually the numerically smallest angles, except in the case of cot–1x, and
if positive & negative angles have same numerical value, the positive angle has been chosen.
It is worthwhile noting that the functions sinx, cosx etc are in general not invertible. Their inverse is defined by
choosing an appropriate domain & co-domain so that they become invertible. For this reason the chosen value
is usually the simplest and easy to remember.
æ p pö
(3) tan–1x xÎR ç- 2, 2 ÷
è ø
ìpü é pö æp ù
(4) sec–1x |x| ³ 1 ëé0, pûù - í 2 ý or ê0, ÷ È ç , pú
î þ ë 2ø è2 û
é p pù
(5) cosec–1x |x| ³ 1 ê - 2 , 2 ú - {0}
ë û
(6) cot–1x xÎR (0, p)
é -p p ù
(a) f : ê , ú ® [-1, 1] f -1 : [-1, 1] ® [–p/2, p/2]
ë 2 2û
y y
y=arcsinx y=arc sinx
p/2 y=x p/2
Node-1\Target-2020-21\1.JEE(M+A)\Module\Enthusiast\English\Maths\EM-1\18.ITF\Th.
1 y=sinx
–p/2 –1 –1
x x
0 1 p/2 0 1
y=sinx –1
99
JEE-Mathematics
y
y=arc cosx y=x y
p
p/2 p
1
p/2 p p/2
0 x
–1
–1
y=cosx –1 1 x
O
y=x
y
p y=tanx
y=x y
p/2
y=arc tanx
p/2
y=arc tanx
x x
–p/2 0 p/2 p 0
y=arc tanx y=arc tanx
p/2 –p/2
y=x
y=tanx –p
(Taking image of tan x about y = x) (y = tan–1x)
(d) f : (0, p) ® R f -1 : R ® (0, p )
f(x) = cot x f -1 (x) = cot -1 x
y
y=x y
p
Node-1\Target-2020-21\1.JEE(M+A)\Module\Enthusiast\English\Maths\EM-1\18.ITF\Th.
p
y=arc cotx
y=arc cotx
p/2
y=arc cotx p/2
0 x y=arc cotx
0 x
–p/2
y=x –p
y=cotx
100
Inverse Trig onometric Fu nctions
y
(e) f : [0, p/2) È (p/2, p] ® ( -¥, –1] È [1, ¥)
f(x) = sec x p
f -1 (x) = sec -1 x
x
–1 0
(f) f : [–p/2, 0) È (0, p/2] ® ( -¥, –1] È [1, ¥)
y
f(x) = cosec x
p/2
æ 1ö æ 1ö
Illustration 1. The value of tan–1(1) + cos–1 ç - ÷ + sin–1 çè - 2 ÷ø is equal to
è 2ø
p 5p 3p 13p
(A) (B) (C) (D)
4 12 4 12
1 1 p 2p p p p 3 p
Solution tan–1 ( 1) + cos–1 æç - ö÷ + sin–1 æç - ö÷ = + - = + = Ans.(C)
è 2ø è 2ø 4 3 6 4 2 4
2n 2n
Illustration 2 . If å cos -1
x i = 0 then find the value of åx i
i =1 i =1
Hence, each value cos–1x1, cos–1x2,cos–1x3,......,cos–1x2n are non-negative their sum is zero only
when each value is zero.
i.e., cos–1xi = 0 for all i
Þ xi = 1 for all i
2n
= x1 + x2 + x3 + ...... + x2n = {1 + 1 + 1......... + 1}
43 = 2n
\ åx i
1444444444244444444
2n times
{using (i)}
i =1
2n
Þ åx i = 2n Ans.
i =1
101
JEE-Mathematics
1 1
1. cos-1 + 2sin -1 is equal to
2 2
p p p 2p
(A) (B) (C) (D)
4 6 3 3
é -1 æ 3 ö pù
2. The value of cos ê cos ç 2 ÷÷ + 6 ú is
ç -
ëê è ø ûú
(A) 1 (B) –1 (C) 0 (D) None of these
é 1 æ - 1 öù
3. tan ê cos -1 + tan -1 ç ÷ ú is equal to
ë 2 è 3 øû
é -1 -1 æ 3 öù
4. The value of sin ê tan (- 3) + cos çç - ÷ú
ëê è 2 ÷ø ûú is
(A) both cos –1a and cos –1b are real (B) both cosec –1a and cosec –1b are real
(C) both cot –1a and cot –1b are real (D) none of these
6. If sin x + sin y = p and x = ky, then find the value of 39 2k + 5 k .
–1 –1
æ 1 ö æ 1 ö
7. tan -1 ç 1 - x 2 - 2 ÷ + sin -1 ç x 2 + 2 - 1 ÷ (where x ¹ 0) is equal to
è x ø è x ø
p p 3p
(A) (B) (C) (D) p
2 4 4
(A) x Î[ -1,1] (B) x Î( -¥, - 1] È[1, ¥) (C) x Î( -¥, ¥) (D) None of these
æ æ x öö
9. The domain of the function sin -1 ç log 2 ç ÷ ÷ is-
è è 3 øø
Node-1\Target-2020-21\1.JEE(M+A)\Module\Enthusiast\English\Maths\EM-1\18.ITF\Th.
é1 ù é1 ù é3 ù é1 ù
(A) ê , 3ú (B) ê ,3ú (C) ê , 6 ú (D) ê , 2ú
ë2 û ë2 û ë2 û ë2 û
p
(A) x Î (-¥, ¥) (B) x Î[ -1,1] (C) x = (2n + 1) ,n ÎI (D) x = np,n ÎZ
2
11. The domain of the function f (x) = cos -1 (x + [x]), where [ × ] denotes the greatest integer function, is
102
Inverse Trig onometric Fu nctions
12. Domain of the function ƒ(x) = logecos–1 { x} is, where {.} represents fractional part function -
é p pù é p ù é pù
(A) ê - , ú (B) ê -, 0ú (C) ê 0, (D) [–1, 1]
ë 2 2û ë 2 û ë 2 úû
14. Range of ƒ(x) = cot–1(loge(1 – x2)) is -
æ pö ép ö æ pù
(A) (0,p) (B) ç 0, ÷ (C) ê , p ÷ (D) ç 0,
è 2ø ë2 ø è 2 úû
15. Find the domain and range of the following functions .
(Read the symbols [*] and {*} as greatest integers and fractional part functions respectively.)
(i) f (x) = cot-1(2x - x²) (ii) f (x) = sec-1 (log3 tan x + logtan x 3)
æ 2 x2 + 1 ö æ ö
(iii) f(x) = cos-1 ç
ç x +1 ÷
2
÷ -1
(
(iv) f (x) = tan çç log 4 5x - 8x + 4 ÷÷
è
2
)ø
è ø 5
–1
y
(ii) y = cos (cos–1 x) = x
1
x Î [–1,1], y Î [–1,1]
x
y=
45°
–1 O +1 x
–1
Node-1\Target-2020-21\1.JEE(M+A)\Module\Enthusiast\English\Maths\EM-1\18.ITF\Th.
(iii) y = tan(tan–1 x) = x
x Î R, y Î R
y
x
y=
45° x
O
103
JEE-Mathematics
(iv) y = cot(cot–1 x) = x,
x ÎR; y ÎR
y
x
y=
x
O
x
=
y
1
–1 O 1 x
–1
x
=
y
|x| > 1, |y| > 1
(vi) y = sec(sec–1 x) = x |x| > 1 ; |y| > 1
y
x
=
y
1
–1 O 1 x
–1
x
=
y
æp 1 ö
(i) sin(cos–13/5) (ii) cos(tan–1 3/4) (iii) sin ç - sin -1 æç - ö÷ ÷
è2 è 2 øø
Node-1\Target-2020-21\1.JEE(M+A)\Module\Enthusiast\English\Maths\EM-1\18.ITF\Th.
æp æ -1 ö ö æ p æ pöö 2p 3
(iii) sin ç - sin -1 ç ÷ ÷ = sin ç - ç - ÷ ÷ = sin = Ans.
è2 è 2 øø è 2 è 6ø ø 3 2
104
Inverse Trig onometric Fu nctions
é p pù
P-2 (i) y = sin–1 (sin x), x Î R, y Î ê - , ú periodic with period 2p and it is an odd function.
ë 2 2û
y
p
ì p
ï -p - x , -p £ x £ -
y
x
2
2p
=
+
=
2
x
2p
x–
–(
y=
p–
ï p 3p
p+
=
=
x
–
ï p p
y
2
x)
y
45° 2
sin -1 (sin x) = í x , - <x< –2p
–
3p –p O p p 2p x
ï 2 2 2 2
ï p
ïp - x , £x£p
î 2 p
–
2
(ii) y = cos–1 (cos x), x Î R, y Î [0,p], periodic with period 2p and it is an even function.
y
p
x
y
2p
p =
=
– y
ì - x , -p £ x £ 0 + 2
2p
–x
x
cos -1 (cos x) = í
–x
=
y
îx , 0 < x £ p
x
– 2p –p p O p p 2p
– 2
2
ì 3p p
ïx + p , -
2
<x<-
2
ï
p
p
x
x+
x–
y=
-1 ï p p
y=
tan (tan x) = í x - <x<
=
, 2p
y
ï 2 2 –2p 3p –p –p O x
ï p 3p 2 2
ïx - p , <x<
î 2 2
p
–
2
(iv) y = cot–1(cot x), x Î R – {n p, n Î I}, y Î (0, p), periodic with period p and neither even nor odd function.
y
p
p
2p
ì x + p , -p < x < 0
+
x–
x
+
ï
y=
y=
=
x
-1
cot (cot x) = í x , 0<x<p
=
y
y
ï x - p , p < x < 2p
î
–2p –p O p 2p
Node-1\Target-2020-21\1.JEE(M+A)\Module\Enthusiast\English\Maths\EM-1\18.ITF\Th.
é p ö æ pù
(v) y = cosec–1 (cosec x), x Î R – {n p, n Î I} y Î ê - ,0÷ È ç 0, ú , is periodic with period 2p and it is an
ë 2 ø è 2û
odd function.
y
p
2p
y
2
=
=
x
x–
y=
p 3p
–(
p–
–
p+
2 45° 2
y
x)
3p –p O p p 2p x
–
2 2
p
–
2
105
JEE-Mathematics
p p x
2 =
+ p y
y
x –
=
2
y=
–x
2p
ì p ü é pö æ p ù
x Î R – í(2n - 1) n Î I ý , y Î ê0, ÷ È ç , p ú
–
x
î 2 þ ë 2ø è 2 û –2p –p O p 2p x
3p p p 3p
– –
2 2 2 2
Solution ( )
sin–1 - 3 / 2 = – sin–1 ( )
3 /2 = – p / 3
and cos–1 (cos (7p / 6) = cos–1 cos (2p – 5p / 6) = cos–1 cos ( 5 p / 6) = 5p/6
p 5p p
Hence sin–1 (– 3 / 2 ) + cos–1 (cos 7 p / 6) = - + = Ans.(B)
3 6 2
-1 æ 7p ö
(i) sin–1(sin(p/4)) (ii) cos çè cos ÷ø
6
p
Solution (i) sin–1(sin(p/4)) =
4
æ 7p ö 7p 7p
(ii) cos–1 ç cos ÷ ¹ , because does not lie between 0 and p.
è 6 ø 6 6
æ 7p ö æ æ 5p ö ö ì 7p 5p ü
Now, cos–1 çè cos ÷ø = cos–1 çè cos çè 2p - ÷÷ íQ = 2p - ý
6 6 øø î 6 6 þ
æ 5p ö 5p
= cos -1 ç cos ÷ = Ans.
è 6ø 6
Illustration 6. Evaluate the following :
(i) sin –1(sin10) (ii) tan–1(tan (– 6)) (iii) cot–1(cot 4)
–1
Solution (i) We know that sin (sinq) = q, if –p/2 £ q £ p/2
Here, q = 10 radians which does not lie between –p/2 and p/2
Node-1\Target-2020-21\1.JEE(M+A)\Module\Enthusiast\English\Maths\EM-1\18.ITF\Th.
p p
But, 3p – q i.e., 3p – 10 lie between – and
2 2
Also, sin(3p – 10) = sin 10
\ sin–1(sin 10) = sin–1 (sin (3p – 10)) = (3p – 10)
(ii) We know that,
tan–1(tanq) = q, if –p/2 < q < p/2. Here, q = –6, radians which does not lie between
–p/2 and p/2. We find that 2p – 6 lies between –p/2 and p/2 such that;
tan (2p – 6) = –tan 6 = tan(–6)
\ tan–1(tan(–6)) = tan–1 (tan(2p – 6)) = (2p – 6)
(iii) cot–1(cot4) = cot–1(cot(p + (4 – p))) = cot–1(cot(4 – p)) = (4 – p) Ans.
106
Inverse Trig onometric Fu nctions
Illustration 7. Prove that sec2(tan–12) + cosec2(cot–1 3) = 15
Solution We have,
sec2 (tan–12) + cosec2 (cot–13)
2 2
ì æ 2 öü ì æ 3 öü
{ ( )} + {cos ec ( cot 3)}
2 2
= sec tan 2-1 -1
= ísec ç tan -1 ÷ ý + ícos ec ç cot -1 ÷ ý
î è 1 øþ î è 1 øþ
{ ( )} + {cosec ( cosec )} = ( 5 ) + ( )
2 2 2 2
= sec sec -1 5 –1
10 10 = 15
Illustration 8. Find the number of solutions of (x, y) which satisfy |y| = cos x and y = sin–1(sin x), where
|x| £ 3p.
Solution Graphs of y = sin–1(sinx) and |y| = cosx meet exactly six times in [–3p, 3p].
5p p 3p
–3p 2 2 O 2 3p
x
–2p 3p p 2p 5p
2 2 2
æ -1 æ 8 ö ö æ1 -1 æ 4 ö ö æ -1 æ 3 ö ö
(a) tan ç cos ç ÷ ÷ (b) sin ç cos ç ÷ ÷ (c) cos ç sin ç - ÷ ÷
è è 17 ø ø è2 è 5 øø è è 5 øø
æp -1 æ - 1 ö ö æ -1 1 ö æ -1 3 ö
(d) sin ç - sin ç ÷÷ (e) cos çè sin (f) sin ç cos
è 3 è 2 øø 2 ÷ø è 5 ÷ø
-1 æ 13p ö -1 æ æ 7p ö ö æ æ 5p ö ö
(a) cos ç cos (b) tan ç tan ç ÷ ÷ (c) sin –1(sin2) (d) sin -1 ç sin ç
è 6 ÷ø è è 6 øø è è 6 øø
÷÷
-1 æ 2p ö -1 æ 3p ö -1 æ 4p ö
(e) tan ç tan ÷ (f) tan ç tan (g) cos ç cos
è 3ø è 4 ÷ø è 3 ÷ø
æ æ 8p ö æ 8p ö ö
3. cosç cos -1 cosç ÷ + tan -1 tanç ÷ ÷ has the value equal to
è è 7ø è 7 øø
Node-1\Target-2020-21\1.JEE(M+A)\Module\Enthusiast\English\Maths\EM-1\18.ITF\Th.
p
(A) 1 (B) –1 (C) cos (D) 0
7
é æ 50 p ö æ 31p ö ù
4. The value of sec êsin -1 ç - sin -1
÷ + cos cos ç - ÷ is equal to
ë è 9 ø è 9 ø úû
10 p p
(A) sec (B) sec (C) 1 (D) –1
9 9
107
JEE-Mathematics
2æ -1 1ö 2æ -1 1 ö
5. The value of sin ç cos ÷ + cos ç sin ÷ is
è 2ø è 3ø
17 59 36
(A) (B) (C) (D) None of these
36 36 59
é1 æ æ 14 p ö ö ù
6*. The value of cos ê cos-1 ç cos ç - ÷ ÷ ú is :
ë2 è è 5 øø û
æ 7 pö æ pö æ 2pö æ 3p ö
(A) cos ç - ÷ (B) sin ç ÷ (C) cos ç ÷ (D) - cos ç ÷
è 5ø è 10 ø è 5ø è 5ø
-1
é 1 - sin x + 1 + sin x ù æ pö
7. The value of cot ê ú is ( x Î ç 0, ÷ )
ë 1 - sin x - 1 + sin x û è 2ø
x x
(A) p - x (B) 2 p - x (C) (D) p -
2 2
11. If 2 £ a < 3, then the value of cos -1 cos[a] + cosec-1 cosec[a] + cot -1 cot[a] , (where [.] denotes greatest
integer less than equal to x) is equal to
(A) 2 – p (B) 2 + p (C) p (D) 6
1 - x2 1 + x2 x 1- x2
(A) ± ,x ¹ 0 (B) ,x ¹ 0 (C) (D) ,x ¹ 0
x x 1 + x2 x
x2 +1
14. Prove that sin cot cos tan x =
–1 –1
x2 + 2
15. Prove that : sin cot–1 tan cos–1 x = sin cosec–1 cot tan–1x = x where x Î (0,1]
108
Inverse Trig onometric Fu nctions
p
P-3 (i) sin–1 x + cos–1 x = –1 < x < 1
2
p
(ii) tan–1 x + cot–1 x = xÎR
2
p
(iii) cosec–1 x + sec–1 x = |x| > 1
2
P-4 (i) sin–1 (–x) = – sin–1 x , –1 < x < 1
(ii) cosec–1(–x) = – cosec–1 x, x < –1 or x > 1
(iii) tan–1 (–x) = – tan–1 x , xÎR
(iv) cot–1 (–x) = p – cot–1 x , xÎR
(v) cos–1 (–x) = p – cos–1 x , –1 < x < 1
(vi) sec–1 (–x) = p – sec–1 x , x <–1 or x > 1
1
P-5 (i) cosec–1 x = sin–1 ; x <–1, x >1
x
1
(ii) sec–1 x = cos–1 ; x <–1, x >1
x
ì -1 1
ï tan ; x>0
ï x
(iii) cot -1 x = í
ï p + tan -1 1 ; x < 0
ïî x
-1 -1 1 ì p / 2 , if x > 0
Illustration 9. Prove that tan x + tan =í
x î -p / 2 , if x < 0
æ 1 ö ì cot x
-1
, for x > 0
Solution We have , tan -1 ç ÷ = í
è x ø î -p + cot x , for x < 0
-1
1. Prove that tan (tan–1 x + tan–1 y + tan–1 z) = cot ( cot–1 x + cot–1 y + cot–1 z)
Node-1\Target-2020-21\1.JEE(M+A)\Module\Enthusiast\English\Maths\EM-1\18.ITF\Th.
1
2. Find the value of cos (2 cos-1 x + sin-1 x) at x =
5
1
3. Find the value of sin(tan –1a + tan –1 ); a ¹ 0
a
-1 -1 -1
4*. If range of f(x) = tan x + cot x + sin x is [a, b] then (x Î R)
p p
(A) a = 0 (B) b = (C) a = (D) b = p
2 4
109
JEE-Mathematics
æ a 2 a3 ö p
5. Number of integral ordered pairs (a,b) for which sin–1(1 + b + b2 +.... ¥) + cos -1 ç a - + - .....¥ ÷ = is -
è 3 9 ø 2
(A) 0 (B) 4 (C) 9 (D) Infinitely many
6. Prove the following
æ 5 ö æ 12 ö
(a) cos -1 ç ÷ = tan -1 ç ÷
è 13 ø è 5 ø
æ 4ö æ 4ö æ 3ö
(b) sin -1 ç - ÷ = tan -1 ç - ÷ = cos -1 ç - ÷ - p
è 5ø è 3ø è 5ø
7. Evaluate : sin [ cos–1 (3/5) + tan–1 (– 2 )]
If a is only real root of the equation x3 + (cos1) x2 + (sin1) x + 1 = 0, then æç tan -1 a + tan -1 ö÷ cannot
10*.
1
è aø
be equal to-
x+y
ì tan 1 - xy where x > 0, y > 0 & xy < 1
–1
ï
ïï x+y
P-6 (i) (a) tan–1 x + tan–1 y = í p +tan–1 where x > 0, y > 0 & xy > 1
1 - xy
ï
ï
ïî p , where x > 0, y > 0 & xy = 1
2
x-y
(b) tan–1 x – tan–1 y = tan–1 where x > 0, y > 0
1 + xy
é x + y + z - xyz ù
(c) tan–1x + tan–1y + tan–1 z = tan–1 ê ú if x > 0, y > 0, z > 0 & xy + yz + zx < 1
ë 1 - xy - yz - zx û
ì -1 ( 2
ïcos xy + 1 - x 1 - y
2
; ) x < y, x, y > 0
(b) cos x – cos y = í
–1 –1
(
ïî- cos -1 xy + 1 - x 2 1 - y 2 ; ) x > y, x, y > 0
110
Inverse Trig onometric Fu nctions
Note – In the above results x & y are taken positive. In case if these are given as negative, we first apply P-4 and
then use above results.
æ -1 1 1ö æ 1 1ö
(ii) çè tan + tan-1 ÷ + ç tan-1 + tan-1 ÷
5 7 ø è 3 8ø
æ 1 1 ö æ 1 1 ö
+ +
-1 ç 5 7 ÷ -1 ç 3 8 ÷ æ 6ö æ 11 ö
= tan ç + tan = tan-1 ç ÷ + tan-1 ç ÷
1 1÷ ç 1 1÷ è 17 ø è 23 ø
ç1- ´ ÷ ç1- ´ ÷
è 5 7ø è 3 8ø
æ 6 11 ö
ç + ÷ æ 325 ö p
= tan ç 17 23 ÷ = tan-1 ç
-1 -1
÷ = tan (1) = Ans.
çç 1 - 6 11 ÷÷ è 325 ø 4
´
è 17 23 ø
12 4 63
Illustration 11. Prove that sin -1 + cot -1 + tan -1 =p
13 3 16
Solution We have,
12 4 63
sin -1 + cos -1 + tan -1
13 5 16
12 3 63 é - 1 12 -1 12 -1 4 -1 3 ù
êQ sin 13 = tan 5 and cos 5 = tan 4 ú
-1
= tan + tan-1 + tan -1
5 4 16 ë û
ì 12 3 ü
ï + ï 63 é ù
p + tan-1 í 5 4 ý + tan-1 -1 æ x + y ö
êQ tan x + tan y = p + tan ç
-1 -1
= 12 3 16 ÷ ,if xy > 1ú
ï1 - ´ ï ë è 1 - xy ø û
î 5 4þ
-1 æ 63 ö æ 63 ö
= p + tan ç ÷ + tan -1 ç ÷
è -16 ø è 16 ø
Node-1\Target-2020-21\1.JEE(M+A)\Module\Enthusiast\English\Maths\EM-1\18.ITF\Th.
111
JEE-Mathematics
Solution Here,
ì 1
x = cosec(tan–1(cos(cot–1(sec(sin–1a))))) í Let sinq = a Þ secq =
î 1 - a2
= cosec(tan–1(cos(cot–1(secq))))
æ æ æ æ 1 öööö ì -1 æ 1 ö 1
Þ x = cosec ç tan -1 ç cos ç cot -1 ÷ ï Let cot ç = f Þ cot f =
ç 2 ÷÷÷ 2 ÷
è 1-a ø
è è è è 1 - a øøøø 1 - a2
ïï
í
ï
ï
ïî 1
= cosec(tan–1(cosf)) therefore cos f =
2 - a2
ì
ï
æ æ 1 öö ïï
æ 1 ö 1
Þ x = cosec ç tan -1 ç í Let, tan -1 ç = y Þ tan y =
è è 2 - a 2 ÷ø ÷ø ï è 2 - a 2 ÷ø 2 - a2
ï
ïî
Þ x = 3 - a2 ...... (i)
ì 1
and y = sec(cot–1(sin(tan–1(cosec(cos–1 a))))) í Let cos–1a = a Þ cosa = aÞ coseca =
î 1 - a2
= sec(cot–1(sin(tan–1(coseca))))
= sec(cot–1(sin(b)))
2 - a2
ìï 1 1
æ 1 öö
Þ y = sec cot -1 æ
-1
íLet cot = g Þ cot g = Þ sec g = 3 - a 2
ç çè ÷÷ ïî 2- a 2
2 - a2
è 2 - a2 ø ø
= sec g
Þ y = 3 - a2 ...... (ii)
–p/2
y
p
D p/2 I
-1
æ 1 - x2 ö é 2 tan -1 x if x ³ 0
(b) y = f(x) = cos ç ÷ =ê
è 1 + x2 ø êë -2 tan -1 x if x < 0
–1 0 1 X
y
p/2
I I
é 2 tan -1 x if |x|< 1 x
-1 2x
ê -1
–1 1
(c) y = f(x) = tan = ê p + 2 tan x if x < -1
1 - x2 ê I
-1 I
êë - (p - 2 tan x) if x > 1
–p/2
y
(d) y = f(x) = sin (3x – 4x )
–1 3 p/2
é -1 1
ê -(p + 3sin x) if -1 £ x < - 2 D I D
ê
1 1 – 3/2
= ê 3sin-1 x
–1/2 +1/2
if - £ x £ x
ê –1 3/2 1
2 2 I
ê 1
ê p - 3sin-1 x if < x£1
êë 2
Node-1\Target-2020-21\1.JEE(M+A)\Module\Enthusiast\English\Maths\EM-1\18.ITF\Th.
–p/2
y
(e) y = f(x) = cos -1 (4x 3 - 3x) p
é -1 1
I
ê3cos x - 2p if -1 £ x < - 2 D D
ê
1 1 p/2
= ê2p - 3cos-1 x if - £ x £
ê 2 2
ê 1
ê 3cos x-1
if < x£1 I
êë 2 x
–1/2 +1/2
–1 – 3 /2 3/2 1
113
JEE-Mathematics
p/2
ì ( -1 ) 1 1
ï - p + 2sin x -1 £ x < -
2
– x
ï 2
-1 ( 2) ï -1 1 1 1
(f) sin 2x 1 - x = í2sin x - £x£
2
ï 2 2
ï -1 1
ï p - 2sin x < x£1 -p/2
î 2
y
p
x
–1
2c
s
co
os
–2
–1
-1
ïì2cos x 0 £ x £ 1
(g) cos ( 2x - 1) = í
x
2p
-1 2
-1
ïî2p - 2cos x -1 £ x < 0 –1 0 1 x
ì 1 pü ïì1 -1 5 ïü
Illustration 14. Evalulate : (i) tan í2 tan -1 - ý (ii) tan í cos ý
î 5 4þ ïî 2 3 ïþ
ì æ 1 ö ü
ì 1 pü ï ç 2´ 5 ÷ ï é -1 æ 2x ù
Solution (i) tan í2 tan -1 - ý = tan ïí tan -1 ç -1 ï
÷ - tan 1ý êQ2 tan x = tan ç
-1 ö
2 ÷
, if|x|< 1ú
î 5 4þ ï çç 1 - 1÷÷ ï ë è1- x ø û
ïî è 25 ø ïþ
ì æ 5 öü
ïï -1 ç 12 - 1 ÷ ïï ì -1 æ -7 ö ü -7
ì -1 5 ü ÷ ý = tan í tan ç ÷ý =
= tan í tan - tan -1 1ý = tan í tan ç 5
î 12 þ ï çç 1 + ÷÷ ï î è 17 ø þ 17
îï è 12 ø þï
5 5
(ii) Let cos-1 = q. Then, cosq = , 0 < q < p/2
3 3
æ1 5ö
Now, tan çç cos -1 ÷
è2 3 ÷ø
Node-1\Target-2020-21\1.JEE(M+A)\Module\Enthusiast\English\Maths\EM-1\18.ITF\Th.
5
1-
q 1 - cos q 3
= tan = =
2 1 + cos q 5
1+
3
3- 5 (3 - 5 )2 (3 - 5)2 3 - 5
= = = =
3+ 5 (3 + 5)(3 - 5) 9-5 2
114
Inverse Trig onometric Fu nctions
1 1 31
Illustration 15. Prove that : 2 tan -1 + tan -1 = tan -1
2 7 17
-1 1 1
Solution We have, 2 tan + tan -1
2 7
ì ü
ï 2´ 1 ï
-1 ï 2 ï + tan -1 1 é -1 æ 2 x ö ù
êQ2 tan x = tan ç
-1
= tan í 2ý 2 ÷
,if - 1 < x < 1ú
ï1 - æ 1 ö ï 7 ë è1 - x ø û
ï ç2÷ ï
î è ø þ
ì 4 1 ü
ï +
tan
4-1
+ tan -1 1
= tan -1 ï 3 7 ïï = tan-1 31
= í ý
3 7 ï1 - 4 ´ 1 ï 17
ïî 3 7 ïþ
1 æ1- x ö
Illustration 16. Prove that tan -1 x = cos -1 ç ÷ , x Î éë0,1ùû
2 è1 + x ø
ì1 -
( x) ü
2
1 -1 æ 1 - x ö 1 -1 ï ï 1
Solution We have, cos ç ÷ = 2 cos í 2 ý
= ´ 2 tan-1 x = tan -1 x.
2 è1 + x ø ï1 +
î ( x) ï 2
þ
é pù
Alter : Putting x = tan q, we have Þ q Î ê0, ú
ë 4û
1 æ1 - x ö 1 æ 1 - tan2 q ö 1 æ é pù ö
RHS = cos -1 ç ÷ = cos -1 ç -1
÷ = cos (cos 2q) = q Q ç 2q Î ê0, ú ÷
2 è1 + x ø 2
2
è 1 + tan q ø 2 è ë 2û ø
= tan -1 x = LHS
1 1 1 p
Illustration 17. Prove that : (i) 4 tan -1 - tan -1 + tan -1 =
5 70 99 4
1 5 2 1 p
(ii) 2 tan -1 + sec -1 + 2 tan -1 =
5 7 8 4
1 1 1 ì 1ü 1 1
Solution (i) 4 tan -1 - tan -1 + tan -1 = 2 í2 tan -1 ý - tan -1 + tan -1
5 70 99 î 5þ 70 99
é Q2 tan -1 x ù
ì 2 ´1 / 5 ü 1 1 ê ú
= 2 í tan -1 2 ý
- tan -1 + tan -1 ê = tan -1 2x ,if|x|< 1ú
Node-1\Target-2020-21\1.JEE(M+A)\Module\Enthusiast\English\Maths\EM-1\18.ITF\Th.
î 1 - (1 / 5) þ 70 99
ëê 1 - x2 úû
ì 1 1 ü
ì 2 ´ 5 /12 ü ïï 70 - 99 ïï
5 ì 1 1 ü = tan í
-1 -1
- tan . í
= 2 tan -1 - í tan -1 - tan -1 ý 2ý ý
12 î 70 99 þ î1 - (5 /12) þ ï1 + 1 ´ 1 ï
îï 70 99 þï
ì 120 1 ü
120 29 -
120 1 ïï ïï p
= tan-1 í 119 239 ý = tan -1 1 =
-1
= tan - tan -1 = tan-1 - tan-1
119 6931 119 239 ï 120 1 ï 4
1+ ´
îï 119 239 þï
115
JEE-Mathematics
1 5 2 1 ì -1 1 1ü 5 2
(ii) 2 tan -1 + sec -1 + 2 tan -1 = 2 ítan + tan-1 ý + sec -1
5 7 8 î 5 8 þ 7
ì 1 1 ü
ïï 5 + 8 ïï
2
-1 -1
æ5 2 ö
= 2 tan í ý + tan çç ÷÷ - 1 éQsec -1 x = tan -1 x 2 - 1 ù
1 1
ï1 - ´ ï è 7 ø êë úû
îï 5 8 þï
13 1 1 1
= 2 tan -1 + tan -1 = 2 tan -1 + tan -1
39 7 3 7
ì 2´1/ 3 ü 1 é 2x ù
= tan -1 í + tan -1 êQ2 tan x = tan 1 - x 2 , if|x|< 1ú
-1 -1
2 ý
î1 - (1 / 3) þ 7 ë û
ì 3 1 ü
ï +
= tan -1 3
+ tan -1 1 -1 ï 4
= tan í 7 ïï = tan-1 1 = p
ý
4 7 ï1 - 3 ´ 1 ï 4
ïî 4 7þ ï
1. Prove that:
-1 3p 41 p
(i) tan 2 + tan -1 3 = (ii) cot
-1
9 + cos ec -1 =
4 4 4
2 6 +1 p
(iii) arc cos - arc cos = (iv) 4 tan–1 (1/5) – tan–1 (1/70) + tan–1 ( 1/99) = p /4.
3 2 3 6
(v) tan–1 1 + tan–1 2 + tan–1 3 = 2 [tan–1 1 + tan–1 1/2 + tan–1 1/3]
3 16 1 7
(vi) 2 cos -1 + cot -1 + cos -1 =p
13 63 2 25
2. Prove the following results
-1 æ 1 ö -1 æ 1 ö -1 æ 4 ö -1 æ 3 ö -1 æ 17 ö p
(a) 2 tan ç ÷ + tan ç ÷ = tan ç ÷ (b) 2 sin ç ÷ - tan ç ÷ =
5
è ø 8
è ø è 7ø è5ø è 31 ø 4
Node-1\Target-2020-21\1.JEE(M+A)\Module\Enthusiast\English\Maths\EM-1\18.ITF\Th.
-1 æ 3 ö -1 æ 8 ö -1 æ 36 ö -1 æ 3 ö -1 æ 3 ö -1 æ 8 ö p
(a) sin ç 5 ÷ + sin ç 17 ÷ = cos ç 85 ÷ (b) tan ç ÷ + tan ç ÷ - tan ç ÷=4
è ø è ø è ø 4
è ø 5
è ø è 19 ø
-1 æ 2 ö -1 æ 7 ö æ1ö
(c) tan ç ÷ + tan ç ÷ = tan -1 ç ÷
è 11 ø è 24 ø è2ø
116
Inverse Trig onometric Fu nctions
æxö x-y
5. tan -1 ç ÷ - tan -1 is
è yø x+y
p p p p 3p
(A) (B) (C) (D) or -
4 3 2 4 4
é 1 - ab ù
Prove the following : tan a + tan b = cos ê ú (where a,b>0)
6.
{(1 + )(1 + )}
–1 –1 –1
ê a 2
b 2 ú
ë û
é1 æ 2a ö 1 æ 1 - a 2 öù
7. Prove the following : tan ê sin -1 çç ÷÷ + cos -1 ç ÷ú = 2a
è1 + a 2 ø 2 ç1 + a 2 ÷ 1 - a 2
êë 2 è øúû
-1 é æa - bö 1 ù -1 æ a cos q + b ö
8. Prove that: 2 tan ê ç ÷ tan q ú = cos ç ÷.
êë è a + b ø 2 úû è a + b cos q ø
ìa ü ìb ü -1 ì c ü
tan -1 í ( a + b + c ) ý + tan -1 í ( a + b + c ) ý + tan í ( a + b + c )ý = p
î bc þ î ca þ î ab þ
-1 æyz ö -1 æ zx ö -1 æ xy ö p 2 2 2 2
12. Prove that: tan ç ÷ + tan çç ÷÷ + tan ç ÷ = , where r = x + y + z .
è xr ø è yr ø è zr ø 2
13. If cos–1 x + cos–1y + cos–1z= p , show that x2 + y2 + z2 +2 xyz =1.
11p
Illustration 18. The equation 2cos–1x + sin–1x = has
6
(A) no solution (B) only one solution
(C) two solutions (D) three solutions
Node-1\Target-2020-21\1.JEE(M+A)\Module\Enthusiast\English\Maths\EM-1\18.ITF\Th.
11p
Solution Given equation is 2 cos–1 x + sin–1x =
6
11p p 11p
Þ cos–1x + ( cos–1 x + sin–1x ) = Þ cos–1 x + = Þ cos–1 x = 4p / 3
6 2 6
117
JEE-Mathematics
(p/2)2 – 2tan–1 x (p/2 – tan–1 x ) = 5p2 / 8
Þ 2(tan–1 x)2 – 2 (p/2) tan–1 x – 3p2 / 8 = 0 Þ tan–1 x = – p / 4 Þ x = –1 Ans. (A)
x -1 x +1 p
Illustration 20. Solve the equation : tan-1 + tan -1 =
x-2 x+2 4
x -1 x +1 p
Solution tan-1 + tan -1 =
x-2 x+2 4
taking tangent on both sides
æ æ x - 1ö æ x + 1öö
Þ tan ç tan -1 ç + tan -1 ç =1
è è x - 2 ÷ø è x + 2 ÷ø ÷ø
æ æ x - 1ö ö æ æ x + 1ö ö
tan ç tan-1 ç + tan ç tan -1 ç
è è x - 2 ÷ø ÷ø è è x + 2 ÷ø ÷ø
Þ =1
æ æ x - 1öö æ æ x + 1öö
1 - tan ç tan -1 ç tan ç tan-1 ç
è è x - 2 ÷ø ÷ø è è x + 2 ÷ø ÷ø
x -1 x +1
+
x -2 x + 2 =1
Þ
x -1 x +1
1- .
x-2 x+2
(x - 1)(x + 2) + (x - 2)(x + 1)
Þ =1
x2 - 4 - (x 2 - 1)
1
Þ 2x2 – 4 = – 3 Þ x=±
2
1
Now verify x=
2
æ 1 ö æ 1 ö
-1 +1
ç 2 ÷ ç 2 ÷ æ 2 -1 ö æ 2 +1 ö
= tan -1 ç -1
÷ + tan ç ÷ = tan -1 ç ÷ + tan -1 ç
ç 1
- 2 ÷÷ ç 1
+ 2 ÷÷ è 2 2 - 1ø è 2 2 + 1÷ø
çè ø çè ø
2 2
æ (2 2 + 1)( 2 - 1) + ( 2 2 - 1)( 2 + 1) ö æ 6ö p
= tan-1 ç ÷ = tan -1 ç ÷ = tan-1 (1) =
è (2 2 - 1)( 2 2 + 1) - ( 2 - 1)( 2 + 1) ø è 6ø 4
1
x=–
2
æ 1 ö æ 1 ö
- -1 - +1
ç 2 ÷ ç 2 ÷
= tan -1 ç -1
÷ + tan ç ÷
ç- 1 1
çè - 2 ÷÷ ç-
çè + 2 ÷÷
2 ø 2 ø
Node-1\Target-2020-21\1.JEE(M+A)\Module\Enthusiast\English\Maths\EM-1\18.ITF\Th.
-1
æ 2 +1 ö æ 2 -1 ö
= tan ç ÷ + tan -1 ç {same as above}
è 2 2 + 1ø è 2 2 - 1 ÷ø
p
= tan -1 (1) =
4
1
\ x =± are solutions Ans.
2
Illustration 21. Solve the equation : 2 tan–1(2x + 1) = cos–1x.
Solution Here, 2 tan–1(2x + 1) = cos–1x
ì 1 - tan2 q ü
or cos(2tan–1(2x +1)) = x í We know cos2q = ý
î 1 + tan2 q þï
118
Inverse Trig onometric Fu nctions
1 - (2x + 1)2
\ = x
1 + (2x + 1)2
Þ (1 – 2x – 1)(1 + 2x + 1) = x(4x2 + 4x + 2)
Þ – 2x . 2(x + 1) = 2x(2x2 + 2x + 1) Þ 2x(2x2 + 2x + 1 + 2x + 2) = 0
Þ 2x(2x2 + 4x + 3) = 0 Þ x= 0 or 2x2 + 4x + 3 = 0 {No solution}
Verify x = 0
p p
2tan–1(1) = cos–1(1) Þ =
2 2
\ x = 0 is only the solution Ans.
Illustration 22. Find the complete solution set of sin–1(sin5) > x2 – 4x.
Solution sin–1(sin5) > x2 – 4x Þ sin–1[sin(5 – 2p)] > x2 – 4x
Þ x2 – 4x < 5 – 2p Þ x2 – 4x + (2p – 5) < 0
Illustration 23. Find the complete solution set of [cot–1x]2 – 6[cot–1x] + 9 £ 0, where [.] denotes the greatest
integer function.
Solution [cot–1x]2 – 6[cot–1x] + 9 £ 0
Þ ([cot–1x] – 3)2 £ 0 Þ [cot–1x] = 3 Þ 3 £ cot–1x < 4 Þ x Î (–¥, cot3]
n p
Illustration 24. If cot –1 > , n Î N , then the maximum value of n is -
p 6
(A) 1 (B) 5 (C) 9 (D) None of these
n p
Solution cot –1 >
p 6
æ æ n öö æ pö n
Þ cot ç cot -1 ç ÷ ÷ < cot ç ÷ Þ < 3
è p
è øø 6
è ø p
Þ n<p 3 Þ n < 5.5 (approx)
Þ n=5 Q (n Î N) Ans. ( B)
8. 0 SUMMATION OF SERIES
æc x-yö -1 æ c 2 - c1 ö -1 æ c 3 - c 2 ö -1 æ c n - c n -1 ö -1 æ 1 ö -1 æ x ö
tan-1 ç 1 ÷ + tan ç ÷ + tan ç ÷ + ... + tan ç ÷ + tan ç ÷ = tan ç ÷
è c1 y + x ø è 1 + c 2 c1 ø è 1 + c 3 c2 ø è 1 + c n c n -1 ø è cn ø èyø
Solution L.H.S.
æc x-yö -1 æ c2 - c1 ö -1 æ c 3 - c 2 ö -1 æ c n - c n -1 ö -1 æ 1 ö
tan-1 ç 1 ÷ + tan ç ÷ + tan ç ÷ + ... + tan ç ÷ + tan ç ÷
c
è 1 y + x ø è 1 + c c
2 1 ø è 1 + c c
3 2 ø è 1 + c c
n n -1 ø è cn ø
æ x 1 ö æ 1 1 ö æ 1 1 ö æ 1 1 ö
ç y-c ÷ ç c -c ÷ ç c -c ÷ ç c -c ÷ æ1ö
= tan ç -1 1 ÷ + tan ç 1
-1 2 ÷ + tan ç 2
-1 3 ÷ + ... + tan ç
-1 n -1 n ÷ + tan -1 ç ÷
ç1 + x . 1 ÷ ç1 + 1 . 1 ÷ ç1+ 1 . 1 ÷ ç1 + 1 . 1 ÷ è cn ø
ç y c1 ÷ ç c1 c 2 ÷ ç c2 c 3 ÷ ç c n -1 c n ÷
è ø è ø è ø è ø
119
JEE-Mathematics
æ -1 x 1ö æ 1 1ö æ 1 1ö
= ç tan - tan -1 ÷ + ç tan-1 - tan -1 ÷ + ç tan -1 tan -1 ÷ +......
è y c1 ø è c1 c2 ø è c2 c3 ø
æ 1 1ö æ1 ö
+ ç tan -1 - tan-1 ÷ + tan -1 ç ÷ = tan -1 æç x ö÷ = R.H.S.
è c n-1 cn ø è cn ø èyø
-1 -1 p
(c) cot x - cot (x + 2) = , where x > 0.
12
2. Solve the following equations / system of equations :
-1 -1 p
(i) sin x + sin 2 x =
3
-1 1 1 2
(ii) tan + tan -1 = tan -1
1 + 2x 1 + 4x x2
(iii) tan–1 ( x –1 ) + tan–1 ( x )+ tan–1 ( x+1)= tan–1 ( 3x )
-1 1 p
(iv) sin + cos -1 x =
5 4
-1 -1 2p p
(v) sin x + sin y = & cos -1 x - cos -1 y =
3 3
3. Solve the following equations / system of equations:
(a) tan–1 ( x + 1)+ tan–1 (x – 1) = tan–1 (8/31).
(b) sin [ 2cos–1 {cot (2 tan–1 x )}] = 0
æ x + 1ö -1 æ x - 1 ö
÷ = tan (- 7 ).
-1 -1
(c) tan ç ÷ + tan ç
è x - 1ø è x ø
-1 5p2
4. If (sin x) 2 + (cos -1 x) 2 = , then x is equal to
8
1 1
Node-1\Target-2020-21\1.JEE(M+A)\Module\Enthusiast\English\Maths\EM-1\18.ITF\Th.
-1 x p
6. If tan < , x Î N , then the maximum value of x is
p 3
120
Inverse Trig onometric Fu nctions
¥
æ 2 ö
7. Evaluate : å tan-1 çè 1 + (2r + 1)(2r - 1) ÷ø
r =1
æ ab + 1 ö æ bc + 1 ö æ ca + 1 ö
8. If a > b > c > 0 then find the value of : cot–1 ç ÷ + cot–1 ç ÷ + cot–1 ç ÷.
è a-b ø è b-c ø è c-a ø
¥
1
å tan -1
9. Evaluate:
1 (n 2
+ n +1 ).
10. Find the sum of the series:
1 2 2 n -1
(a) tan-1 + tan-1 + ..... + tan-1 1 + 2 2 n - 1 + ..... ¥
3 9
1 1 1 1
(b) tan-1 + tan-1 + tan-1 + tan-1 + ..... ¥
2 8 18 32
11. If a1, a2, a3, ….. form an A. P. with a common difference d, a i > 0, d > 0.
-1 d d d -1 nd
Prove that tan + tan-1 + tan-1 +…to n terms = tan .
1 + a1 a2 1 + a 2 a3 1 + a3a4 1 + a 1 a n +1
1 2 -1 n - n-1
13. Evaluate: sin-1 + sin-1 + ..... + sin-1 + ...... ¥
2 6 n (n + 1)
p
Illustration 1. If tan–1 y = 4 tan–1 x, æç|x|< tan ö÷ , find y as an algebraic function of x and hence prove that
è 8ø
p
tan is a root of the equation x4 – 6x2 + 1 = 0.
8
Solution We have tan–1 y = 4 tan–1 x
2x
Node-1\Target-2020-21\1.JEE(M+A)\Module\Enthusiast\English\Maths\EM-1\18.ITF\Th.
4x
1 - x2
-1 4x(1 - x2 ) æ 2x ö
= tan = tan -1 çè as < 1÷
4x 2 x 4 - 6x2 + 1 1 - x2 ø
1-
(1 - x 2 )2
4x(1 - x2 )
Þ y=
x 4 - 6x2 + 1
p -1 -1 p
If x = tan Þ tan y = 4 tan x = Þy is not defined Þ x4 – 6x2 + 1 = 0 Ans.
8 2
121
JEE-Mathematics
2p
Þ A > 2tan–1 ( 3 ) Þ A> ..... (i)
3
æ 1ö æ 1ö æ 1ö p
also we have, sin -1 ç ÷ < sin -1 ç ÷ Þ sin -1 ç ÷ <
è 3ø è 2ø è 3ø 6
Þ æ 1ö p
3 sin -1 ç ÷ <
è 3ø 2
æ 1 æ 1ö ö -1 æ 23 ö
3
æ 1ö –1
also, 3 sin -1 ç ÷ = sin -1 ç 3. - 4 ç ÷ ÷ = sin çè ÷ø = sin (0.852)
è 3ø è 3 è 3ø ø 27
2p
Hence, B = 3sin–1 (1/3) + sin–1 (3/5) < ........ (ii)
3
From (i) and (ii), we have A > B.
Illustration 3. Solve for x : If [sin–1cos–1sin–1tan–1x] = 1, where [.] denotes the greatest integer function.
Solution We have, [sin–1cos–1sin–1tan–1x] = 1
p
Þ 1 £ sin–1 . cos–1 . sin–1 . tan–1x £ Þ sin1 £ cos–1 . sin–1 . tan–1x £ 1
2
Þ cos sin1 ³ sin–1 . tan–1x ³ cos1 Þ sin cos sin1 ³ tan–1x ³ sin cos1
Þ tan sin cos sin1 ³ x ³ tan sin cos1
Hence, x Î [tan sin cos 1, tan sin cos sin1] Ans.
1 æ 3sin 2q ö
Illustration 4. If q = tan–1(2 tan2q) - sin -1 ç then find the sum of all possible values of tanq.
2 è 5 + 4cos2q ÷ø
1 æ 3sin 2q ö
Solution q = tan–1(2 tan2q) - sin -1 ç Þ q = tan–1(2 tan2q)
2 è 5 + 4 cos 2q ÷ø
1 æ 6 tan q ö
- sin -1 ç
2 è 9 + tan2 q ÷ø
Node-1\Target-2020-21\1.JEE(M+A)\Module\Enthusiast\English\Maths\EM-1\18.ITF\Th.
é æ1 ö ù
2 tan q÷ ú
1 -1 ê çè 3 ø
Þ q = tan (2 tan q) - sin ê
–1 2
2
ú Þ q = tan–1(2 tan2q)
2 ê æ1 ö ú
ê1 + çè 3 tan q÷ø ú
ë û
2 æ1 ö
- tan-1 ç tan q÷
2 è 3 ø
122
Inverse Trig onometric Fu nctions
-1 æ 1 ö
Þ q = tan–1(2 tan2q) - tan çè tan q÷ø ........ (i)
3
taking tangent on both sides
6 tan2 q - tan q
Þ tan q =
3 + 2 tan3 q
Þ 2tanq(tan3q –3 tanq + 2) = 0
\ sum = 0 + 1 – 2 = –1 Ans.
Illustration 5. Transform sin–1x in other inverse trigonometric functions, where x Î (–1, 1) – {0}
Solution Case -I : 0 < x < 1
æ pö
Let sin–1x = q, q Î ç 0, ÷
è 2ø
1
Now, cos q = 1 - sin2 q Þ q = cos -1 1 - x2 x
q
æ 1 ö
Þ -1 -1 2 -1 2
sin x = cos 1 - x = sec ç ÷ 1– x
ç 2 ÷
è 1- x ø
x
tan q =
1 - x2
x x
Þ q = tan -1 Þ sin -1 x = tan -1
1 - x2 1 - x2
x æ 1 - x2 ö
Þ sin -1 x = tan -1 = cot -1 ç ÷
1 - x2 ç x ÷
è ø
1 æ x ö æ 1 - x2 ö 1
= sec -1 = tan -1 ç -1
÷÷ = cot ç ÷ = cosec -1 æç ö÷ , 0 < x < 1
ç ç x ÷ èxø
1 - x2 è 1-x
2
ø è ø
æ p ö
Let sin -1 x = q q Î ç - , 0 ÷ , Then x = sinq
è 2 ø
Þ cos q = 1 - x2 Þ cos ( -q ) = 1 - x 2
æ 1 ö
Þ q = - cos -1 1 - x2 Þ sin-1 x = - cos-1 1 - x2 = - sec -1 ç ÷
ç 2 ÷
è 1- x ø
x x x
Again, tan q = Þ q = tan -1 Þ sin -1 x = tan -1
1 - x2 1 - x2 1 - x2
123
JEE-Mathematics
x æ 1 - x2 ö é -1 -1 æ 1 ö ù
Þ sin -1 x = tan -1 = -p + cot -1 ç ÷ êQ tan x = -p + cot ç ÷ , x < 0ú
1 - x2 ç x ÷ ë èxø û
è ø
1 æ x ö æ 1 - x2 ö 1
= - sec
-1
= tan-1 ç -1
÷÷ = -p + cot ç ÷ = cosec -1 æç ö÷ , - 1 < x < 0
2 ç 2 ç x ÷ èxø
1- x è 1- x ø è ø
æ 5p ö æ æ - 3 öö
Illustration 6. Find the value of cos–1 ç - sin ÷ + sin–1 ç cos çç sin 1 ÷÷
è 6 ø ç
è è 2 ÷ø ÷ø ?
æ 1ö 1 2p p 5p
Solution Given expression = cos–1 ç - ÷ + sin–1 = + =
è 2 ø 2 3 6 6
æ 5 -1 ö
Illustration 7. Find the value of cos–1 (cos(2 tan–1 çç ÷ )) ?
÷
è 10 + 2 5 ø
5 -1 p
5 -1 sin p
4 10
Solution = = = tan
10 + 2 5 10 + 2 5 p 10
cos
4 10
æ p ö æ pö p
\ Given expression = cos–1 ç cos 2 . ÷ = cos–1 ç cos . ÷ =
è 10 ø è 5ø 5
x æ 4x ö
Illustration 8. Solution of 2 tan–1 + sin–1 ç ÷ = p is
2 è 4 + x2 ø
ì -1 x
æ ö ï-p - 2 tan 2 : x < -2
ç 2x ÷ ï
ï -1 x
ç 2 ÷ í 2 tan : -2 £ x £ 2
Solution sin–1 ç 2 ÷
= ï 2
x
çç 1 + æç ö÷ ÷÷ ï -1 x
è è 2ø ø ï p - 2 tan 2 : x>2
Node-1\Target-2020-21\1.JEE(M+A)\Module\Enthusiast\English\Maths\EM-1\18.ITF\Th.
x x
If – 2 £ x £ 2, then 4 tan–1 =p i.e. = 1 i.e. x=2
2 2
If x > 2, then p = p
\ x Î [2, ¥)
124
Inverse Trig onometric Fu nctions
Illustration 9. Number of solutions of the equation sin –1 |x| = |cos–1 x|, is
p
2
\ 1 solution
O x
p
–
2
Illustration 10. The number of real solution of the equation 1 + cos 2x = 2 sin (sinx), –p £ x £ p is
–1
y= |cos x|
y=sin–1(sinx)
–p
–p/2 p/2 p
æ 3 x2 ö b
Illustration 11. The range of the function f(x) = cot -1 çç 2 ÷
÷ is (a, b], then the value of is
a
è 1+ x ø
3 x2 p æ 3x2 ö p
Solution 0£ < 3 Þ < cot -1 çç ÷£
2÷
1+ x2 6 è 1+ x ø 2
-1 p
So Sn = tan (n + 1) -
4
Node-1\Target-2020-21\1.JEE(M+A)\Module\Enthusiast\English\Maths\EM-1\18.ITF\Th.
p
lim Sn =
n®¥ 4
Illustration 13. No. of solution of the equstion 2(sin-1 x)2 - (sin-1 x) - 6 = 0 is ...
Solution sin–1x = y
2y2 – y – 6 = 0
y = – 1.5, 2
sin–1x = – 1.5, 2
Hence, sin–1 x = –1.5 (rejecting 2)
Only one solution
125
JEE-Mathematics
p
Illustration 14. The range of the function y = is
sin-1 x
-p p
Solution £ sin-1 x £
2 2
1 æ -2 ù é 2 ö
Þ -1
Î ç -¥, ú È ê , ¥ ÷
sin x è p û ëp ø
p
Þ Î ( -¥, -2] È [ 2, ¥ )
sin-1 x
Illustration 15. The number of integral roots of the equaiton. sin -1 sin x = cos -1 ( cos x ) in x Î [ 0, 4 p ] are
–1
cos (cos x)
sin -1 sin x
p p
Solution 2
p x
x 2p 3p 4p
p p 3p 2p
2 2
Node-1\Target-2020-21\1.JEE(M+A)\Module\Enthusiast\English\Maths\EM-1\18.ITF\Th.
126
Inverse Trig onometric Fu nctions
SINGLE CORRECT
é æ 3ö æ 1 öù
1. tan -1 ê cos ç 2 tan -1 ÷ + sin ç 2cot -1 ÷ ú is
ë è 4ø è 2 øû
(A) not real (B) equal to p / 4 (C) greater then p / 4 (D) less than p / 4
æ1 4ö
2. ( -1
)
If x = sin 2 tan 2 and y = sin ç tan
-1
÷ , then
è2 3ø
æ 1ö
3. The three angles given by a = 2 tan
-1
( )
2 - 1 , b = 3sin
-1 1
2
1
+ sin -1 ç - ÷ and g = cos-1 .
è 2ø 3
(A) a > b (B) b > g > a (C) a > g (D) None of these
4. Let f (x) = a + b cos-1 x (b > 0). If domain and range of f(x) are the same set then (b – a) is equal to
1 2 2
(A) 1- (B) +1 (C) 1- (D) 2
p p p
æ p 3p ö é p 3p ù ìp 3p ü
(A) ç , ÷ (B) ê , (C) í , ý (D) None of these
è4 4 ø ë 4 4 úû î4 4 þ
If ëé sin x ûù + ëé cos x ûù = 0 , where ‘x’ is a non-negative real number and [.] denotes the greatest integer
-1 -1
6.
-
( -
) ( )
Node-1\Target-2020-21\1.JEE(M+A)\Module\Enthusiast\English\Maths\EM-1\18.ITF\Ex.
The set of values of a for which x + ax + sin x - 4 x + 5 + cos x - 4 x + 5 = 0 has at least one solution
2 1 2 1 2
7.
is
(A) (-¥ , - 2p ] È [ 2p , ¥) (
(B) -¥, - 2p È ) ( 2p , ¥ )
(C) R (D) None of these
127
JEE-Mathematics
9. Which one of the following quantities is negative ?
-1
(
(A) cos tan (tan 4) ) (
-1
(B) sin cot (cot 4) )
-1
(
(C) tan cos (cos 5) ) (
-1
(D) cot sin (sin 4) )
5p 2
( ) + ( cot x )
-1 2 -1 2
10. If tan x = , then x equals
8
( ) + ( cos ec x )
-1 2 -1 2
11. The maximum value of sec x is equal to
p2 p2 5p 2
(A) (B) (C) p 2 (D)
2 4 4
æ æ -2 ö ö
12. tan ç arc tan ç ÷ + arc tan ( 5 ) ÷ equals
è è 3 ø ø
-1 2 6 +1
13. The value of cos - cos -1 is equal to
3 2 3
p p p p
(A) (B) (C) (D)
3 4 2 6
14.
-1 -1
If 3 sin x = -p - sin 3 x - 4 x , then
3
( )
é 1ù é1 ù 1
(A) x Î ê -1, - ú (B) x Î ê ,1ú (C) x £ (D) None of these
2 ë û ë2 û 2
1 1
(A) (B) – (C) 2 -1 (D) no finite value
2 2
-1 æ 2r -1 ö
n
p p
-1
(A) tan 2
n
( ) -1
(B) tan 2 -
n
( ) 4
-1
(C) tan 2
n+1
( ) -1
(D) tan 2
n+1
(
- ) 4
128
Inverse Trig onometric Fu nctions
n æ r - r -1 ö
17. å sin -1
ç ÷
ç r ( r + 1) ÷ is equal to
r= 1
è ø
(A) tan
-1
( n ) - p4 (B) tan
-1
( )
n +1 -
p
4
(C) tan
-1
( n) (D) tan
- 1
( n +1 )
p
Point P(x, y) satisfying the equation sin x + cos y + cos ( 2xy ) =
-1 -1 -1
18. lies on
2
(A) the bisector of the first and third quadrant
(B) bisector of the second and fourth quadrant
19. (sin -1 x)2 + (sin -1 y)2 + 2(sin -1 x)(sin -1 y) = p2 , then x2+y2 is equal to -
(A) 1 (B) 3/2 (C) 2 (D) 1/2
1 2
20. If sin–1 + sin–1 = sin–1x, then the value of x is-
3 3
( 5 - 4 2) ( 5 + 4 2) p
(A) 0 (B) (C) (D)
9 9 2
21. The solution of the inequality (tan -1 x)2 - 3 tan -1 x + 2 ³ 0 is -
(A) ( -¥, tan1] È [ tan 2, ¥ ) (B) ( -¥, tan1] (C) ( -¥, - tan1] È [ tan2, ¥) (D) [ tan2, ¥)
22. If a1,a2,a3........an is in A.P. with common difference d, then
é d d d ù
tan ê tan-1 + tan-1 + .... + tan -1 ú
ë 1 + a1a 2 1 + a 2a 3 1 + a n -1a n û is equal to-
(n - 1)d (n - 1)d nd a n - a1
(A) a a (B) 1 a a (C) 1 + a a (D) a
1 + n + 1 n 1 n n + a1
n
2r + 1
23. lim å tan -1 is equal to -
n ®¥
r =1 r + 2r 3 + r 2 + 1
4
p 3p p p
(A) (B) (C) (D) –
4 4 2 8
Node-1\Target-2020-21\1.JEE(M+A)\Module\Enthusiast\English\Maths\EM-1\18.ITF\Ex.
¥
æ r((r + 1)!) ö
24. å tan -1
ç 2 ÷ is equal to -
r =0 è (r + 1) + ((r + 1)!) ø
p p
(A) (B) (C) cot–13 (D) tan–12
2 4
-1 æ 1 ö -1 -1 3
25. The value of tan çè tan2A÷ø + tan (cot A) + tan (cot A) for 0 < A < (p/4) is -
2
(A) 4 tan–1(1) (B) 2 tan–1(2) (C) 0 (D) none
129
JEE-Mathematics
-1
é ì p 1 -1 æ a ö ü ì p 1 -1 æ a ö üù
26. The value of ê tan í + sin çè ÷ø ý + tan í - sin çè ÷ø ýú , where ( 0 < a < b), is -
ë î 4 2 b þ î4 2 b þû
b a b2 - a 2 b2 - a 2
(A) (B) (C) (D)
2a 2b 2b 2a
27. Value of k for which the point (a, sin–1a)(a > 0) lies inside the triangle formed by x + y = k with co-ordinate axes is -
æ p ö æ æ pö æ pöö æ p ö
(A) ç 1 + , ¥÷
è ø
(B) ç - çè 1 + ÷ø , çè 1 + 2 ÷ø ÷ø (C) çè -¥ , 1 + ÷ø (D) (–1–sin1, 1+sin1)
2 è 2 2
æ 2x2 + 3 ö 5
28. Solution set of the inequality sin -1 ç sin ÷ £ p - 2 is -
2
è x +1 ø
(A) (-¥ , 1) È (1 , ¥) (B) [–1, 1] (C) (–1, 1) (D) (-¥ , -1] È [1 , ¥)
æ 1 ö æ 1 ö
(A) all values of x (B) x Î ç 0, ÷ (C) x Î ç ,1÷ (D) x = 0.75
è 2ø è 2 ø
ì-1 æ4ö
-1 æ 2 öü a
30. If the numerical value of tan ícos ç ÷ + tan ç ÷ ý is , then (where a and b are coprime number)
î è5ø è 3 øþ b
æ 5 - 1ö æ 5 + 1ö
(A) x 2 = çè ÷ (B) x 2 = çè ÷
2 ø 2 ø
æ 5 - 1ö æ 5 - 1ö
(C) sin(cos -1 x) = çè ÷ (D) tan(cos -1 x) = çè ÷
2 ø 2 ø
32. Identify the pair(s) of functions which are identical -
1 - x2 1
(A) y = tan (cos–1x) ; y = (B) y = tan (cot–1x) ; y =
x x
Node-1\Target-2020-21\1.JEE(M+A)\Module\Enthusiast\English\Maths\EM-1\18.ITF\Ex.
x
(C) y = sin (arc tan x) ; y = (D) y = cos (arc tan x) ; y = sin (arc cot x)
1 + x2
33. The sum of the infinite terms of the series -
æ 3ö æ 3ö æ 3ö
cot -1 ç 12 + ÷ + cot -1 ç 22 + ÷ + cot -1 ç 32 + ÷ + ........... is equal to -
è 4ø è 4ø è 4ø
3p
(A) tan–1(1) (B) tan–1(2) (C) tan–1(3) (D) - tan -1 3
4
130
Inverse Trig onometric Fu nctions
æ cos-1 x ö æ sin -1 x ö
Consider the two equations in x ; (i) sin ç ÷ =1 (ii) cos ç ÷ =0
è y ø è y ø
Comprehension – 2
Let h1(x) = sin–1(3x – 4x3) ; h2(x) = cos–1(4x3 – 3x) & f(x) = h1(x) + h2(x)
-1
when x Î [–1, ] ; let f(x) = a cos–1x + bp ; a, b Î Q
2
h1(x) = p sin–1x + qp ; p, q Î Q
h2(x) = r cos–1x + sp ; r, s Î Q
Let C1 be the circle with centre (p, q) & radius 1 & C2 be the circle with centre (r, s) & radius 1.
4. p + r + 2q – s =
(A) 0 (B) 1 (C) 2 (D) 4
9 -3
(A) (B) 6 (C) (D) none of these
2 2
131
JEE-Mathematics
Match the Column
Following questions contains statements given in two columns, which have to be matched. The statements in
Column-I are labelled as A, B, C and D while the statements in Column-II are labelled as p, q, r and s. Any given
statement in Column-I can have correct matching with ONE statement in Column-II.
7. Column-I Column-II
æ 33p ö
(A) sin–1 çè sin ÷ (p) –2p/7
7 ø
æ 46p ö
(B) cos–1 çè cos ÷ (q) 2p/7
7 ø
æ æ -33p ö ö
(C) tan–1 çè tan çè ÷÷ (r) 3p/7
7 øø
æ æ -46p ö ö
(D) cot–1 ç cot çè ÷÷ (s) 4p/7
è 7 øø
8. Column-I Column-II
(A) sin(tan x)–1
(p) x
x
(B) cos(tan–1x) (q)
x2 + 1
1
(C) sin(cot–1(tan(cos–1x))); x Î(0,1] (r) 2
x +1
Following question contains statements given in two columns, which have to be matched. The statements in
Column-I are labelled as A, B, C and D while the statements in Column-II are labelled as p, q, r and s. Any given
statement in Column-I can have correct matching with ONE OR MORE statement(s) in Column-II.
p
(A) xy + yz + zx = 1, then (p) k =
2
(B) x + y + z = xyz, then (q) k=p
Node-1\Target-2020-21\1.JEE(M+A)\Module\Enthusiast\English\Maths\EM-1\18.ITF\Ex.
7p
(D) x = y = z and xyz ³ 3 3 , then k can be equal to (s) k =
6
132
Inverse Trig onometric Fu nctions
INTEGER/SUBJECTIVE TYPE QUESTIONS
10. Find the domain of definition the following functions.
(Read the symbols [ * ] and { * } as greatest integers and fractional part functions respectively)
2 1 1
(a) f(x) = cos -1 (b) f(x) = + 2arc sin x +
2 + sin x x x-2
-1 éx ù 1 æ x - 3ö
(c) ecos x
+ cot -1 ê - 1ú + ln{x} (d) f(x) = sin -1 ç - log10 ( 4 - x )
ë 2 û 2 è 2 ÷ø
1 - sin x
(e) f(x) = + cos -1 (1 - {x} )
log5 (1 - 4x2 )
æ 3 - 2x ö
(f) f(x) = 3 - x + cos -1 ç + log6 ( 2|x|-3 ) + sin -1 (log2 x )
è 5 ÷ø
11. Draw the graph of the following functions :
(a) f(x) = sin–1(x + 2) (b) g(x) = [cos–1x], where [ ] denotes greatest integer function.
(c) h(x) = –|tan–1(3x)|
æx 1 ö
12. Express f(x) = arc cos x + arc cos ç + 3 - 3x2 ÷ in simplest form and hence find the values of
è2 2 ø
æ 2ö æ 1ö
(a) fç ÷ (b) fç ÷
è 3ø è 3ø
1 1 p 1
13. Prove that : 3 tan -1 + tan -1 = - tan -1
4 20 4 1985
14. If a and b are the root of the equation x2 + 5x – 49 = 0, then find the value of cot(cot–1a + cot–1b).
15. Let cos–1x + cos–1(2x) + cos–1(3x) = p. If x satisfies the cubic ax3 + bx2 + cx – 1 = 0, then find the value of
a + b + c.
16. Solve the following equations
(a) sin -1 x = cos -1 x + sin -1 ( 3x - 2) (b) sin -1 x + sin -1 (1 - x ) = cos -1 x
1 - a2 1 - b2 x2 - 1 2x 2p
(c) 2 tan -1 x = cos -1 2
- cos -1 2 a > 0, b > 0 (d) cos -1 + tan -1 =
1+ a 1+ b 2
x +1 2
x -1 3
Find all values of k for which there is a triangle whose angles have measure tan -1 ç ÷ , tan -1 ç
æ1ö æ1 ö
17. + k ÷ and
è 2ø è2 ø
æ1 ö
tan -1 ç + 2k ÷ .
è2 ø
Node-1\Target-2020-21\1.JEE(M+A)\Module\Enthusiast\English\Maths\EM-1\18.ITF\Ex.
æ 10
ö a
18. The value of tan ç
è
å cot
k =1
-1
(1 + k + k 2 ) ÷ = where a and b are coprime, find the value of (a + b).
ø b
19. If the total area between the curves f (x) = cos –1(sin x) and g(x) = sin–1(cos x) on the interval [–7p, 7p] is A, find
the value of 49A. (Take p = 22/7)
20. Solve the following system of inequations
4 (tan–1 x)2 – 8 tan–1 x + 3 < 0 and 4 cot–1 x – (cot–1 x)2 – 3 > 0
21. Find the set of values of ‘a’ for which the equation 2 cos –1 x = a + a2(cos–1 x)–1 posses a solution.
133
JEE-Mathematics
-1 æ 1ö
1. Find the principal value of : cos çè - ÷ø
2
-1 æ 2p ö
2. Find the principal value of : sin ç sin ÷
è 3ø
-1 æ 7p ö
3. Find the principal value of : tan çè tan ÷ø
6
æ -1 5 ö
4. Evaluate: cos çè sec ÷
3ø
æ
-1 æ 3 öö
5. Evaluate: sin ç 2cos ç - ÷ ÷
è è 5 øø
-1 -1 æ 1ö -1 æ 1ö
7. Find the value of: tan (1) + cos çè - ÷ø + sin çè - ÷ø
2 2
-1 1 1 31
8. Prove that : 2 tan + tan-1 = tan -1
2 7 17
æ - 1 ö
9. If sin ç sin
1
+ cos-1 x ÷ = 1 , then find the value of x .
è 5 ø
-1 x -1 x +1 p
10. If tan + tan -1 = , then find the value of x .
x-2 x+2 4
11. Find the value of sin -1 (cos(sin -1 x)) + cos -1 (sin(cos-1 x)).
-1 -1 1- x 1+ x
12. Prove that cos x = 2sin = 2cos -1 .
2 2
-1 é æ -1 1ö ù
Node-1\Target-2020-21\1.JEE(M+A)\Module\Enthusiast\English\Maths\EM-1\18.ITF\Ex.
ìï 1 + cos x + 1 - cos x üï p x
tan -1 í ý= -
îï 1 + cos x - 1 - cos x ïþ 4 2
3p
if p < x < .
2
x
If y = cot -1 ( cos x ) - tan -1 ( cos x ), prove that sin y = tan
2
15. .
2
134
Inverse Trig onometric Fu nctions
ANSWER KEY
BE GINNER 'S BOX-1
1
1. (D) 2. (B) 3. 4. (A) 5. (C) 6. (1526)
3
7. (B) 8. (B) 9. (C) 10. (C) 11. (B) 12. (B)
13. (A) 14. (C)
15. (i) D : x e R R : [p/4 , p)
æ pö ì pü é p 2p ù épù
(ii) D: x Î ç np, np + ÷- íx x = np + ý n Î I ; R: ê , ú - ê2ú
è 2ø î 4þ ë3 3 û ë û
é pö æ p pù
(iii) D: xÎR R : ê0 , ÷ (iv) D: xÎR R : ç- , ú
ë 2ø è
2 4 û
15 1 4 3 4
1. (a) (b) (c) (d) 1 (e) (f)
8 10 5 2 5
p p p p -p
2. (a) (b) (c) p – 2 (d) (e) - (f)
6 6 6 3 4
2p
(g)
3
13p
3. (B) 4. (D) 5. (B) 6. (B,C,D) 7. (D) 8.
7
9. 8p - 21 10. (53) 11. (B) 12. (B) 13. (D)
BE GINNER 'S BOX-3
2 6 ì1, if a >0 -2
2. - 3. í 4. (A,D) 5. (A ) 7.
5 î-1, if a<0 5 5
8. –1/3 9. 204/325 10. (ABD)
BE GINNER 'S BOX-4
4. B 5. D
BE GINNER 'S BOX-5
1
Node-1\Target-2020-21\1.JEE(M+A)\Module\Enthusiast\English\Maths\EM-1\18.ITF\Ex.
1. (a ) (b) 1 (c) 3
5
1 3 1 1 3 1
2. (i) x = (ii) x =3 (iii) x = 0, ,- (iv) x = (v) x = , y =1
2 7 2 2 10 2
3. (a) 1/4 (b) ± 1, –1 ± 2,1± 2 (c) no solution
4. (D) 5. (A) 6. (B) 7. p/4 8. p 9. p /4
p p
10. (a) (b) 11. cot–1 (a1) 12. arc tan (x + n) - arc tan x
4 4
p é2n + 5ù
13. 14. arc cot ê ú
2 ë n û
135
JEE-Mathematics
EXERCISE-1
Que. 1 2 3 4 5 6 7 8 9 10
A ns. C B B B C D D A D A
Que. 11 12 13 14 15 16 17 18 19 20
A ns. D C D A D B C D C C
Que. 21 22 23 24 25 26 27 28 29 30
A ns. B B A B A C A B CD ABC
Que. 31 32 33
A ns. AC ABCD BD
EXERCISE-2
Comprehe nsion # 2 : 4. A 5. C 6. A
l Match the Column 7. (A) ® (q), (B) ® (s), (C) ® (q), (D) ® (r)
æ3 ù
(e) x Î (–1/2, 1/2), x ¹ 0 (f) ç , 2ú
è2 û
Node-1\Target-2020-21\1.JEE(M+A)\Module\Enthusiast\English\Maths\EM-1\18.ITF\Ex.
y
p/2 p y
3
2 0 x
p/2
12. (a) –3 –2 –1 0 x (b) 1
(c)
-p/2 - p/2
–1 cos3 cos2 0 cos1 1 x
p æ 1ö p
1 3 . (a) (b) 2cos –1 çè ÷ø –
3 3 3
136
Inverse Trig onometric Fu nctions
1 1 a-b
1 6 . (a) x = 1, (b) x = 0, (c) x = (d) x = 2 – 3 OR 3
2 2 1 + ab
11
17. k= 18. 0011 19. 3388
4
æ 1 ö
20. x = ç tan , cot 1 ÷ 21. a Î [ -2p, p ] - { 0}
è 2 ø
NCERT CORNER
2p p p 3 24
1. 2. 3. 4. 5. -
3 3 6 5 25
3p 1 1 p p
6. (0) 7. 9. 10. ± 11. 13.
4 5 2 2 4
******
Node-1\Target-2020-21\1.JEE(M+A)\Module\Enthusiast\English\Maths\EM-1\18.ITF\Ex.
137
138
JEE-Mathematics
IMPORTANT NOTES
Node-1\Target-2020-21\1.JEE(M+A)\Module\Enthusiast\English\Maths\EM-1\18.ITF\Ex.